Sei sulla pagina 1di 151

Faculty of Actuaries

Institute of Actuaries

EXAMINATION
12 April 2005 (pm)

Subject ST5
Finance and Investment
Specialist Technical A
Time allowed: Three hours
INSTRUCTIONS TO THE CANDIDATE
1.

Enter all the candidate and examination details as requested on the front of your answer
booklet.

2.

You have 15 minutes at the start of the examination in which to read the questions.
You are strongly encouraged to use this time for reading only, but notes may be made.
You then have three hours to complete the paper.

3.

You must not start writing your answers in the booklet until instructed to do so by the
supervisor.

4.

Mark allocations are shown in brackets.

5.

Attempt all 7 questions, beginning your answer to each question on a separate sheet.

6.

Candidates should show calculations where this is appropriate.

AT THE END OF THE EXAMINATION


Hand in BOTH your answer booklet, with any additional sheets firmly attached, and this
question paper.
In addition to this paper you should have available the 2002 edition of the
Formulae and Tables and your own electronic calculator.

ST5 A2005

Faculty of Actuaries
Institute of Actuaries

(i)

(ii)

State the taxation factors and influences on these factors that need to be
considered when selecting investments which maximise after tax returns.
Outline the three main systems of corporation tax.

[6]

[3]
[Total 9]

You are a fund manager managing a portfolio of international equities. As part of a


presentation to trustees you have presented the following table:
Region

Fund Return

Market Return

US
Japan

+5.5%
+9.4%

+5.1%
+8.3%

Market returns are stated with reference to the S&P 500 for the US and Topix for
Japan.
A trustee points out that the Dow Jones Index rose by 6% over the period and the
Nikkei rose by over 10% and therefore the fund has actually underperformed in these
regions.
Outline the points you would make in your response.

[8]

An insurance company has a line of shares held within its shareholders funds. The
shares are in a quoted investment management organisation to whom the insurance
company has outsourced the management of its policyholders funds and, due to good
recent performance, these funds are experiencing significant positive cashflows. The
rest of the shareholders funds are invested in bonds and property. The insurance
company wishes to transfer half the line of stock to the policyholders global equity
fund at a discount to the prevailing bid price and has been required by the regulator to
commission an independent valuation of the discounted share price at which the
transfer should take place.
(i)

State the key objectives of financial regulation.

[3]

(ii)

Explain why the company might wish to transfer the shares to the
policyholders funds.

[3]

(iii)

(iv)

ST5 A2005

(a)

Outline the principal factors to be considered in determining an


appropriate price.

(b)

List the additional information you would consider in setting the price.
[3]

Set out the other business issues that should be considered by the committee
responsible for agreeing the discount on behalf of the policyholders.
[11]
[Total 20]

(i)

Describe how to construct a zero coupon yield curve.

[5]

(ii)

Define the par yield for a bond.

[2]

(iii)

You have the following information for three conventional gilts:


Gilt 1: coupon 6.75%, maturity 1 year, price 102.01
Gilt 2: coupon 9.50%, maturity 2 years, redemption yield 4.90%
Gilt 3: coupon 7.75%, maturity 3 years, price 110.41
Assume that gilts pay annual coupons and that a coupon has just been paid.
Calculate the spot yields for the next 3 years.
[5]
[Total 12]

The table below contains information about a pension fund and index returns. The
benchmark for the fund is an investment that is 50% equities and 50% bonds.
31/12/03
31/12/04
Contributions
Values (m) Values (m)
(m)

Equities
Bonds
Cash
Equity Total Return Index
Equity Index Yield
Bond Total Return Index
Bond Index
Base Rate

600.0
350.0
50.0
1,000.0
3.00%
1,220.0
275.0
3.50

700.0
450.0
50.0
1,115.0
3.12%
1,299.3
280.5
4.00

12.0
63.0
1.0

Investment
Income
(m)
15.0
25.0
1.0

(i)

Defining all formulae used and stating any assumptions made, analyse the
performance of the fund.
[11]

(ii)

Comment on the results of your analysis and any investment features that the
data may suggest.
[9]
[Total 20]

In Nestl v. National Westminster Bank plc [1994] the judge considered that
decisions of trustees should be judged by modern portfolio theory and that the risk
level of the whole portfolio is considered rather than just individual investments.
(i)

Outline how Behavioural Finance challenges this view.


[10]

(ii)

ST5 A2005

Give examples of how trustees could make poor manager selection decisions
based on these behaviours.
[4]
[Total 14]

PLEASE TURN OVER

Spenser & Michael (S&M) is a UK-based food retailer which is well known
throughout Europe and the Far East but largely unknown in the United States of
America. S&M have tried to borrow US$500m at a fixed rate of interest in US
dollars but the interest rates S&M can secure are prohibitively expensive.
S&M have been quoted a five-year fixed rate of 6% per annum for a sterling
denominated loan.
BIM is a US-based food retailer and would like to borrow the sterling equivalent of
US$500m over five years at a fixed rate of interest in sterling. Like S&M, BIM has
been quoted prohibitively expensive rates for a sterling loan.
BIM has been quoted a five-year fixed rate of 5.25% per annum for a US$
denominated loan.
At the time of the transaction, the yield on five-year government bonds is 5.25% in
the UK and 4.75% in the US.
You are the head of the currency swap desk of a global investment bank.
(i)

Describe, using the above information, the factors that will influence the
design of a five-year currency swap.

[8]

(ii)

Design a five-year currency swap for S&M and BIM that will net the global
investment bank 0.45% per annum over the life of the swap while ensuring
that S&M and BIM have no exchange rate risk on their exchange of interest
payments.
[4]

(iii)

Describe the risks that the global investment bank takes on in structuring this
swap for BIM and S&M and how the global investment bank can hedge these
risks.
[5]
[Total 17]

END OF PAPER

ST5 A2005

Faculty of Actuaries

Institute of Actuaries

EXAMINATION
April 2005

Subject ST5
Finance and Investment
Specialist Technical A
EXAMINERS REPORT

Introduction
The attached subject report has been written by the Principal Examiner with
the aim of helping candidates. The questions and comments are based around
Core Reading as the interpretation of the syllabus to which the examiners are
working. They have however given credit for any alternative approach or
interpretation which they consider to be reasonable.

M Flaherty
Chairman of the Board of Examiners
28 June 2005

Faculty of Actuaries
Institute of Actuaries

Subject ST5 (Finance and Investment Specialist Technical A)

April 2005

Examiners Report

Generally candidates were able to make reasonable attempts at most questions.


Whilst there were some incidences of weak knowledge of bookwork, those candidates
who failed usually scored poorly on application and or higher level skill aspects of
questions. The following comments are written to assist candidates with
understanding what the examiners are looking for. The solutions should not be
regarded as complete but as a guide to the amount of knowledge and level expected.
Other reasonable points and interpretations were awarded appropriate marks.
Q1. This was a bookwork question and well answered. Candidates failed to pick up
marks primarily under the influences section of (i). Marks were also lost for
incomplete explanations in (ii) especially under classical and split-rate where thye
shareholder s position was often ignored.
Q2. Another bookwork question that was done well. Points tended to be lost because
of incomplete explanation of points.
Q3. This question was poorly done. Whilst most could attempt (i), the remainder of
the question seemed beyond many candidates as they were unable to demonstrate that
they understood what the issues were and how they needed to be dealt with.
Q4. This was well done although it was interesting that continuous interest rates were
determined rather than yearly rates. Calculation of the price of the 2-year bond
required the use of yearly rates given the information supplied. Where an error was
made in the calculations this was penalised only once and provided this was carried
through marks were awarded for subsequent calculations.
Q5. This type of question gets set on a regular basis and the examiners are always
surprised at the spread of answers that they encounter and the failure to use specific
pieces of information supplied, particularly in relation to part (ii) of this question.
Candidates should realise that all information is there for a purpose usually to assist
them in framing their answers in an appropriate manner. The solution shows the
answer that is most likely to be encountered in the real world. However no penalties
were applied for the more unrealistic assumptions that were often used regarding the
timing of contributions and income. The examiners believe that full analysis should
be undertaken in a question of this nature as illustrated in the solution.
Candidates are encouraged to set out the formulae that they use as this will ensure
that when a mistake is made marks can still be awarded for subsequent calculations.
While many candidates scored well in (i), few candidates were able to attain
significant amounts of marks in (ii). Explanations tended to lack the detail necessary
to attain the marks on offer.
Q6. This question was done well being predominantly bookwork.
Q7. Of all the questions this was the one that candidates had most difficulty with.
Answers in (i) were basically what the examiners view as brain dumps and were
not focused on the question as set. Accordingly poor marks were awarded. Very few
good answers were seen for (ii) but many managed to collect around half marks in
(iii) despite poor attempts at the earlier sections.

Page 2

Subject ST5 (Finance and Investment Specialist Technical A)

April 2005

Examiners Report

The factors to be considered are:


The total rate of tax on an investment.
How the tax is split between different components of the investment return.
The timing of tax payments.
Whether the tax is deducted at source or has to be paid subsequently.
The extent to which tax deducted at source can be reclaimed by the investor.
To what extent losses or gains can be aggregated between different investments or
over different time periods for tax purposes.
Influences on these factors are:
tax rates on capital gains
tax rates on income/dividends
exemptions and allowances against tax
rules on particular assets
investor s own status
investor s financial position
investment vehicle s tax efficiency

(ii)

Classical: A company s profits are taxed twice: once in the hands of the
company and once in the hands of the shareholder. The shareholder may be
subject to tax on dividends and/or capital gains arising from increases in the
share price.
Split-rate: Similar to the classical system excepting that different tax rates
may be levied on retained profits and distributed profits. The system might be
used in conjunction with a system that taxes investor s income and capital
gains at different rates.
Imputation: A system designed to enable a company s profit to be taxed once
rather than twice. Dividends paid from taxed profits are paid to shareholders
together with a tax credit. The rules vary greatly and can be quite complex but
it is often the case that the tax credit received is sufficient to offset the tax due
on the net dividend. Also, lower taxed investors can often reclaim the tax
credit.

Page 3

Subject ST5 (Finance and Investment Specialist Technical A)

April 2005

Examiners Report

While the Dow Jones and Nikkei are indices that are often quoted they are not
particularly representative of their markets. The Dow Jones Index is based on 30
shares and the Nikkei is based on 225 shares.
The S&P 500 and Topix are more broadly based being based on 500 and 1,100 shares
respectively. They are therefore more representative.
Both the DJ and N are unweighted indices, that means that every company has the
same impact on the index.
Both the T and S&P are weighted indices, the weights being the market capitalisations
of the companies, this means that larger companies have more influence on the index
than small companies.
The constituents of the N have changed little since inception whereas the Japanese
stock market has changed significantly.
The DJ is made up of 30 industrial stocks and therefore ignores the impact of other
areas e.g. financials.
The constituents of both the S&P & T are revised regularly and encompass the full
range of companies operating in their respective markets.
Therefore the S&P & T are better indices to use when looking at fund performance as
they better represent the universe from which fund managers can select stocks.

(i)
to correct market inefficiencies and to promote efficient and orderly
markets
to protect consumers of financial products
to maintain confidence in the financial system
(ii)
to remove volatility in the insurance companies solvency margin
to reduce the inherent investment risk of the shareholders investments
get a better price than a public sale
to avoid negative publicity/speculation arising from a public sale
(iii)

(a) Any regulatory or recommended limit on the maximum (and minimum


discount).
Discounts applying to similar private placements or block trades.
(b) Normal daily volume in the shares and transaction prices.

(iv)

Page 4

Being seen to avoid conflicts of interest (who instigated transaction and why).

Subject ST5 (Finance and Investment Specialist Technical A)

April 2005

Examiners Report

Treating customers fairly (suitability of investment; appropriateness of terms)


and so satisfy regulator s Principles of Business.
Is the asset manager involved in the transfer and will they be paid for
managing the holdings thereafter.
Any lock in period that restricts the shareholders dumping the stock at a later
date.
Respective size of the policyholders fund into which the stock is being
transferred,
its benchmark and mandate, concentration of holdings, nature of underlying
investments (direct or commingled funds)
Who are other shareholders
how free is float?
Volatility of share price and impact on portfolio risk

(i)

Starting point is to take the market prices of conventional bonds (e.g. gilts) for
a range of possible maturities.
Starting at the shortest maturity, T1 say, use the observed market price and
solve for the yield. This yield is an approximation for the zero coupon rate for
maturity T1, called R1, say.
Using the next shortest maturity conventional bond maturing at T2, again take
the observed price and using R1 solve for the forward rate starting at T1 for
the period T2 T1. Now solve for the spot rate R2.
Repeat using the next maturity conventional bond until the longest maturity
bond has been used. This fixes the longest spot rate at the maturity of the
longest bond.
Plot the spot rates R(T) against T

to arrive at the zero coupon yield curve.

(ii)

This is the coupon rate that the bond would be required to make the theoretical
value of the bond equal to its nominal value under the prevailing pattern of
zero coupon interest rates.

(iii)

S1: 102.01 * (1 + S1) = 100 + 6.75

S1 = 4.65%

No price given for 2 year gilt so need to calculate:


P2: P2 * (1 + 0.049)2 = 9.5 * (1 + 0.049) + 100 + 9.5

P2 = 108.56

Now solve for S2


108.56 * (1 + S2)2 = 9.5 * (1 + 1f1) + 109.5 where 1f1 is the 1 year forward
rate starting in 1 years time
Page 5

Subject ST5 (Finance and Investment Specialist Technical A)

April 2005

Examiners Report

(1 + S2)2 = (1 + S1) * (1 + 1f1)


S2 = 4.91%
S3: 110.41 * (1 + S3)3 = 7.75 (1 + 2f1)2 + 7.75 * (1 + 1f2) + 107.75
(1 + 2f1)2 = (1 + S3)3 / (1 + S1)

(1 + 1f2) = (1 + S3)3 / (1 + S2)2

S3 = 3.94%
If continuous rates used the answers are S1=4.54%, S2=4.80% and S3=3.86%.
P2=108.32(using continuous rates) is not correct but we do not penalise in other calculations.

(i)

Contributions occur halfway through the period.


Generalised formula for return (each class):
MV(1) = MV(0)*(1+i) + C*(1+i/2) using the normal approximation for a half
year s interest.
[MV(1)-MV(0) - C] / [MV(0) + C/2] = Return (%)
Equity return = (700-600+12)/[600

12/2] = 18.86%

Bond return = (450-350-63)/[350 + 63/2] = 9.70%


Cash return = (50-50+1)/[50 1/2] = 2.02%
Fund total return = (1,200-1000-50)/[1,000 + 50/2] = 14.63%
Bond Index return = 1,299.3/1,220

1 = 6.50%

Equity Index return = 1,115/1,000

1 = 11.50%

Benchmark return = [50 * 1.065 + 50 * 1.1150]/100

1 = 9.0%

In allocating performance the fund weights should be the average weights


based on the denominator of the fund returns but initial market values used as
weights would not be marked as wrong
Equity Asset Contribution = (594/1,025
= 0.20
Bond Asset Contribution = 0.32

Page 6

0.50) * (11.50

9.0)

Subject ST5 (Finance and Investment Specialist Technical A)

April 2005

Examiners Report

Cash Asset Contribution = 0.34 (as it is not part of the index assume return =
actual, although 3.5 to 4.0 could be used and then a stock contribution would
have to be calculated)
Equity Stock Contribution = 594/1,025 * (18.86

11.50) = 4.27%

Bond Stock Contribution = 381.5/1,025 * (9.70

6.50) = 1.19%

The sum of the parts = 5.64% compared with actual 5.63% due to rounding
error.

(ii)

Asset allocation positive in both bond and equity decisions but holding cash a
negative.
Stock selection very positive for both bonds and equities.
Yield on equity investments was 15/594 = 2.53% compared with 3.12% for
index so investment strategy in equities is capital growth orientated or the
timing of purchases and sales was such that a full year of dividend income has
not been received.
Yield on bonds was 25/381.5 = 6.55% compared with 4.5% [ difference
between bond index and total return index] for index. This suggests a
portfolio away from the index possibly in lower quality corporate bonds or
emerging market debt.
Cash return very poor given base rates.
Fund manager should be asked to comment on the strategy.
Strategy should be compared with mandate.

(i)

Behavioural Finance looks at the mental biases and decision-making errors


that affect financial decisions
The following challenge the view of the judge:
Anchoring past experience adjusted to allow for evident differences to
current conditions herd instinct
Prospect Theory how decisions are made when faced with risk and
uncertainty
How questions are framed, especially if of a structured response variety
How risk aversion changes as time-frame or number of opportunities increases
Dislike of negative events influences perceived probability of outcome
Ease of imagining apparent likelihood increases with detail

Page 7

Subject ST5 (Finance and Investment Specialist Technical A)

April 2005

Examiners Report

Ease of bringing to mind


Overconfidence hindsight bias
- confirmation bias
mental account count individual gains/losses rather than netting them
primary/recency effect first or last option to be presented more influential
range of options effect more choice means less decision
status quo preference to keep things as they are
regret effect don t do anything in order to create no regret
ambiguity effect premium for rules

(ii)

Suitable examples/conclusions include:


Recent past performance.
The what, not the how, and subject to manipulation.
Size and reputation.
No one was sacked for buying IBM.
Image.
Advertising, familiarity with brand.
Client list.
Assume the others do the due diligence.

(i)

S&M (quoted + 0.75% over sterling government bonds) has a poorer credit
rating than BIM (quoted +0.50% over US$ government bonds) as evidenced
by the spread over corresponding five-year government rates.
To avoid any exchange rate risk on the exchange of interest rate payments,
S&M will need to borrow at the five-year fixed rate of 6% per annum in
sterling and receive payments at a rate of 6% per annum fixed for five years
from the global investment bank as part of the swap design.
To avoid any exchange rate risk on the exchange of interest rate payments,
BIM will need to borrow at the five-year fixed rate of 5.25% per annum in
US$ and receive payments at a rate of 5.25% per annum fixed for five years
from the global investment bank as part of the swap design.
The difference between the US$ payments by S&M to the global investment
bank and the sterling payments by BIM to the global investment bank provides
the margin for the global investment bank.
However, the global investment bank will probably want to charge a higher
rate of interest to S&M than to BIM to reflect the poorer credit rating of the
former.

(ii)

Page 8

In arriving at its fee the global investment bank would probably wish to tilt the
charges to BIM and S&M to reflect their relative credit ratings. Thus the

Subject ST5 (Finance and Investment Specialist Technical A)

April 2005

Examiners Report

global investment bank may wish to charge S&M a somewhat wider margin
than BIM.
One possibility would be to charge S&M a US$ five-year fixed rate of 5.80%
(1.05% over five-year US$ government bonds) and BIM a sterling five-year
fixed rate of 5.90% (0.95% over five-year sterling government bonds).
(iii)

The global investment bank is left with a residual foreign exchange risk on
each exchange of interest payments between the two parties.
This risk could be hedged by forward foreign exchange contracts.
The global investment bank is also left with credit risk.
Credit risk could be hedged using credit derivatives.

Page 9

Faculty of Actuaries

Institute of Actuaries

EXAMINATION
13 September 2005 (pm)

Subject ST5
Finance and Investment
Specialist Technical A
Time allowed: Three hours
INSTRUCTIONS TO THE CANDIDATE
1.

Enter all the candidate and examination details as requested on the front of your answer
booklet.

2.

You have 15 minutes at the start of the examination in which to read the questions.
You are strongly encouraged to use this time for reading only, but notes may be made.
You then have three hours to complete the paper.

3.

You must not start writing your answers in the booklet until instructed to do so by the
supervisor.

4.

Mark allocations are shown in brackets.

5.

Attempt all 9 questions, beginning your answer to each question on a separate sheet.

6.

Candidates should show calculations where this is appropriate.

AT THE END OF THE EXAMINATION


Hand in BOTH your answer booklet, with any additional sheets firmly attached, and this
question paper.
In addition to this paper you should have available the 2002 edition of the
Formulae and Tables and your own electronic calculator.

ST5 S2005

Faculty of Actuaries
Institute of Actuaries

Define the role of a custodian, and list the services that they might offer in addition to
document safe keeping.
[4]
[Total 4]

(i)

Explain which of the two types of bond portfolio switches are more likely to
be carried out by the following type of investor:
(a)

A liability driven investor, investing in bonds to match liabilities of a


particular duration.

(b)

An unconstrained investor, who is investing in bonds to maximise


returns relative to LIBOR.
[2]

(ii)

Explain why an analysis of reinvestment rates might mean an investor favours


investment in a 10 year bond with a 5% coupon over investing in a 20 year
bond with a 10% coupon, when the yield on the longer bond is 0.5% p.a.
higher.
[2]

(iii)

Set out the processes involved in assessing whether or not there is a potential
yield difference between a 10 year AA rated corporate bond and a 10 year
government bond that can be exploited.
[4]
[Total 8]

(i)

Describe the critical difference between a mortgage backed security and a


regular fixed income security.
[3]

(ii)

A collateralised loan obligation (CLO) is backed by a pool of residential


mortgages and divided into three investment classes W, X and Y. Scheduled
and pre-payment principal repayments are channelled to class W investors
until that group of investors has been completely repaid. Thereafter,
scheduled and pre-payment principal repayments are channelled to class X
investors until that group of investors has been completely repaid. When class
W and class X investors have been fully repaid, all remaining principal
repayments are directed to class Y investors.

ST5 S2005

(a)

State, with reasons, which class of investor bears the most pre-payment
risk.

(b)

Rank in order of increasing duration the three investment classes and


give a reason for your answer.

(c)

Describe, using a simple example, how the par value of the three
classes influences the pre-payment risk of class Y investors.
[5]
[Total 8]

(i)

Describe the main aims of the field of Behavioural Finance as applied to


investment management.

[4]

(ii)

Explain how Behavioral Finance arguments might be used by an equity


investment manager to explain why they hold a persistently underperforming
stock in a portfolio rather than selling out of the position and re-investing in an
alternative stock.
[5]

(iii)

Outline how the irrational actions suggested by Behavioral Finance might be


eliminated.
[3]
[Total 12]

(i)

Describe, with reasons, the types of merger that a UK based retail bank might
consider with a similar sized organisation operating in the UK.
[8]

(ii)

Describe the other factors that would need to be considered if the proposed
merger was to be with an organisation domiciled in another EU country. [6]
[Total 14]

The trustees of a pension fund have appointed an investment manager to invest in


global equities and bonds. The investment manager s performance will be compared
with a benchmark equally divided between two representative broad market indices.
The trustees wish to impose limits on the investment manager s ability to deviate
from the 50/50 proportions to which the benchmark will rebalance on a quarterly
basis.
(i)

Explain why the investment manager might actively deviate from the
benchmark asset allocation on a short-term basis.

[4]

(ii)

List the factors that should be considered by the trustees in setting the limits.
[3]

(iii)

Outline appropriate constraints on these factors.

ST5 S2005

[5]
[Total 12]

PLEASE TURN OVER

You are the investment manager of a life assurance company that has substantial
assets under management and invests a significant proportion of these assets in
alternative investments and derivative-type structures.
A sales person from an investment bank approaches you regarding the purchase of a
complex derivative product.

(i)

List the key questions regarding the derivative product that you would ask the
sales person.
[5]

(ii)

Describe the factors which you would consider in assessing the characteristics
of the derivative in terms of its fit with the life assurance company s
investment portfolios.
[5]
[Total 10]

(i)

Explain why equities are usually analysed in sector or industry groupings. [5]

(ii)

(a)

Discuss the advantages and disadvantages of this sub division.

(b)

Suggest, with reasons, two possible alternative groupings.


[5]

(iii)

State the features that characterise each of the following economic groups:
General Industries
Consumer Goods
Utilities

[6]
[Total 16]

You are an investment consultant to the trustees of a pension scheme. You are given
the following total return data for the fund and the indices included in the benchmark.

UK Equities
Overseas Equities
UK Bonds
Overseas Bonds

Fund Returns
Year 1 Year 2

Year 3

Index Returns
Year 1 Year 2

Year 3

10.1%
14.2%
6.4%
3.2%

17.6%
13.2%
4.1%
1.5%

8.5%
13.5%
7.2%
8.1%

16.0%
1.0%
5.4%
3.1%

2.5%
3.2%
3.1%
1.8%

5.4%
2.1%
4.5%
2.8%

The asset mix of the fund and of the benchmark at the start of year 1 was as follows:

UK Equities
Overseas Equities
UK Bonds
Overseas Bonds

ST5 S2005

Fund

Benchmark

45%
30%
15%
10%

60%
20%
10%
10%

You may assume that there is no rebalancing at any time. No contributions were paid
in and no benefits were paid out during the period.
(i)

Describe the principal sources of deviation between a fund and its benchmark.
[2]

(ii)

Calculate:
(a)
(b)
(c)

the total return for the fund


the benchmark return
the respective contributions from the items in (i)
[10]

(iii)

Comment on the results of your calculations.

END OF PAPER

ST5 S2005

[4]
[Total 16]

Faculty of Actuaries

Institute of Actuaries

EXAMINATION
September 2005

Subject ST5 Finance and Investment


Specialist Technical A
EXAMINERS REPORT

Faculty of Actuaries
Institute of Actuaries

Subject ST5 (Finance and Investment Specialist Technical A)

Sept 2005

Examiners Report

A custodian ensures that financial instruments are housed under a proper system that
permits investment for proper purposes with proper authority.
The following services may also be provided: income collection, tax recovery, cash
management, securities settlement, foreign exchange, stock lending.

(i)

(a)

Anomaly switches, as these are less likely to alter the duration match
of the liabilities.

(b)

Policy switches, as these allow the investor the freedom to attempt to


maximise return based on the investor s view of future changes in the
level and shape of the yield curve.

(ii)

The investor may take the view that despite the 0.5% pa higher yield on the 20
year bond, the need for a greater level of reinvestment during the bond s term
means that the additional return is insufficient compensation for the risk that
reinvestment terms might worsen. This is particularly likely if yields are
currently considered to be high relative to historical levels.

(iii)

First an estimate of the risk premium for the AA rated bond will be needed,
with a view to how this might change over the length of the anomaly switch.
This view will take into account default risk and other factors affecting the
yield such as lack of liquidity, coupon and any tax differences.
This will be used to estimate the additional yield that could be obtained
through changes in the risk premium relative to benchmark government bond
yields.
An allowance should be made for transaction costs at both ends of the switch.
Based on the additional yield, a decision as to whether to proceed with the
switch can be made.

(i)

The borrowers underlying the mortgages in a mortgage backed security have a


right to (but are under no obligation to) repay the mortgage to the lender
at any time during the life of the mortgage at its face value.
This introduces pre-payment risk for the holders of mortgage backed
securities.

Page 2

Subject ST5 (Finance and Investment Specialist Technical A)

(ii)

(a)

Sept 2005

Examiners Report

The W investment class.


All principal repayments go in the first instance to the class W
investors; a rush of prepayments will impact this class of investors the
most.

(i)

(b)

W, X and Y. Class W will be repaid first because of the structure of


the CLO and is likely to have the shortest duration followed in order
by class X and class Y.

(c)

Class Y investors bear very little pre-payment risk if the ratio of the
par value of the securities is 5:2:1 but class Y investors bear significant
pre-payment risk if the ratio of the par values is reversed to read 1:2:5.

BF is based on the idea that a variety of mental biases and decision making
errors affect financial decisions.
Central to BF is the psychology of how and why financial decisions are made.
Analysis within this field is believed to have some predictive qualities and the
findings used to help the proponents in their own decision making.

(ii)

Analysts and investment managers can be unreasonably drawn to a stock


based on understanding and belief in the company s business model and/or
management. This affection can lead them to persistently discount bad
news about the stock.
Investment managers can be highly confident in their own abilities. Over
confidence can lead the practice of managers using good news on a stock to
reinforce their position whilst putting any bad news down to the ignorance of
other market participants.
Managers will be reticent to take responsibility and crystallize a loss having
defended the position to date.

(iii)

Institute a level of discipline in the investment process to reduce bias without


stifling invention and ideas.
Hold periodic peer reviews of the portfolio/decisions.
Create a culture, which actively encourages challenge and debate.
Introduce hard code rules and risk controls to take out some of the human
element.

Page 3

Subject ST5 (Finance and Investment Specialist Technical A)

Sept 2005

Examiners Report

Needs to cover:
(i)

Description of all the types of merger and the reasons why they are instigated
(and why not) as per Unit 8 section 4.

(ii)

Barriers to European integration as per Unit 10, section 9.

(i)

Aim is to outperform benchmark


To outperform you need to be different
Extent of difference will be controlled to minimise risk of underperformance
(and potential termination of mandate)
Justified because:
Markets and asset classes are not perfectly correlated
Asset class and country returns may be predictable because:
Valuations can drift away from fair value
Investors may be slow to incorporate new information
Risk premiums change over time among global markets
Structural barriers exist across global markets
Some market participants (e.g. central banks) may not be motivated by
profits

(ii)

Performance target
Style of management
Correlations between assets
Cash flows and income
Costs of rebalancing
Rebalancing frequency
Scope to use derivatives
Risk Tolerance

(iii)

Not all factors will have limits placed on them as they are not manageable e.g.
cash flows and income but they will be taken into account when considering
other limits i.e. rebalancing to ensure efficient management.
Performance target limit the under performance by more than a set
percentage over a defined period, out performance also needs to be reviewed
(too high risk?).
Style of management

permit only a limited percentage in non-style assets.

Correlations between assets


particular magnitude.

Page 4

monitor for increases or decreases of a

Subject ST5 (Finance and Investment Specialist Technical A)

Costs of rebalancing

Sept 2005

Examiners Report

set a limit on management costs.

Rebalancing frequency place a limit on asset allocation shifts other than


due to cash flow or income.
Scope to use derivatives
of assets.

limit exposure to derivatives to certain percentage

Risk tolerance the funding level, corporate sponsor s financial status and
asset class risk/return expectations will all have different influences at
different times depending on how the risk of each is viewed. e.g. a well
funded scheme with a large surplus may be prepared to consider wider
parameters than one which is less well funded when the company sponsor is
financially sound.

(i)

What market, instrument or other underlying asset drives the performance of


the derivative?
What is the formula for the magnitude of the cash flows likely to arise under
the derivative contract?
What is the timing of those cash flows?
What is the likelihood of receipt or payment of the cash flows?
What is the liquidity of the contract?

(ii)

The factors would be:


return expected what is it going to contribute to overall returns
taxation are there any implications for returns or other aspects of the
portfolio
applicable regulatory valuation rules
what rules may apply and will
they cause complications
accounting treatment of the derivative
is it treated as capital or are
there income aspects that need to be allowed for
integration with the existing portfolio
does it make strategic sense
its effect on risk
what does it do to the overall portfolio s position

(i)

Grouping by industry
To reduce the number of factors that have to be taken into account when
analysing the share.
Quite a lot of industry statistics are available and are usually grouped by
industry.

Page 5

Subject ST5 (Finance and Investment Specialist Technical A)

Sept 2005

Examiners Report

Methods of raising finance are often similar within a group.


Experience similar labour problems, costs of raw materials, consumer demand,
economy will affect them similarly.
Financial statements are prepared in a similar fashion and will use similar
jargon.
No other factor has proven to correlate to shares as closely as industry
statistics.
(ii)

Disadvantages
Tend not to look at companies between sectors, only companies within
sectors.
Some shares don t move with their industry.
Advantages
Can become expert in one sector and understand it very well.
We can decide which factors affect a share price within an industry, then
analyse companies with this in mind.
Could group by:
Large cap/small cap
valuations tend to reflect growth/maturity/financial
strengths differently for large and small cap stocks.
Growth/Value
reflects the different universes and economic/market drivers
operating on these companies.
Exporters/Importers
reflects earnings from domestic or overseas influences
and impact of different regions economic growths and currencies.
Any two sensible suggestions should earn marks.

(iii)

Page 6

See Core reading Unit 3 pages 2 & 3.

Subject ST5 (Finance and Investment Specialist Technical A)

(i)

Sept 2005

Examiners Report

The overall investment performance of a fund can be divided into:


Sector or asset class selection: the extent to which the fund s proportions
in the various sectors will have affected performance.
Stock selection: within any one sector, have the selected stocks performed
better or worse than the sector as a whole?

(ii)

Fund Benchmark Fund Ret Index Ret


UK Eq
O seas Eq
UK Bond
O seas Bond
Total

45%
30%
15%
10%
100%

60%
20%
10%
10%
100%

26.24%
25.14%
14.20%
6.63%
22.14%

19.06%
17.04%
18.07%
14.57%
18.11%

Asset Class
Selection
0.14%
0.11%
0.00%
0.00%
0.25%

Stock
Selection
3.23%
2.43%
0.58%
0.79%
4.29%

Total

3.09%
2.32%
0.58%
0.79%
4.04%

There is a rounding error of 0.01%.


(iii)

The fund has out performed by 4.03%. Asset allocation was poor due to under
weighting UK equities and over weighting overseas equities. Equity stock
selection in both markets was very good whilst bond selection was poor in
both markets. Bond performance was consistently poor for both classes in
each year. UK equity performance was consistently good but overseas equity
performance was volatile with the third year accounting for more than the
stock selection added value in total.

END OF EXAMINERS REPORT

Page 7

Faculty of Actuaries

Institute of Actuaries

EXAMINATION
30 March 2006 (pm)

Subject ST5 Finance and Investment


Specialist Technical A
Time allowed: Three hours
INSTRUCTIONS TO THE CANDIDATE
1.

Enter all the candidate and examination details as requested on the front of your answer
booklet.

2.

You have 15 minutes at the start of the examination in which to read the questions.
You are strongly encouraged to use this time for reading only, but notes may be made.
You then have three hours to complete the paper.

3.

You must not start writing your answers in the booklet until instructed to do so by the
supervisor.

4.

Mark allocations are shown in brackets.

5.

Attempt all 7 questions, beginning your answer to each question on a separate sheet.

6.

Candidates should show calculations where this is appropriate.

AT THE END OF THE EXAMINATION


Hand in BOTH your answer booklet, with any additional sheets firmly attached, and this
question paper.
In addition to this paper you should have available the 2002 edition of the
Formulae and Tables and your own electronic calculator.

ST5 A2006

Faculty of Actuaries
Institute of Actuaries

You are a trustee to a mature but underfunded retirement benefit scheme which has
investments in a wide range of properties. Discuss the appropriateness of passive
management of the property assets within the fund.
[8]

In relation to portfolio construction:


(a)

Define prospective and retrospective tracking errors.

(b)

Discuss the use of prospective and retrospective tracking errors as measures of


risk.

(c)

State why the two measures of risk might differ.


[9]

You are advising the government of a country on how to set up a tax system. The
chief minister wants to tax investment income at a higher rate than earned income,
and to exempt capital gains from tax. His aims are to encourage entrepreneurial
activity and investment and to redistribute wealth from the rich to the poor. Explain
the flaws and unintended consequences of his proposed approach.
[8]

(i)

(ii)

Outline the key principles underlying the relevant legislation relating to


providers of financial services and their clients.

[7]

Describe the points that should be covered by a Statement of Investment


Principles .
[4]
[Total 11]

The following information is provided on a non-UK equity fund.


US
Value at 31.12.04 m.
Value at 31.12.05 m.
Income in period m.
Net New Investment in period m.
Local Index 31.12.04
Local Index 31.12.05
Currency Rate at 31.12.04 v
Currency Rate at 31.12.05 v
Index Yield at 31.12.05 (%)

250
300
5
-10
100
120
1.90
1.75
2.5

Japan
100
140
2
-5
325
400
190
200
1.0

Asset
Europe Asia
100
96
3
10
200
185
1.40
1.50
2.0

50
70
4
0
210
296
1.90
1.75
3.0

Cash

Total

0
5
0
5
100
100
1.00
1.00
4.0

500
611
14
0

The benchmark index is weighted 50% U.S., 30% Europe, 15% Japan and 5% Asia.

ST5 A2006 2

(i)

Evaluate the performance of the fund through a full attribution analysis by


market stating any assumptions that you make.
[15]

(ii)

(a)

Comment on the returns and the results.

(b)

Suggest any investigations that you might undertake to better


understand the results.
[5]
[Total 20]

A fund manager operates an OEIC whose investment strategy is to identify pairs of


shares in the same industry or market sector one of which is forecast to rise in value
while the other is forecast to fall in value over a one-year time period. The manager
buys the shares that he forecasts will rise in value. He borrows the shares that he
expects will fall in value, sells them in the market hoping to buy them back later at a
lower price, and then return the borrowed shares to the lender.
One of the current pairs of shares that the manager has in his portfolio is
from the pharmaceutical sector. He expects PharmaUP to rise in value over the
next year and he expects PharmaDOWN to fall in value over the next year.
(i)

Discuss the risks in this strategy under the following headings:


(a)

The pairing of two shares in the same industry or sector and identifying
one as expected to rise in value while the other as expected to fall in
value.
[6]

(b)

Borrowing shares and selling them in the market hoping to buy them
back at a lower price.
[5]

(c)

The costs of implementing the investment strategy.

(d)

The impact of the majority of investors redeeming their holdings in the


fund on the shareholders who do not wish to redeem their holdings at
some point before the one-year investment time horizon has expired.
[4]

[4]

(ii)

Describe how the manager might implement his pairing strategy to eliminate
some of the risks in (i) (a).
[2]

(iii)

How would the manager reduce the risk to the investors who do not redeem
their holdings in (i) (d).
[1]
[Total 22]

ST5 A2006 3

PLEASE TURN OVER

A mobile phone company requires additional financing and its treasury team has
noticed that it has an asset in the form of future payments by subscribers who are
required to make payments under their monthly contracts prior to the minimum period
ending. The team has approached your bank in order to set up an asset backed
security (ABS) issue via a special purpose vehicle (SPV). The company will transfer
the asset to the SPV in return for a payment of 500m, and the SPV will then issue an
ABS in 3 tranches as shown below, backed by these assets. The company will
purchase the equity tranche from the SPV.
Tranche
Senior
Mezzanine
Equity

Credit rating

Tranche size

AA
BB
n/a

400m
50m
50m

(i)

Explain why the company might wish to use its asset in such a way.

(ii)

Describe the tranched structure, including why there is an equity tranche, and
why the company might wish to purchase it.
[6]

(iii)

Explain the key risks to a purchaser of the mezzanine tranche, and describe
situations when these risks might result in losses.
[5]

(iv)

Comment on how your answer to (iii) would change if the asset value was
based on expected contract receipts for all existing contracts over the next 5
years, including payments after the minimum contract period expires.
[5]
[Total 22]

END OF PAPER

ST5 A2006 4

[6]

Faculty of Actuaries

Institute of Actuaries

EXAMINATION
April 2006

Subject ST5
Finance and Investment
Specialist Technical A
EXAMINERS REPORT
Introduction
The attached subject report has been written by the Principal Examiner with the aim of
helping candidates. The questions and comments are based around Core Reading as the
interpretation of the syllabus to which the examiners are working. They have however given
credit for any alternative approach or interpretation which they consider to be reasonable.

M Flaherty
Chairman of the Board of Examiners
June 2006

Faculty of Actuaries
Institute of Actuaries

Subject ST5 (Finance and Investment Specialist Technical A)

April 2006

Examiners Report

Comments
The solutions should not be taken as comprehensive. There are a number of additional points
that can be made in certain questions and these were awarded appropriate marks. There are
also a number of different solutions that can be derived for question 5 and these were also
awarded appropriate marks despite the fact that it would be unusual to see them in practice.
However alternative solutions tended not to give the same degree of information as those
shown here and consequently marks in subsequent sections of the question were lost.
In general candidates did bookwork well but failed to carry this through in the application
parts of questions and only the better candidates scored anything like reasonable marks in
higher skills parts. Roughly half the paper relates to application and candidates often gave
us bookwork rather than applying their knowledge to the problem in hand. Questions 2, 3, 6
and 7 were good examples of this with candidates on average scoring under 50% of the
marks available. Question 4 was well done by most candidates being bookwork. Question 1,
despite being bookwork, was not as well done as other bookwork sections possibly because of
the context in which it was framed. As has been the case in the past we were disappointed
with answers to question 5 despite the fact that arithmetical errors are not penalised because
we are more concerned that candidates understand the methods and assumptions that they
are using. In particular candidates show too little working, fail to use all the information
that is provided and do not think widely enough. Consequently they did not score well in the
second part of the question.

Page 2

Subject ST5 (Finance and Investment Specialist Technical A)

April 2006

Examiners Report

Passive management aims to track the performance of a specified benchmark index


with no active investment risk.
This approach limits downside risk of poor manager performance.
It also removes scope to benefit from good manager performance.
An under funded scheme may find active management more appealing as a means to
catch up some lost capital.
Fees are generally low however this generalised point is not likely to be true for
property where transaction and tax costs are significant.
Does a suitable index exist covering the wide range of properties held?
Property indices have numerous problems
availability of prices.

non homogeneity, timing, frequency and

It may be possible to gain better matching of an index through a derivative


instrument, but note that derivatives are often based on an investable index.
Active management aims to provide a return in excess of a specified benchmark by
taking investment risk within the portfolio.
It is difficult to correctly identify managers who will consistently outperform.
Fees are generally higher than for passive management.

Solution not split into (a) (c) as answers unlikely to conform to split of question.
Prospective tracking error is an estimation of the scale of volatility of prospective
relative performance given the current portfolio and benchmark.
It is usually expressed in a number of basis points per year.
It provides an estimate of the aggregate amount of investment risk within a portfolio
at the time it is calculated.
It is calculated using a quantitative model and depends on assumptions including: the
likely future volatility of individual stocks and markets relative to the benchmark and
correlations between different stocks and markets.
Tracking errors make no distinction between upside risk and downside risk.
In this regard it may not fit well with the trustees attitude to risk which will likely be
skewed to seek positive performance with a high probability whilst minimising the
probability of a significant negative performance.

Page 3

Subject ST5 (Finance and Investment Specialist Technical A)

April 2006

Examiners Report

Retrospective tracking error is defined as the standard deviation of realised past


annualised relative performance.
It is also usually expressed in a number of basis points per year.
It seeks to provide an ex post summary measure of investment risk based on the
volatility in the monthly (or quarterly) pattern of realised returns.
Prospective and retrospective tracking error will differ because:
Prospective assumes a static and unchanged portfolio and benchmark.
It also relies on many assumptions that are unlikely to be achieved in practice.
Whereas retrospective tracking error is based on the actual portfolio and on the
performance of that portfolio against the benchmark

By exempting capital gains from tax and taxing investment income at a higher rate
than earned income, capital gains will become the most tax efficient method of wealth
creation and investment income will become the least tax efficient method of gaining
wealth.
Potentially this will encourage companies to retain profits rather than distribute them,
leading to higher levels of corporate investment
but it may also reduce the average rate of return for new investment opportunities
as opportunities that would previously have been ignored will now be developed.
The higher level of corporate investment will not necessarily increase the overall
growth rate of the economy as there will be less distributed income reinvested by
companies and individuals.
Individuals will be encouraged to pay themselves in the form of capital gains rather
than income, if they are able to do so, to minimise their tax liability.
This may lead to the creation of schemes that convert income into capital gains, which
across the economy as a whole are unlikely to result in any net wealth creation.
It is worth noting that wealthier individuals will typically be least dependent on earned
income, and will therefore be most able to structure their affairs to minimise their tax
liabilities.
Conversely poorer individuals are unlikely to have such flexibility and will therefore
pay more tax than if they were remunerated through capital gains.
Overseas investors are similarly likely to be able to structure their affairs in a way that
minimises tax.

Page 4

Subject ST5 (Finance and Investment Specialist Technical A)

April 2006

Examiners Report

Attempts to prevent this (e.g. withholding taxes) may inadvertently result in the
country s assets commanding a lower purchase price, impacting on the wealth of the
nation as and when domestic assets are sold to overseas investors.

(i)

The key principles are:


Integrity

high standards and fair dealing.

Skill, care and diligence.


Market practice

observe proper market conduct.

Information about customers


objectives.
Information for customers
dealings.
Conflicts of interest
disclosure to client.
Customer assets
assets.

comprehensive and timely information in all

should be avoided where possible, but otherwise full

proper safeguard, segregation and management of all

Financial resources
risks.

firm must have sufficient assets to meet its business

Internal organisation
keeping.
Relations with regulator
(ii)

details of circumstances and investment

properly trained and supervised staff, proper record

open and co-operative manner.

A Statement of Investment Principles should set out:


Who is taking what decisions and why this structure has been selected.
Fund s investment objective.
Fund s planned asset allocation strategy, asset class projected returns and how
strategy has been arrived at.
What mandates have been given to all advisers and managers.
Nature of fees structures for all advisers and managers, reason behind why set
of structures selected.

Page 5

Subject ST5 (Finance and Investment Specialist Technical A)

(i)

US
Japan
Europe
Asia
Cash
Total
(ii)

April 2006

Examiners Report

Assumption: net new investment occurs in middle of period. Simplification


(1+i/2) allowed. [Whilst candidates were not asked for formulae, it would be
good practice to write out what formulae were used as this shows how the
table of results is arrived at. Since no marks were being awarded for
formulae, the report does not specify them.]
Fund Rtn Bmark Rtn
24.49%
30.29%
46.15%
16.92%
-13.33% -13.67%
40.00%
53.03%
0.00%
0.00%
22.81%
16.23%
(a)

Curr Rtn
8.57%
-5.00%
-6.67%
8.57%
0.00%
1.96%

Alloc
0.00
0.03
2.99
1.84
0.00
4.86

Curr Stock Alt Rtn


0.00 -2.90
0.24
-0.35 5.85
0.45
-0.14
0.86 0.07
0.33 -1.30
0.4
0.00 0.00
0
0.85 1.71 22.20%

Stock
-3.14
5.62
-0.07
-1.30
0.00
1.10

Fund outperformed by 6.58%/5.97%.


Stock selection added 1.71/1. 1%.
Asset allocation added 4.86%.
Currency added 0.85% of the 4.86%.
Under weight Europe and over weight Asia were positive asset
allocation contributions.
Good stock selection for Japan, offset by poor US and Asia stock
selection.

(b)

Income needs investigating as looks like growth stocks in US but value


stocks in rest of world
check style.
Check if any currency hedging in place as Japan return very high vis a
vis benchmark

(i)

(a)

The manager s valuation model may prove incorrect and if it does


investors will lose money on the transaction as the share bought falls in
value while the share shorted rises in value.
The manager takes on a very high degree of stock specific risk with
this strategy.
In particular, PharmaUP may suffer a significant fall in price due to a
factor that is specific to PharmaUP.
For example, an announcement that the Federal Drug Administration
in the US is withdrawing its approval for one of the drugs that
contributes significantly to PharmaUP s profits as a result of the deaths

Page 6

Subject ST5 (Finance and Investment Specialist Technical A)

April 2006

Examiners Report

of several individuals using the drug while the majority of other


pharmaceutical companies may see their share price generally rise.
(b)

There is an assumption in the strategy that the manager will be able to


borrow the shares for a period of one year and this may not be possible
in practice.
If the lender of the shares recalls the loan of the shares the manager
may have to unwind his strategy at a point in time when the share
bought has fallen in value while the share shorted has risen in value
forcing investors to realise a loss

(c)

The strategy will not be profitable unless the difference between the
rise in value of PharmaUP and the fall in value of PharmaDOWN is
sufficient to cover all the following costs:
Bid-Offer spreads
any impact on market depth
stamp duty and other levies
commission/brokerage fees
The manager will have to fund the cost of any dividends arising on the
shares borrowed and this adds to the minimum return necessary to
make the strategy profitable.
These costs will be offset to some extent on the interest income arising
on the cash generated by shorting the shares.

(d)

(ii)

If some investors redeem their shares during the one-year holding


period at a point where the value of PharmaUP is less than its value on
the date the transaction is initiated
or the value of PhamaDOWN is above its value on the date the
transaction is initiated
this may distort the relationship between the pair of stocks and make
the entire trade unprofitable
if the price of PharmaDOWN rises unexpectedly on the forced buying
in operation
and the price of PharmaUP falls unexpected from the forced sale of
shares.

To reduce stock specific risk, the manager might buy in a sector of the market
he forecasts will outperform over the next year and go short in a sector of the
market that will under perform in the same period.
This could be achieved using exchange traded funds.

(iii)

The manager could insist on a minimum investment term of one-year so as not


to interrupt investment strategies with redemptions.

Page 7

Subject ST5 (Finance and Investment Specialist Technical A)

(i)

April 2006

Examiners Report

The asset is being used as collateral to back a loan issue.


The asset is intended to provide higher security than an unsecured loan issued
by the company.
As the ABS is issued through a SPV, purchasers are protected against other
claims made against the company.
These factors will provide lenders (purchasers of the ABS) with more
certainty of repayment than purchasers of an unsecured loan stock, reducing
the company s cost of capital.
The yield on the ABS will therefore be lower than the yield on an unsecured
loan stock of similar term and structure.
Furthermore, it may not be possible for the company to borrow funds in the
market at a realistic rate (e.g. the issuer credit rating is weak).
Using the asset in this way would potentially lead to a more efficient balance
sheet structure, as the asset is illiquid and not receiving any investment return.
However, this must be offset against the transaction costs of the ABS issue,
and the issue may have negative implications for the company s credit
rating as it will reduce asset and income cover for existing unsecured
creditors.

(ii)

The equity tranche has been created to protect purchasers of higher tranches
against the value of the asset being lower than calculated in the prospectus.
Defaults in the customer contracts would first be set against the equity tranche,
then the mezzanine tranche and finally the senior tranche.
The default risks are reflected in the credit ratings, and the equity tranche is
unrated.
The ABS has been structured in this way to reduce the cost of borrowing.
The spread (additional yield) increases steeply as the credit rating declines.
The company may wish to purchase the equity tranche if it is confident that
the asset has been conservatively valued,
and therefore does not want to give up excess returns to other parties.
The nominal return on the equity tranche is likely to be well over 10%, and
therefore higher than the company s average (and possibly marginal) cost of
capital.
In practice it may be difficult to market the equity tranche as contract and
customer retention is to some extent under the control of the company, and

Page 8

Subject ST5 (Finance and Investment Specialist Technical A)

April 2006

Examiners Report

some level of company participation in the issue would be needed to avoid


moral hazard.
(iii)

There is some switching risk, although this should be low as subscribers will
have a penalty if they terminate contracts prior to the minimum period ending.
If the nature of the marketplace changes then this may result in losses.
There is some default risk if subscribers are unable to meet their remaining
contract payments. This risk would be higher if the contract automatically
terminates in certain situations (e.g. redundancy) and losses are likely to
increase in an economic downturn.
Most of the risk is likely to be restricted to the equity tranche.

(iv)

The key factor determining whether a loss will occur is how actual switches
and defaults compare to what has been assumed in the asset valuation.
The switching risk would increase materially as there will be a high expected
number of switches once the minimum contract period ends and the number of
switches is likely to remain at a moderate level for the duration of the 5 year
period
The default risk will also be higher although this is likely to remain broadly
stable over the 5 year period.
A possible exception to this pattern might be if subscribers with lower default
risk were cherry picked by a competitor.
In contrast to the previous scenario, it is likely that the mezzanine tranche will
have significant exposure to these risks, in addition to purchasers of the equity
tranche.
These additional risks would, all other things being equal, result in a higher
spread on the ABS particularly on the mezzanine tranche.

END OF EXAMINERS REPORT

Page 9

Faculty of Actuaries

Institute of Actuaries

EXAMINATION
7 September 2006 (pm)

Subject ST5
Finance and Investment
Specialist Technical A
Time allowed: Three hours
INSTRUCTIONS TO THE CANDIDATE
1.

Enter all the candidate and examination details as requested on the front of your answer
booklet.

2.

You have 15 minutes at the start of the examination in which to read the questions.
You are strongly encouraged to use this time for reading only, but notes may be made.
You then have three hours to complete the paper.

3.

You must not start writing your answers in the booklet until instructed to do so by the
supervisor.

4.

Mark allocations are shown in brackets.

5.

Attempt all 7 questions, beginning your answer to each question on a separate sheet.

6.

Candidates should show calculations where this is appropriate.

AT THE END OF THE EXAMINATION


Hand in BOTH your answer booklet, with any additional sheets firmly attached, and this
question paper.
In addition to this paper you should have available the 2002 edition of the
Formulae and Tables and your own electronic calculator.

ST5 S2006

Faculty of Actuaries
Institute of Actuaries

You are employed by a national government to oversee all aspects of the national debt
management policy.
Discuss how you might use an asset liability model, outlining the inputs, in this task.
[8]

An equity portfolio has a target beta of 1.2 relative to the market index.
(i)

Explain the term beta, and the meaning of a value of 1.2.

[2]

(ii)

Describe the investigations you would make to determine the value added by
the fund manager to the portfolio, assuming full data is available.
[3]

(iii)

List reasons why the performance of the portfolio might differ from that of the
index.
[3]

(iv)

The beta of the portfolio has moved away from the target of 1.2. Explain how
individual stocks can be used to correct the beta.
[2]
[Total 10]

A futures exchange is considering setting up a new European equity futures contract


and there are several existing indices on which the contract could be based.
(i)

Outline the attributes of the relevant indices that you would consider in
deciding which index to use, and name two potentially suitable indices.

[6]

Two years after launch of the new contract, it consistently ranks in the top five
contracts on the exchange. The exchange is however concerned that take-up in
the asset management community has been lower than initially anticipated.
It believes that this reflects the widespread usage of restrictions in mandates for
European equity managers.
(ii)

Describe the types of restrictions that might be made on a European equity


mandate, giving suitable examples.
[4]

The exchange decides to expand its range of contracts by splitting the European
equity contract into constituents that can be combined to re-create the existing
contract.
(iii)

ST5 S2006

List four classifications by which the contract may be sub-divided, and two
other variations on the original contract that might expand its appeal.
[3]
[Total 13]

ABC plc has a share price of 57 pence and paid a dividend of 2 pence per share in the
previous twelve months. It is considering issuing:
a convertible debenture, with a zero coupon, which is convertible into ordinary
shares on the basis of 150 shares per 100 nominal at any time over the next five
years; or
a zero dividend preference share at 100 that will be redeemed in five years time
at 138

(i)

Evaluate the returns that might be achieved from each of these investments
stating any assumptions.
[5]

(ii)

Explain which investors may prefer each investment giving reasons.

[7]
[Total 12]

An investment trust provided the following information to shareholders:


Balance Sheet

Investments
Cash
Total

130
10
140

Income Statement

Dividends
Interest Income

5.0
0.5

Total

5.5

m
Shareholder Funds
Debenture 2016 (nominal value)
Total

110
30
140
m

Interest Expense
Management Expenses
Other Expenses
Dividends Paid
Total

2.4
0.9
0.3
1.9
5.5

The current share price is 170 pence per share and there are 55 million shares in issue.
(i)

Calculate appropriate accounting ratios for the above trust and comment
briefly on them.

[7]

In the recent past many trusts have been paying off their long-term debt.
(ii)

ST5 S2006

Calculate the effect that paying off this trust s debt might have on the results
in (i). You may assume that there are no restrictions to paying off the
investment trust s debt.
[7]
[Total 14]

PLEASE TURN OVER

You are the broker to a UK charitable foundation with assets of 1bn invested in a
diversified portfolio of assets. The foundation is considering making large changes to
its asset allocation and the trustees are particularly concerned about the possibility of
moving market prices (both on sales and purchases of lines of stock).
(i)

List four approaches that might help mitigate this problem, explaining how
these will help.
[4]

(ii)

Describe the assets for which the problem might be particularly acute, and for
which the solutions listed above are unlikely to be effective.
[2]

At a meeting with the trustees, they explained that they would like to implement the
switch in stages, and that their rules currently prohibit them from using derivatives.
For the first stage of the switch they will be investing 100m in Japanese equities and
financing this by selling 50m of UK and 50m of US equities. They wish the switch
to proceed as soon as possible.

(iii)

Describe the practical problems of carrying out such a switch by transferring


physical stock, in addition to the risk of moving market prices.
[10]

(iv)

Explain how the switching process would operate if derivatives were used, and
list the advantages of doing this rather than a physical asset switch.
[5]
[Total 21]

You are an investment manager specialising in equities. You want to set up a fund
that will track the FTSE All-Share index. The fund will match the index weight in
each industry sector, but will not necessarily include every index stock to achieve the
sector weight.
(i)

List the eight economic groups in the FTSE Actuaries Classification and
briefly explain their key characteristics.
[10]

(ii)

Explain in detail why the fund manager will not include every index stock to
achieve the sector weight.
[7]

(iii)

Outline the quantitative investigations that could be made to ensure that the
fund effectively tracks the index after the fund has been set up.
[5]
[Total 22]

END OF PAPER

ST5 S2006

Faculty of Actuaries

Institute of Actuaries

EXAMINATION
September 2006

Subject ST5 Finance and Investment


Specialist Technical A
EXAMINERS REPORT
Introduction
The attached subject report has been written by the Principal Examiner with the aim of
helping candidates. The questions and comments are based around Core Reading as the
interpretation of the syllabus to which the examiners are working. They have however given
credit for any alternative approach or interpretation which they consider to be reasonable.
M A Stocker
Chairman of the Board of Examiners
November 2006

Faculty of Actuaries
Institute of Actuaries

Subject ST5 (Finance and Investment Specialist Technical A) September 2006 Examiners Report

General comments
Overall candidates scored well on the bookwork sections of the paper. However, as in
previous sittings, many fared less well where answers required application (perhaps
reflecting a lack of experience of practical investment problems), especially when numerical
answers were required; as a result questions 4 and 5 proved difficult generally with even
better candidates seeming unable to work their way through demonstrating that they
understood the investments and their key features. This explains why the pass rate was higher
than it has been in some previous sittings but more importantly why there were so many
FA fails.
It is concerning that many candidates appear to have little understanding of the outside
world. An example of this is that hardly any realised that the debenture in Q5 would have to
be repaid at todays price and not par if it was redeemed.
Better candidates were able to apply the bookwork to the situation outlined in the question.
The solutions should not be taken as comprehensive. There were a number of additional
points that could have been made for various questions and these have been rewarded
appropriately.
Comments on individual questions
Q1

In general this question was answered poorly. Many candidates failed to discuss how
asset liability models could be used to oversee national debt management policy,
instead describing the steps involved when carrying out an asset liability study.
Consequently many candidates only picked up the bookwork marks.

Q2

A bookwork question which was answered well by most candidates.

Q3

Overall candidates scored well on this question. In particular Part (ii) was well
answered. Better candidates scored highly on Part (iii).

Q4

This question was answered very poorly with many candidates scoring low marks. It
was clear that candidates struggled to understand the various investment options and
therefore, were unable to calculate the expected returns required in Part (i). The
answers to Part ii were slightly better.

Q5

Another numerical question that was answered poorly. Many candidates failed to
apply the ratios relevant for an Investment Trust. For Part (ii) candidates assumed
that debt could immediately be paid off at face value rather than using a discounted
cashflow approach.

Q6

Candidates answered the bookwork sections of this question well, in particular


Part (i). The range of answers in Part iii distinguished the better candidates as they
were able to articulate the practical issues when implementing an investment switch.
Part iv was generally well answered with candidates showing a good level of
knowledge on derivatives.

Q7

Most candidates scored full marks on Part (i). In general Parts (ii) and (iii) were also
well answered.

Page 2

Subject ST5 (Finance and Investment Specialist Technical A) September 2006 Examiners Report

ALM is an investigation into longer-term issues, involving the projection of assets


and liabilities over periods of several years.
The inputs are the future profile of asset proceeds and liability outgo.
Assets are largely future tax receipts but include future debt issuance and liabilities
are future government spending including redemptions and interest on existing debt.
Future proceeds and outgo will be heavily influenced by the economic and business
cycles together with many politically driven decisions and other government policies.
(Or alternative description of input assumptions.)
The ALM can provide a series of annual differences between income and outgo,
under various simulations of the factors noted above.
Debt issuance (repurchase) is the mechanism by which the government makes up any
shortfall (excess) in the public finances i.e. the pattern of expected amounts of future
issuance is the output of the ALM.
The debt manager will have little or no control over most of the asset or liability items
but can use the ALM study to provide it with information as to the likely pattern of
future issuance requirements.
It will be able to influence the profile of redemptions and therefore have some impact
on the future profile of issuance.

(i)

The Beta of a portfolio is a measure of the portfolios volatility relative to


movements in the whole market.
It is usually defined as the covariance of the return on the portfolio with the
return on the market, divided by the variance of the market return.
A beta of 1.2 means the change in value of the portfolio should be 20% greater
than the change in value of the market.

(ii)

The performance of the portfolio would be compared to the return on the


index.
The portfolios target return should recognise the pre specified level of risk.
Using an index representative of the market the portfolio is invested in, target
returns could be calculated as 1.2x the index return.
(Alternative descriptions using a risk adjusted measure received equivalent
credits.)

Page 3

Subject ST5 (Finance and Investment Specialist Technical A) September 2006 Examiners Report

Quarterly returns for the portfolio could be compared to the quarterly returns
on the target over, say, a five year period.
The excess return would indicate the level of value added by the manager.
Other relevant points.
(iii)

The performance will differ because the portfolio will be unlikely to hold
stocks and sectors in weights that are wholly representative of the index/may
have taken an active position.
The portfolios beta over the period may have varied to levels above or below
1.2 affecting returns.
The portfolio may have other objectives/constraints which affect performance.
The beta is different.
The diversification (or lack of it ) may affect volatility of portfolio returns.
The volume and dealing cost impact of trades in the portfolio.
The effects of cash flow.
The impact of tax.
The effects of expenses.

(iv)

The beta of individual stocks can be assessed by calculating the covariance of


the return for an individual investment with the return on the index.
If the beta of an individual stock was below 1.2 then the stock could be used
as a diversifier for the portfolio as its inclusion would reduce the overall risk
of the portfolio.

(i)

Popularity how widely followed is the index by various classes of investors


(e.g. asset managers, investment banks, hedge funds etc.).
Benchmarking is the index used as a benchmark for European equity
mandates?
Base currency what currency should the futures contract use as a base
currency?
Currency hedging should investment returns in the various local currencies
be assumed to be hedged against the base currency?
Frequency how often is the index published?

Page 4

Subject ST5 (Finance and Investment Specialist Technical A) September 2006 Examiners Report

Total return/withholding taxes would need to be a total return index


(preferably net of withholding taxes).
Method of index construction e.g. based on largest x European stocks by
market capitalisation, or based on a weighted average of underlying country
indices?
Frequency of updates to constituents (or any weightings)
Capitalisation weighted, or based on some other weighting process (equal
weights, capped weights, economic value, etc.)
Whether part of a series of indices (by country, style, sector etc.), so giving
scope for exchange to expand its range of futures contracts in the future if the
demand exists.
Suitable indices FTSE All-World Europe, MSCI Europe, Eurotop, Dow
Jones Eurostoxx.
(ii)

Some funds may have restrictions on the use of derivatives. This would mean
that a particular fund could not invest in the futures contract.
Some stocks may be excluded for ethical or social reasons (or possibly other
reasons in some cases). For example a fund may choose not to invest in
alcohol or tobacco stocks.
A subset of the European equity universe may be excluded in some mandates
as it is reflected elsewhere in an investors portfolio, or the investor has
decided for strategic reasons to exclude this subset. For example a UK
investor may have separate UK equity and European ex-UK mandates.
A fund may have restrictions on self-investment and for very large companies
pension funds this may create operational issues with using the contract.
Examples BP, Novartis.
Funds may have maximum holdings in individual securities or countries
within Europe. For example a 5% maximum investment in any single stock is
a common restriction.

Page 5

Subject ST5 (Finance and Investment Specialist Technical A) September 2006 Examiners Report

(iii)

Split by country.
Split by sector.
Split into large, mid and small cap stocks.
Split by high/medium/low dividend yield.
Split into growth and income stocks.
Variations change from stock returns on an unhedged basis to hedged into a
range of different currencies (or vice versa).
Changing the base currency.
Changing the weighting approach.

(i)

The convertible has a minimum return of 100 per 100 invested assuming
company remains solvent. ZDP has maximum of 138, convertible depends on
share price.
Convertible is ZDP plus call option.
Option price = 100 - 100/138 = 27.5.
Convertible return better than ZDP if share price above 92 pence.
ZDP compounds annually at 6.65% p.a.
Share has dividend yield of 3.5% p.a. assuming no change in rate.
Capital growth of 3.15% required to match ZDP over 5 years.

(ii)

The equity could be attractive to any investor. It has a reasonable yield.


(Or equivalent comment from other investment backgrounds.)
The convertible requires a very strong price rise to be attractive to equity
orientated investors.
Convertible needs 10.0% to achieve same goal.
It might be of interest to a hedge fund wishing to carry out arbitrage
investment.
It could be of interest to an absolute return manager if market volatility was
expected to increase.
The ZDP is of interest to risk averse investors with minimum return criteria.

Page 6

Subject ST5 (Finance and Investment Specialist Technical A) September 2006 Examiners Report

It might be of interest to structured product providers as collateral.


(Other examples of possible investors received equivalent credit.)
It could be of interest to hedge funds in conjunction with the ordinary and
convertible to create an absolute/arbitrage return.

(a)

Yield on investments = 3.85%


Yield on year end cash = 5%
Net asset value per share = 200p, discount = 15%
Interest on debt = 8%
Total expense ratio = 0.86%
Dividend per share = 3.45p, yield = 2.0%
Portfolio has high yield ratio to market
Cash return high need to investigate
Gearing = 140/110 = 27%
Gearing slightly high for conventional trust
(Other relevant ratios plus comment received equivalent credit.)

(b)

Debt coupon = 8%, current 10 year interest rate = 4.5% (say)


Cost of paying off debt = 127.69 * 30m/100 = 38.31m
Assets = 101.69m, nav per share = 184.89
Income, assuming cash and prorate reduction = 3.92m
Management expense ~ 0.65% = 0.66m, other = 0.3m
Available for div = 2.96m = 5.38 pence = 3.2% yield
Might suggest reducing yield ratio

(i)

Selling/purchasing assets in small units


Selling/purchasing assets over a period of time (including use of cashflow)
Using algorithmic trading techniques
Using nominee accounts
Using several banks to carry out the trades
Using derivative contracts
Use cross transactions
All of the above methods will mask the trade to other market participants
and can be used in combination.
Derivatives contracts will be suitable in many cases for changing broad
exposures quickly although the underlying stocks will still need to be
sold/purchased in due course unless the asset allocation change is temporary.

Page 7

Subject ST5 (Finance and Investment Specialist Technical A) September 2006 Examiners Report

(ii)

These approaches are difficult to implement for unmarketable securities where


there is no deep or liquid market.
Assets in this category might include small cap or unquoted equities, certain
high yield bonds or smaller bond issues, and property.

(iii)

Significant coordination is needed by the UK, US and Japanese equity


managers to ensure that the switch goes smoothly.
It may be useful to appoint a transition manager to coordinate the process if
none of the existing asset managers has a specialist team who is able to
manage the project.
There may be several asset managers involved in the process.
The managers may need some time to decide which stocks to buy and sell, and
this may create further delays as the transition occurs.
In practice, it is likely that the sales of UK and US equities will need to be
spread over a period of time, particularly for smaller stocks.
Unless borrowing is permitted (unlikely) the Japanese equity manager can
only buy stock after the UK and US equity managers have sold stock
allowing for settlement periods
This will delay the transition.
This may have an adverse impact due to market movements
or currency movements
However, rushing the sales is likely to impact on the value realised from the
sale.
Tax liabilities may be brought forward and crystallised, particularly on stocks
with large gains since purchase.
Conversely tax losses may be created where stocks are being sold at a loss,
and it is not known when those losses might be offset against future gains in
the UK or USA.
The process will be time intensive and may be expensive in terms of
transaction costs.
It is likely that the fund will suffer both buying and selling fees
although these might be mitigated if the asset managers are able to cross
with transactions for their other clients.

Page 8

Subject ST5 (Finance and Investment Specialist Technical A) September 2006 Examiners Report

The transition programme will need ongoing monitoring in case market


movements mean that the target changes in asset allocation are not achieved,
or exceeded.
(iv)

The UK and US equity managers would sell a number of futures contracts


with exposure equal to the amount of stock to be sold.
The Japanese equity manager would buy a number of futures contracts with
exposure equal to the amount of stock to be purchased.
As the physical sales and purchases are completed the number of futures
outstanding will need to be reduced to ensure that the correct exposure is
maintained.
If the physical sales and purchases are not completed before expiry of the
futures contracts, then any outstanding futures positions will need to be rolled
over.
An operational issue with futures is that initial and variation margins will need
to be deposited as the counterparty will want collateral for any outstanding
positions.
If the various managers do not have sufficient assets to deposit as collateral
then borrowing may be necessary.
Advantages:
Switch can be implemented very quickly.
Futures markets is very liquid, with little risk of any market impact
(particularly as the futures contracts will be based on market indices).
Purchase of futures reduces pressure on asset managers to construct and
execute their buy/sell programs quickly.
If the switch needs to be adjusted the positions can be easily and cheaply
reversed without disturbing the underlying stock portfolios.

Page 9

Subject ST5 (Finance and Investment Specialist Technical A) September 2006 Examiners Report

(i)

Resources
These companies are involved in the extraction and supply of primary
products used throughout the economy. Oil is the most important. Key
characteristics are:

large companies
commodity price dependent
risky
global

Basic industries
This group includes the chemical industry and companies in the building
materials and construction industries, as well as companies producing steel
and other metals, and those engaged in forestry and paper. As such, these
companies are mainly producing intermediate goods.
General industries
General industrial companies are involved in the various stages in the supply
and production of goods. Many of the goods tend to be capital items, i.e.
aircraft, ships, machinery, electronic and electrical equipment. The distinctive
features of both industry groups are:

dependent on the level of investment spending


cyclical
company profits tend to move ahead of the trade cycle
dependent on government spending
volatile profits
high profit margins when conditions are good
low gearing because of volatile profits
possibly exposed to overseas markets and competition

Consumer goods
Companies in the consumer goods groups manufacture consumer durables and
non-durables. Durables include cars, furniture, televisions and white goods
such as washing machines. Non-durables include food and drink,
pharmaceuticals, tobacco, health and household products, beverages and
packaging. Generally the impact of an economic cycle is less severe on nondurable consumer goods companies than on general manufacturers. This is
especially true for companies producing basic necessities. Thus, the consumer
goods group is further divided into cyclical (durable) and non-cyclical (nondurable) sectors. Other key features are:

Page 10

increasingly capital intensive


importance of brand names
increasingly international

Subject ST5 (Finance and Investment Specialist Technical A) September 2006 Examiners Report

moderate to high gearing


low profit margins

Services
These are also now divided into cyclical and non-cyclical sectors. Cyclical
service companies include general retailers, transport, hotel and media
companies, distributors, restaurants and pubs and support services. Noncyclicals include food and drug retailers and telecommunication services.
Once again, the impact of the economic cycle will be greater on the cyclical
group. Other key features are:

labour intensive
the more defensive companies in the group may have high gearing
the domestic market is the most important

Utilities
Utilities are involved in the supply of continuously demanded services to
households and business premises. Examples include electricity, water and gas
distribution. Most UK utilities were formerly owned by the government,
having been privatised during the 1980s. They are vulnerable to some political
risk and to changes in the regulations under which they operate. Demand is
very stable because the services that they provide are essential, or nearly
essential, and because their market share will be stable (often at 100%). Thus,
they are less affected by economic cycles than other groups. Other points are:

they usually require an extensive physical infrastructure. This tends to


make them capital intensive
most utility companies are natural monopolies
they are usually subject to tight government regulation of prices and
vulnerable to other forms of political risk
they generally have low growth prospects; this leads to a high gross
dividend yield
despite their stable demand and large capital requirements, gearing is low
they are largely dependent on the domestic market, although some
companies are diversifying internationally

Financials
The financial group companies are the various industries making up the
financial services industry, e.g. clearing banks, investment banks, general
insurance companies and life assurance companies, investment trusts, real
estate (property) companies. The key distinctive feature of financial group
companies is that they tend to be capital intensive. Otherwise, the features of
companies in this group are quite varied between the different sectors:

banks are highly geared and have volatile profits


general insurers also have volatile profits and virtually no borrowings

Page 11

Subject ST5 (Finance and Investment Specialist Technical A) September 2006 Examiners Report

life insurers have stable profits and low gearing


labour costs are important for many companies in the group
the domestic market is most important but there is increasing
internationalisation

Information Technology
These are the companies involved in the new industries of information
technology hardware, software and the provision of computer services. While
investor demand for such shares has caused share prices to increase
dramatically in the past, many of the companies have yet to make profits or
pay dividends. Dividend yields on these companies are therefore low, and
their assets can be largely intangible.
(ii)

The fund manager only has to track the performance of the index.
So replicating the index is not essential.
Investing in many stocks and so having relatively small individual holdings in
each stock will result in high dealing costs (necessary each time the relative
sector weightings change).
This would reduce the performance of the fund and so cause
underperformance relative to the index.
Research has shown that, after overall market movements have been taken into
consideration, the share price movements of companies within industrial
groupings tends to correlate more closely with each other than with companies
in other industries, so holding a subset may well replicate the performance of
the sector.
The share price movements reflect the changes that have occurred in the
operating environment, and such changes affect companies in the same
industries in similar ways.
A specific sector may only represent a small percentage of the index and
within that sector the small number of stocks the manager proposes to use may
represent a substantial proportion of the total market capitalisation of the
sector.
Stratified sampling of the performance of each sector may have shown that the
performance of the chosen stocks is a very accurate measure of the
performance of the sector as a whole.
Sampling may enable the fund to choose its timing in addressing whether or
when to replicate changes to the underlying index.

(iii)

Page 12

Compare dividend yields, earnings growth and price earnings ratios with the
Index.

Subject ST5 (Finance and Investment Specialist Technical A) September 2006 Examiners Report

For example, within each sector for the fund and the index:

Rank the holdings by increasing yield


Split each sector into an equal number of holdings (e.g. quintiles)
Calculate the weighted average yield of each quintile allowing for the
value of shareholdings as weights

This will help to determine:

Consistency with the portfolio


Identify any style biases (e.g. growth or value)
How risky the portfolios are relative to the index

Historic comparison of the fund performance with the index quarterly over a
period of around three years to determine how well the fund has tracked the
index.
Comparison of risk adjusted performance measures e.g. Sharp or pre-specified
standard deviation.

END OF EXAMINERS REPORT

Page 13

Faculty of Actuaries

Institute of Actuaries

EXAMINATION
13 April 2007 (pm)

Subject ST5 Finance and Investment


Specialist Technical A
Time allowed: Three hours
INSTRUCTIONS TO THE CANDIDATE
1.

Enter all the candidate and examination details as requested on the front of your answer
booklet.

2.

You have 15 minutes at the start of the examination in which to read the questions.
You are strongly encouraged to use this time for reading only, but notes may be made.
You then have three hours to complete the paper.

3.

You must not start writing your answers in the booklet until instructed to do so by the
supervisor.

4.

Mark allocations are shown in brackets.

5.

Attempt all 9 questions, beginning your answer to each question on a separate sheet.

6.

Candidates should show calculations where this is appropriate.

AT THE END OF THE EXAMINATION


Hand in BOTH your answer booklet, with any additional sheets firmly attached, and this
question paper.
In addition to this paper you should have available the 2002 edition of the
Formulae and Tables and your own electronic calculator.

ST5 A2007

Faculty of Actuaries
Institute of Actuaries

(i)

(a)

Define risk-free return.

(b)

List three types of asset that would be expected to provide such a


return.
[2]

(ii)

Write down an equation for the required return on an asset.

(iii)

Explain why it is necessary to estimate the equity risk premium from historical
data.
[2]

(iv)

Give examples of:

(i)

[1]

(a)

the types of distortion that need to be allowed for in any analysis of


historical risk premia.

(b)

the likely impact of allowing for these adjustments.

[6]
[Total 11]

Describe how the economic cycle can impact companies price/earnings ratios.
[5]

A stock market comprises three securities:

Market capitalisation 31/12/2005


Earnings 31/12/200531/12/2006
Share price 31/12/2005
Dividend yield 31/12/2005
Share price 31/12/2006
Dividend yield 31/12/2006
(ii)

Stock A

Stock B

Stock C

$100m
$10m
$20
3%
$24
3.2%

$500m
$25m
$20
2.8%
$25
2.5%

$200m
$25m
$20
4%
$16
4.5%

Describe the characteristics of growth and value stocks.

[2]

A value investor constructs a portfolio at 31 December 2005 comprising 10% stock A,


30% stock B and 60% stock C. In contrast a growth investor has a portfolio of 20%
stock A, 70% stock B and 10% stock C.
(iii)

Explain why the two investors may have constructed their portfolios as they
did.
[3]

(iv)

Calculate the portfolio returns over the period for each of the two investors. [3]

(v)

Describe three other equity styles that other investors may be using.

ST5 A20072

[3]
[Total 16]

(i)

(a)
(b)

Explain what is meant by an agency cost.


Explain how agency costs can arise.
[4]

(ii)

(a)

Explain why conflicts of interest may arise between equity and bond
holders in a company.

(b)

Give three examples of such conflicts of interest including details of


who gains and who loses from the action.
[4]
[Total 8]

A professional fund manager invests in the constituent shares of the FTSE 100 index
and weights the investments of the fund on an arithmetic average basis using the
market capitalisation of the constituents of the FTSE 100 index. The fund manager
adjusts the constituent shares and their weights in line with changes in the weights
used in the construction of the index. To all intents and purposes, the fund attempts to
track the price and yield performance of the FTSE 100 index.
The investors in the fund asked an independent investment consultant to evaluate the
total return performance of the professional fund manager relative to the total return
performance of the FTSE 100 index over the last ten years. The independent
investment consultants report examined the fund managers total return and
concluded that the professional fund manager had under performed the FTSE 100
Total Return Index over the ten year period in question.

(i)

Describe the most likely reasons for the fund managers under performance
relative to the FTSE 100 index over the past 10 years.
[12]

(ii)

Outline two ways of reducing the under performance of the fund manager
relative to the FTSE 100 index.
[2]
[Total 14]

(i)

Sketch, on the same diagram, the payoff at maturity of the following options
on a quoted share assuming that they have the same strike price, same maturity
date and that the ratio of premiums is 2:1 with the call option being the more
expensive option:
(a)
(b)

a put option
a call option
[3]

(ii)

Using the same diagram as in part (i) of this question, sketch the payoff profile
of an equally weighted portfolio consisting of options (i)(a) and (i)(b).
[3]

(iii)

Suggest an investment scenario in which the payoff profile in part (ii) might
be of interest to an investor.
[2]
[Total 8]

ST5 A20073

PLEASE TURN OVER

(i)

Describe three reasons why regulation is considered important in the financial


services industry.
[3]

(ii)

Outline the direct and indirect economic costs of regulation.

(i)

Describe the concept of a trust as it arises in trust law.

(ii)

Suggest the advantages of a debenture trust deed.

[6]
[Total 9]

[5]
[3]
[Total 8]

You are the finance director for a regional Japanese bank and are considering the
issue of a bond. You have the option of issuing the bond in sterling or in local
currency and have available the following information:

Sterling issue
Domestic issue

Spread over government bonds

Government bond yield

+125bps
+175bps

4.50%
2.00%

You should assume the yield curve is flat across all maturities in both the sterling and
the domestic market.
(i)

List the main reasons for issuing a bond.

(ii)

Discuss why the spread over government bonds may differ between the UK
and Japanese markets for the same issuer.
[6]

(iii)

Calculate the return an investor would achieve, in yen terms, by investing and
holding each bond to maturity. State any assumptions made.
[6]

(iv)

Outline the steps you would take to lock into the lowest funding cost.
[2]
[Total 18]

ST5 A20074

[4]

You are a portfolio manager for a high alpha actively managed bond fund. Your
annual performance target is 1.5% p.a. above the return on the all stocks
government treasury bond index. This target is after deduction of your 0.4% annual
management fee.
One of your bond dealer contacts has invited you to subscribe to a new innovative 20
year amortising bond whose return is linked to the mortality experience on a pool of
insured annuitants. Under this bond, the coupon and principal payments are reduced
(or increased) if more (or fewer) annuitants survive than expected, within minimum
and maximum amounts.
The dealer has made a compelling pitch to you that this is the first of a number of
such bond issues and that the spreads on these bonds will narrow considerably over
time as the market gains familiarity with these issues. The Chief Investment Officer
of your investment house requires you to submit a formal analysis of the bond issue
for consideration at the next Investment Committee meeting.
(i)

Outline the issues you would consider in your analysis of the opportunity to
generate outperformance for the bond fund by participating in the offer.
[6]

(ii)

Describe what further investigations and information you would require before
you could make a recommendation on whether to participate and how much to
invest.
[2]
[Total 8]

END OF PAPER

ST5 A20075

Faculty of Actuaries

Institute of Actuaries

EXAMINATION
April 2007

Subject ST5 Finance and Investment


Specialist Technical A
EXAMINERS REPORT

Introduction
The attached subject report has been written by the Principal Examiner with the aim of
helping candidates. The questions and comments are based around Core Reading as the
interpretation of the syllabus to which the examiners are working. They have however given
credit for any alternative approach or interpretation which they consider to be reasonable.

M A Stocker
Chairman of the Board of Examiners
June 2007

Faculty of Actuaries
Institute of Actuaries

Subject ST5 (Finance and Investment Specialist Technical A) April 2007 Examiners Report

Comments
Most candidates scored well on questions 5 and 6 with many achieving full marks. Although
some candidates scored well on questions 2 and 3 also, most candidates attained closer to
half the available marks. Questions 4 and 7 were the worst answered with candidates
achieving typically scores of less than a third of the available marks. For example, few
candidates were able to identify multiple likely reasons for underperformance and most were
unable to describe a debenture trust deed and the advantages thereof. Although it was
pleasing to see the scores achieved by better candidates, it continues to be a source of
frustration and disappointment that candidates appear to ignore valuable information
contained within the question and lose easily achievable marks as a consequence.
In every diet there will be candidates who are very close to the pass mark and yet receive an
FA indeed I suspect candidates would be very surprised to see just how tightly distributed
the marks are; deciding where the pass mark falls will have a material impact on the
numbers of candidates who are successful and the examiners take great care to ensure a
consistency of standard across candidates, subjects and diets. The pass rate for this diet was
very similar to the last session although the pass mark was higher, reflecting the overall
higher scores achieved by candidates on bookwork parts of questions.
All extenuating and mitigating circumstances were considered in awarding grades
coincidentally those candidates who had submitted the most severe mitigating arguments had
in fact achieved sufficiently high marks to justify a Pass grade.
Notwithstanding the high scoring on bookwork elements, candidates should note the bias in
the paper towards recognising higher level skills and practical application this is
intentional and will continue. Likewise the examination system does properly allow for prior
subject knowledge to be assumed. It is not appropriate to repeat all relevant material within
the Core Reading and in the exam creation process, the profession takes great care to ensure
that the paper can be answered by a candidate who has taken a normal route through the
exams - indeed questions have been removed from previous draft papers as a result.
Candidates looking to progress should be aware that the SA series of exams, particularly
investment related, are even less bookwork focussed and require the candidate to
demonstrate a breadth and depth of competency as would be expected from a practising
actuary in a constantly changing discipline. Hence simple regurgitation of bookwork will not
be sufficient to ensure a Pass grade. Candidates should ensure they familiarise themselves
with the current investment issues facing institutional investors in the 18 months preceding a
diet and the solutions being debated by the various stakeholders.

Page 2

Subject ST5 (Finance and Investment Specialist Technical A) April 2007 Examiners Report

(i)

The risk-free rate of return is the rate of return on a security which has no
credit risk. Assets providing such a return include fixed interest government
bonds, inflation-linked government bonds and short-term government bills.

(ii)

Required return = Required risk free real rate of return + expected inflation +
risk premium

(iii)

For an asset with certain cashflows (e.g. a bond) the risk premium can be
estimated from the price since an IRR calculation can be carried out, although
an adjustment needs to be made for default risk.
Conversely, the risk premium from the price of an asset with uncertain future
cashflows (e.g. an equity) cannot be estimated using an IRR calculation.
This leads to a need to analyse historical data, although this may not
necessarily be a good guide as the risk premium is based on expected return,
rather than achieved return.

(iv)

Distortions that need to be corrected for in an analysis of historical risk premia


would include:
Survivorship bias allowance needs to be made for the returns achieved on
securities which have been removed from a sample due to defaults or removal
from a reference index. Allowing for this factor will typically reduce the
measured historical return.
Market valuations/risk preferences allowance should be made for changed
valuations of securities through changes in investor risk preferences. Investors
may be willing to accept a lower/higher risk premium for a given level of
volatility, leading to higher/lower prices currently than in the past.
Achieved dividend growth vs expectations part of the risk premium will
reflect expectations in future dividend growth. If dividends have grown at a
faster or slower rate than originally expected by equity investors, this could
lead to an inaccurate estimate of the risk premium based on historical return
data.

Page 3

Subject ST5 (Finance and Investment Specialist Technical A) April 2007 Examiners Report

(i)

If the economy is moderately buoyant and profits are fairly stable, both
defensive and cyclical companies might be similarly rated in terms of the P/E
ratios.
As the economy starts to move into recession P/E ratios for cyclical companies
are likely to fall while those of defensive companies will remain stable or may
even rise slightly.
At the bottom of the cycle P/E ratios of cyclical companies will probably have
risen from their low point as earnings have fallen, but defensive stocks will
still be more highly rated.
As the economy starts to recover, the P/E ratios of cyclical companies will rise
in anticipation of future earnings growth. P/E ratios of defensive companies
may now be lower than those of cyclical stocks.
As growth continues, the earnings of cyclical companies will catch up with the
share price and P/E ratios will fall back towards their long-term average level.

(ii)

Value stocks typically have low P/E ratios (12 or lower) and higher dividend
yields (4% or more). They tend to be stocks with low expectations of future
earnings growth or are out of favour with investors (reflected in the P/E ratio).
Conversely, growth stocks typically have high P/E ratios (20 or higher) and
lower dividend yields (2.5% or less). These tend to be stocks with higher
expectations of future earnings growth (reflected in the P/E ratio).

(iii)

The neutral weights for the three stocks are 12.5% stock A, 62.5% stock B and
25% stock C. The PE ratios are 10, 20 and 8 respectively, with stock A
yielding close to the average, B having a slightly lower than average yield and
C a high yield. These two measures suggest that stock A has a small value
bias, B has a growth bias and C has a value bias, hence the weightings in the
two investors portfolios.

(iv)

A: 24/20 (1 + 3.2%/2 + 3%/2) - 1 = 23.7%


B: 28.3%
C: -16.6%
Value return = 10% 23.7% + 30% 28.3% + 60% -16.6% = 0.9%
Growth return = 22.9%

(v)

Page 4

Small-cap, large cap, momentum, rotational, contrarian, top-down, bottom-up


(brief description of any three or any other relevant style properly defined)

Subject ST5 (Finance and Investment Specialist Technical A) April 2007 Examiners Report

(i)

Agency costs can arise in an organisation where the owners have delegated
operational decisions.
They become increasingly likely as an organisation grows, although they can
also occur in a smaller organisation where the owners and managers are
separate.
This separation of ownership and management can lead to a divergence of
interests, and such conflicts of interest are called principal-agency conflicts.
Agency costs are defined as the costs of monitoring the agents (managers) and
influencing/incentivising them to act in the interests of the principals
(owners)
and thereby reduce conflicts/create alignment.
Without such monitoring or influencing, management may act in a way that
diverges from the interests of the owners, and this is arguably a consequence
of rational behaviour by the management seeking to exploit their position.

(ii)

Conflicts of interest arise between equity and bond holders since bond holders
have no upside potential beyond return of their capital and interest payments
but are exposed to downside risks.
Conversely equity holders have significant upside potential and typically
exercise day to day control over the company, leading to conflicts.
Possible conflicts can include:

Equity holders underinvesting in a company to increase profits, whilst


reducing long term security for bond holders.

Equity holders influencing management to take excessive risks, increasing


the default risk of the firm (at the expense of bond holders, but not
necessarily for equity holders on an expectation basis).

Equity holders taking excessive dividends from the company, reducing


asset cover of liabilities (including payments due to bond holders).

Page 5

Subject ST5 (Finance and Investment Specialist Technical A) April 2007 Examiners Report

(i)

Fees & Expenses


The investors in the fund must pay investment management fees, custody fees,
audit fees, governance fees and administration fees whereas such fees are not
taken into account in the calculation of returns on FTSE 100 Total Return
Index.
Cost of Rebalancing
The FTSE 100 Total Return Index does not take into account the costs of
rebalancing the index for such activities as new entrants, exits, mergers and
takeovers and changes in the market capitalisation of constituents.
Such costs include stockbrokers commissions, stamp duty and other levies.
Cash Holdings
When the fund manager receives small amounts of dividend income, it may
not be cost effective for her to invest such small amounts across the entire 100
constituents in the correct proportions. The manager will therefore have part
of the portfolio invested in the constituents of the FTSE 100 index and part
invested in cash. The cash holding will cause the manager to under perform
the index in a rising market and out perform the index in a falling market.
This can be a significant issue when there are large inflows or outflows from
the fund, particularly if asset volatility is higher than typical.

(ii)

Futures
The manager may be able to reduce the drag on investment performance
arising from cash holdings by using FTSE 100 futures contracts.
Stock Lending
The manager may be able to reduce the size of her under performance by
engaging in stock lending whereby she will receive a fee for lending stocks to
other institutions (such as the prime brokers of hedge funds) on a collateralised
basis.

Page 6

Subject ST5 (Finance and Investment Specialist Technical A) April 2007 Examiners Report

(i)

Marks should be awarded for the following features:


Same axes and same scales one rather than two diagrams;
Common strike price
2:1 ratio of premiums (suggested on inspection by eye)
General shape of each options hockey stick diagram.

(ii)

Marks should be awarded for the following features:


Same axes and same scales one rather than two diagrams;
Common strike price of the combined option portfolio;
Maximum loss of 3 units (implied by 2:1 ratio of premiums for basic options
in (i)) (suggested on inspection by eye);
V shape of diagram.

(iii)

The portfolio of options pays off at maturity when there is a large move either
way in the price of the underlying stock over the life of the option.
Conversely investors in the portfolio of options will lose out if there is little or
no movement in the price of the underlying stock by the maturity date of the
option.

(i)

Confidence: It is important that there is confidence in the financial system.


Systematic Failure: There is a danger of problems in one area spreading to
other parts of the financial system.
There is believed to be a significant asymmetry of information, negotiating
strength and expertise in the financial markets especially among retail
investors.

(ii)

The direct costs of regulation arise in administering the regulation and in


compliance for the regulated firms.
Some possible indirect costs include:

Consumers behaviour is altered which gives rise to a false sense of


security and a reduced sense of responsibility.

A reduction in the consumer protection measures developed by the market


itself.

Product innovation is reduced.

Competition is reduced.

The sense of professional responsibility amongst intermediaries and


advisors is undermined.

Page 7

Subject ST5 (Finance and Investment Specialist Technical A) April 2007 Examiners Report

(i)

Trusts constitute a relationship between persons in which one person has the
power to manage property and the other person has the privilege of receiving
the benefits from that same property.
The legal owner of the property of a trust is called the trustee and she has the
right to possession of the property.
The beneficiary of the trust is the person who receives all the benefit from the
property.
The divisions between legal and beneficial ownership are normally created by
an express instrument of trust known, usually, as the deed of trust.
The trust deed will also specify the beneficiaries by name or as being persons
who are members of a particular group.
Trustees are required to act in the best interests of the beneficiaries of the trust.
The standard of care required of trustees varies with their level of expertise in
that a higher standard of care is required of those who hold themselves out to
be professional trustees compared with an ordinary man.

(ii)

In a debenture trust deed, the individual interests of the usually substantial


number of holders of a debenture are channelled into the trust.
The trustee holds and protects the aggregate of the personal rights of all
members of the trust.
A debenture trust may be thought of as providing efficient administration.

(i)

Raising new or refinancing capital many possible purposes, acquisition,


investment in the business, fund new projects, financing of pension fund
Restructuring the balance sheet share buy backs, securitising future
cashflows.

(ii)

Spread over government yields reflects the extra premium that the bank must
pay to compensate the investors for extra risks of reduced liquidity and
default. These risks should be similar regardless of the currency of the bond
issue. It is likely that the bank earns its cashflows in local currency so has no
exposure to sterling so one may argue that the sterling denominated issue
has additional currency risks and should have a higher spread to governments
to compensate. The spread is also affected by supply and demand dynamics
the lower spread in reflecting a higher level of demand.

(iii)

Assuming the bond doesnt default


Yen 3.75% p.a.

Page 8

3.25% p.a. (4.5% + 1.25% - 4.5% + 2%)

Subject ST5 (Finance and Investment Specialist Technical A) April 2007 Examiners Report

The government bond used for comparison has a similar maturity/duration to


the Japanese bond being considered.
Currency movements reflect the interest rate differentials between the two
countries.

(iv)

Issue bond in sterling


Implement forward currency transactions to neutralise currency mismatch
risks

(i)

The dealer is probably correct that initially spreads over gilts will be high due
to investors unfamiliarity with such an issue.
Also investors typically (but not always) prefer vanilla bonds without
embedded exposures or options, leading to higher spreads on more complex
issues due to difficulty in assessing if pricing is favourable or not.
However, it is also possible that there is considerable demand in the investor
community for such issues, in which case initial spreads may be lower.
If the market does become more confident about these bonds then subscribing
would offer the prospect of say a 50bps reduction in spreads, which would
amount to 6% increase in value if the duration was 12 years (for example).
Alternatively, if the market did not develop strongly, this could result in the
fund being stuck with a long-term illiquid issue albeit providing a modest
spread over vanilla bonds of a similar term and credit. If the bond had a
shorter term e.g. 5 years, this would be less of a concern.
Such a negative scenario could also occur if the market for mortality-linked
bonds does develop, but with different design features.
Assess impact on duration of the new bond from sensitivity analysis of
expected annuitants survivors. Also important to assess the resultant duration
of the fund from the desired allocation to the new bond, and relative to the
duration of the reference index.

(ii)

Further investigations:

Some sensitivity testing of ultimate proceeds and how mortality


experience might impact on the ultimate return, most likely using
stochastic analysis.

Need to understand how sensitive the return on the bond is to mortality


experience, as opposed to economic factors. In the longer term it may
become important to monitor the sensitivity of the fund to mortality
factors, from a risk control point of view.

Page 9

Subject ST5 (Finance and Investment Specialist Technical A) April 2007 Examiners Report

Investigate size of potential market, from both a supply and a demand


point of view.

Feedback from internal analysts and external market participants, e.g. asset
managers, bond dealers, analysts.

The proposed offer price of the bond is a key bit of information that would
be required.

Specific information on the pool of insured annuitants should be


investigated.

END OF EXAMINERS REPORT

Page 10

Faculty of Actuaries

Institute of Actuaries

EXAMINATION
26 September 2007 (pm)

Subject ST5 Finance and Investment


Specialist Technical A
Time allowed: Three hours
INSTRUCTIONS TO THE CANDIDATE
1.

Enter all the candidate and examination details as requested on the front of your answer
booklet.

2.

You have 15 minutes at the start of the examination in which to read the questions.
You are strongly encouraged to use this time for reading only, but notes may be made.
You then have three hours to complete the paper.

3.

You must not start writing your answers in the booklet until instructed to do so by the
supervisor.

4.

Mark allocations are shown in brackets.

5.

Attempt all 8 questions, beginning your answer to each question on a separate sheet.

6.

Candidates should show calculations where this is appropriate.

AT THE END OF THE EXAMINATION


Hand in BOTH your answer booklet, with any additional sheets firmly attached, and this
question paper.
In addition to this paper you should have available the 2002 edition of the
Formulae and Tables and your own electronic calculator.

ST5 S2007

Faculty of Actuaries
Institute of Actuaries

(i)

State the three main aims of regulation.

(ii)

With any type of regulation there is a cost.

[1]

(a)

Outline the different types of costs involved.

(b)

How do these costs impact the overall objectives of the regulator. [5]
[Total 6]

You are the financial director of a small private software company. Due to a contract
being recently cancelled there is likely to be a short-term liquidity issue where the
company cannot meet its creditors for the next three months, after which the financial
picture looks healthy. The Chairman of the company is opposed to long-term debt.
You have been asked to provide the Chairman with alternative sources of short-term
finance to resolve the impending liquidity issue.

(i)

Describe the financing options available to the company.

[5]

(ii)

Outline the main contract terms that differentiate between the types of
borrowing available.
[4]
[Total 9]

Three companies are intending to raise finance via the debt market.

Company A A government backed company which has a history of strong


profits and low levels of debt.

Company B A public listed company that is well established but has suffered
from decreasing sales and has posted losses for the last two years.

Company C An internet start up company that intends to sell wedding cakes


on-line.

(i)

Explain with reasons the credit rating that is likely to be assigned to each of
the three bond issues.
[2]

(ii)

(a)

Define the term expected default loss.

(b)

Explain how this would vary for each of the three companies.
[3]

All three companies decide to issue zero coupon bonds.


(iii)

ST5 S20072

(a)

Assuming the yield on a comparable treasury stock is 4.25% p.a., state


what yields would be appropriate on a zero coupon bond issued by
each company.

(b)

Calculate the expected default loss for the first year of each bond,
stating any assumptions that you make.

[8]

The Government of the country is concerned that the local currency is appreciating
too fast and decides to introduce immediate controls on transactions by overseas
investors. As a consequence the domestic equity market index has fallen by 15% and
treasury bonds are now yielding 5.25% p.a.
(iv)

Explain how the yields on the bonds issued by the three companies might
change as a consequence of the change in market levels.
[6]
[Total 19]

A friend has decided that they would like to try to increase their wealth by investing
in derivatives. After reading some articles they realise that they do not fully
understand some of the terminology and have approached you for help.
(i)

Explain the main uses of derivatives.

[8]

(ii)

For a derivative contract traded on an exchange:


(a)

Explain the term margin.

(b)

Outline the different types of margin payments that are payable.

(c)

Explain why a clearing house requires these payments.


[3]

(iii)

(a)

Explain the difference between a Put and a Call.

(b)

Explain the term European and American in the context of options.


[2]

The current share price of XYZ is 60p. Your friend has been offered the following
options in XYZ.

(iv)

ST5 S20073

Strike Price

3 Month

6 Month

Call 75p
Put 85p

5p
10p

10p
5p

Given this information, draw the pay-off charts associated with each of these
options clearly identifying the price when the option is in the money.
[6]
[Total 19]

PLEASE TURN OVER

(i)

Describe the main forms of government policy.

[2]

(ii)

Outline the main economic indicators which show whether the policies applied
by the government have been successful.
[2]

To promote growth after a prolonged recession the government has decided to reduce
interest rates from 5% to 3%.
(iii)

Describe how this move is likely to impact:


(a)
(b)
(c)

individuals
businesses
the economy as a whole
[6]
[Total 10]

You are the investment consultant to a 400m pension fund that has a 15% shortfall in
assets compared with the national common funding standard introduced by the newly
created regulator of pension funds. In addition, the new regulator has insisted that all
defined benefit funds have a national common funding level above 105% in seven
years time. One of the trustees has read a newspaper article claiming that more
pension funds are investing in hedge funds as a way of meeting their liabilities and the
requirements of the new regulator.
(i)

(a)

(b)

Outline the main types of hedge fund that the pension fund could
invest in.

[2]

Describe the main investment characteristics of a hedge fund.

[3]

One of the criticisms of hedge funds is the lack of reliable performance data.

(ii)

Explain why there is a lack of credible performance data.

[4]

(iii)

Explain how the fund could invest in hedge funds alongside other assets and
derivatives in order to achieve the national common funding target objectives.
[6]
[Total 15]

An institutional fund management company listed on the Alternative Investment


Market has sought your advice on how it can better manage the operational risks in its
business.
(i)

Outline the risks that you would seek to address.

(ii)

Describe ways in which these risks can be managed.

ST5 S20074

[3]
[9]
[Total 12]

You have recently been appointed as the financial adviser to a private company. The
company wants to provide an initial offering of shares and to be listed on the stock
exchange. You have been asked to provide a valuation of the company.
(i)

Describe the role of a listings authority.

[5]

(ii)

State the information you would wish to see in order to provide a valuation.
[5]
[Total 10]

END OF PAPER

ST5 S20075

Faculty of Actuaries

Institute of Actuaries

EXAMINATION
September 2007

Subject ST5 Finance and Investment


Specialist Technical A
EXAMINERS REPORT
Introduction
The attached subject report has been written by the Principal Examiner with the aim of
helping candidates. The questions and comments are based around Core Reading as the
interpretation of the syllabus to which the examiners are working. They have however given
credit for any alternative approach or interpretation which they consider to be reasonable.

M A Stocker
Chairman of the Board of Examiners
December 2007

Faculty of Actuaries
Institute of Actuaries

Subject ST5 (Finance and Investment Specialist Technical A) September 2007 Examiners Report

Comments
Most candidates scored well on questions 1, 3 and 4 with many achieving full marks.
Although some candidates scored well on questions 2 and 5 also, many candidates attained
closer to half the available marks. Questions 6, 7 and 8 were the worst answered (7 in
particular).
In every diet there will be candidates who are very close to the pass mark and yet receive an
FA indeed I suspect candidates would be very surprised to see just how tightly distributed
the marks are; deciding where the pass mark falls will have a material impact on the
numbers of candidates who are successful and the examiners take great care to ensure a
consistency of standard across candidates, subjects and diets. That said, it was fairly clear
where the hurdle should have been set. The examiners were pleased to see that the pass rate
for this diet was slightly higher than last time even though the pass mark was somewhat
higher. Where candidates scored lower it was typically because although they were able to
reproduce the required bookwork for one or other question, they were unable to apply the
bookwork knowledge appropriately.
Candidates should note the bias in the paper towards recognising higher level skills and
practical application this is intentional and will continue. Likewise the examination system
does properly allow for prior subject knowledge to be assumed. Investment is a necessarily
practical subject and at this level, the examiners expect candidates to demonstrate a breadth
and depth of competency as would be expected from a practising actuary in what is a
frequently evolving discipline. Hence simple regurgitation of bookwork will not be sufficient
to ensure a Pass grade.
Candidates looking to progress should be aware that the SA series of exams, particularly
investment related, are even less bookwork focussed and require the candidate to
demonstrate a breadth and depth of competency as would be expected from a practising
actuary in a constantly changing discipline.
In order to succeed, candidates should ensure they familiarise themselves with the current
investment issues and general market background facing institutional investors in the 18
months preceding a diet and the solutions (and sources of) being debated by the various
stakeholders. A recurring theme in recent years has been a move towards capital market
rather than purely insurance and asset management solutions hence a question regarding
banking and derivative approaches to asset and liability risk management or modern
financial theory and commercial applications should be considered likely scope for
examination.
All extenuating and mitigating circumstances were considered in awarding grades.

Page 2

Subject ST5 (Finance and Investment Specialist Technical A) September 2007 Examiners Report

(i)

To correct market inefficiencies and to promote orderly markets


To protect consumers of financial products
To maintain confidence in the financial system

(ii)

Direct cost

administering the regulation and compliance of firms

Economic cost

An alternation in the behaviour of consumers, who may be given a false


sense of security and a reduced sense of responsibility in their own actions
Undermining of the sense of professional responsibility amongst
intermediaries and advisors
A reduction in consumer protection mechanisms by the market itself
Reduced product innovation
Reduced competition

Money Market Instruments (Unit 5)


(i)

Term loans
Evergreen credit
Revolving credit
Bridging loans
International bank loans
Trade credit

(ii)

Commitment
Maturity
Rate of interest
Security

Page 3

Subject ST5 (Finance and Investment Specialist Technical A) September 2007 Examiners Report

Money Market Instruments (Unit 5)


(i)

Company A, AAA or AA with supporting argument


Company B, AA or A with supporting argument
Company C, BBB/Junk Bond

(ii)

EDL = Value of Treasury Bond Value of Corporate Bond


The expected default loss will go up from A through to C

(iii)

(a)

The following yields or similar would be expected (different yields


would impact subsequent calculations illustrated).
Company
Company A
Company B
Company C
Treasury Bond

(b)

Yield
4.75%
6.00%
8.00%
4.25%

Assume all trade at par.


Treasury Bond = 95.84
Company A = 95.36 loss = 0.48
Company B = 94.18 loss = 1.66
Company C = 92.31 loss = 3.53

(iv)

All else being equal, all bond yields will increase by 100bps to reflect the
change in government bond yields.
However, the equity market has fallen which would imply that there is
concern about the corporate sectors future economic prospects or that
earnings have fallen.
This would suggest a widening of spreads on corporate borrowings relative to
government debt.
The impact on each of the three companies will vary depending on the
sensitivity of their existing and future revenue streams to the factors causing
the economic downturn.
In practice, it is likely that there would be a flight to quality reflecting
reduced liquidity in poorer credits and greater concerns about defaults.
This would mean that demand for higher quality bonds increases and the
demand for lower quality bonds reduces.

Page 4

Subject ST5 (Finance and Investment Specialist Technical A) September 2007 Examiners Report

This would imply that the spreads for companies A/B/C might increase from
50/175/375 bps to 5075/200250/425500bps, before allowing for company
specific factors.

Derivatives (Unit 7)
(i)

The main uses of derivatives are as follows:

Speculation
Exchange-traded derivatives could be used for speculation; effectively
betting on a strong view of a particular market movement. The difference
between speculation using options and speculation using the underlying
asset is that buying the underlying asset requires an initial cash outlay
equal to the total value of what is bought whereas entering into a future
contract or an option contract requires only a fraction of the initial cash
outlay. Thus a much higher level of leverage (gearing) can be achieved.

Arbitrage
Arbitrage involves locking in a riskless profit by simultaneously entering
into two transactions in two or more markets. Using various combinations
of options and the underlying instruments, portfolios with the same return
but with different constituent parts can be created. Arbitrage opportunities
can arise when the prices of these different portfolios get out of alignment
and a riskless profit can be made.
In practice only very small arbitrage opportunities are observed in prices
that are quoted in most financial markets. Also, transaction costs would
probably eliminate the profit for all but the very large investment houses
that face very low transaction costs.

Hedging
Hedging allows a fund manager to reduce a risk that the fund already
faces. Hedging using options, for example, involves taking a long or short
position in a number of options contracts which is the opposite to the
position held in the underlying asset. Conceptually, a loss made in the
underlying asset will be offset by an approximately equal gain on the
options position.
This technique is very useful where say a fund is going to sell its holding
in two or three months and it wishes to avoid a fall in market values.
However, if the market rises there will be a loss on the futures position
approximately equal in value to gain on the underlying equities so the
strategy does close off the opportunity for the fund to participate in any
upward movements in the underlying assets while the hedge is in place.

Page 5

Subject ST5 (Finance and Investment Specialist Technical A) September 2007 Examiners Report

Portfolio management
Options can be used to manage the reallocation of assets from one market
to another. For example, call options on equity indices can be used to gain
exposure to upside movements in the markets; put options can be used to
remove exposure to downside movements in markets. Calls and puts can
be used to change a funds exposure to an asset category or to change a
funds exposure within an asset category.

(ii)

(a)

Margin definition Unit 7 pg1.

(b)

Initial Margin the initial payment put down to cover the risk of the
contract.
Variation Margin the margin which is payable or received on a
daily basis to mark to market.

(iii)

(c)

The clearing house is removing the credit risk and they need some
form of compensation to cover themselves for this risk.

(a)

Put the right to sell an underlying asset for a certain price by a


certain date.
Call the right to buy an underlying asset for a certain price by a
certain date.

(b)

(iv)

American exercised at any date to expiry. European can only be


exercised on expiry date.

Call the loss on the 3 month is 5p to 80p when break even then in the money.
6 month is 10p loss until 85p.
Put 3 month is in profit to 75p then loss of option, for 6 month is in loss at
80p.

Page 6

Subject ST5 (Finance and Investment Specialist Technical A) September 2007 Examiners Report

Government Policy (Unit 11)


(i)

Main forms of government policy


Monetary Policy
Fiscal Policy
National debt management policy
Exchange rate policy
Prices and income policy

(ii)

Main economic indicators


Unemployment
Inflation
Balance of Payments
Economic Growth

(iii)

(a)

Individuals reduce variable rate mortgage and debt payments


(making people better off). Consumers likely to increase spending on
non-essential services and luxury items. Reduced rate of interest will
act as a disincentive for cash savings, although this may be offset by
reduced debt payments for some. Imported goods likely to increase in
cost due to impact on exchange rates.

(b)

Businesses Capital investment and economic growth likely to


increase due to reduced opportunity cost of committing funds.
Increased economic activity likely, offset by lower domestic currency
rates which will increase cost of imports.

(c)

Economy increased inflation and growth expectations (from a low


base), possible recovery and increase in inward foreign investment.
Longer term concerns about uncontrolled inflation may begin to
emerge.

Page 7

Subject ST5 (Finance and Investment Specialist Technical A) September 2007 Examiners Report

Hedge Funds (Unit 5)


(i)

(a)

Global Funds
Event-driven Funds
Market Neutral Funds

(b)

Describe the main investment characteristics of a Hedge Fund


Placing large bets on different asset classes
High level of borrowing
Mix of investments
Willingness to trade in derivatives
Illiquid
High Fees
High risk

(ii)

Partial market coverage


Survivorship bias
Selection bias
Marking to market bias

(iii)

Issues for scheme, trustees and sponsors


Capital management or financing
Pension management

Liability cash flow management


Compensation for inadequacies of bond markets

Deficit reduction
Regulatory, financial and peer group pressures
Finite repair term imposed by regulators scope for absolute return
structures

Surplus control
Short term fix may prove over cautious

Sponsor may be more concerned with volatility of deficit and


earnings/corporate activity impacts rather than size of deficit itself

Page 8

Cash flow hedging coupled with Regulator proposals for Deficit Repair
terms change pension funds from Relative (to yields, inflation) to
Absolute/Targeted Return investors
Alternative assets, including hedge funds, have obvious role in Deficit
Repair

Subject ST5 (Finance and Investment Specialist Technical A) September 2007 Examiners Report

Market neutral hedge funds plus swaps/bonds equal Liabilities Plus fund

Bulk of exposure is through Fund of Funds

Hedge funds have a role as the alpha generator in new products, but could lose
their separate identity

(i)

Not all managers, perhaps even the majority, are skilful

Need a rigorous and different process to identify and invest in persistent


skill

Operational risk is the risk of loss resulting from inadequate or failed internal
processes, people, and systems or from external events.
Operational risk includes IT, legal and compliance risk.
Operational risk differs from market or credit risk as it is typically not directly
taken in return for an expected reward
Operational risk exists in the natural course of corporate activity
Operational risk is more difficult to quantify and measure compare with
market or credit risk.
Operational risk is very important as it seems to have been responsible for
more spectacular corporate losses than the other financial risks.

(ii)

The control of operational risk essentially depends on good management


practices.
To manage operational risk, the investment manager would need to identify,
assess, put together a series of risk mitigation strategies one for each nontrivial risk.
Good management practices include:
Established and documented chains of reporting and responsibility
Separation of duties as between say, front office and back office staff in the
issuing of trading instructions and the confirmation & settlement of such
instructions.
Documented and robust procedure for carrying out essential tasks and for
taking on new activities or developing new products
Ongoing monitoring of risks and their mitigation techniques

Page 9

Subject ST5 (Finance and Investment Specialist Technical A) September 2007 Examiners Report

(i)

A listings authority is responsible for ensuring that any new issue of shares is
conducted in an orderly and fair way, and that the conduct of the company
remains consistent with the listing of the shares after the issue.
A listing authority will ensure that a reasonable amount of financial
information is in the public domain.
Listing authorities are normally concerned with:

(ii)

The production of relevant business and financial information on the issue


of shares.

The process by which shares are offered to potential shareholders and the
price is set for the issue of shares.

Continuing production and dissemination of business and financial


information on a timely basis on companies with listed securities.

The continuing conduct of the market in listed securities with a view to


ensuring that the market is fair to all participants, and that the pricing
process is fair and reasonable.

Rules to ensure that companies with listed securities and connected parties
continue to behave in a manner that does conflict with other objectives of
the listing authority.

The value of the company will be driven by the level and likelihood of future
profits.
General factors

Page 10

Information about the management and an assessment of their ability to


deliver in a public company.
The type and quality of the products sold
Prospects for market growth
How the company fairs against the competition
Details of operating costs
Details of past retained profits
The history of the company

Subject ST5 (Finance and Investment Specialist Technical A) September 2007 Examiners Report

Financial measures

Financial accounts and accounting ratios


Dividend and earnings cover
Profit variability and growth
Level of borrowing
Level of liquidity
Growth in asset values
Comparative figures for other similar companies

END OF EXAMINERS REPORT

Page 11

Faculty of Actuaries

Institute of Actuaries

EXAMINATION
16 April 2008 (pm)

Subject ST5 Finance and Investment


Specialist Technical A
Time allowed: Three hours
INSTRUCTIONS TO THE CANDIDATE
1.

Enter all the candidate and examination details as requested on the front of your answer
booklet.

2.

You have 15 minutes at the start of the examination in which to read the questions.
You are strongly encouraged to use this time for reading only, but notes may be made.
You then have three hours to complete the paper.

3.

You must not start writing your answers in the booklet until instructed to do so by the
supervisor.

4.

Mark allocations are shown in brackets.

5.

Attempt all 7 questions, beginning your answer to each question on a separate sheet.

6.

Candidates should show calculations where this is appropriate.

AT THE END OF THE EXAMINATION


Hand in BOTH your answer booklet, with any additional sheets firmly attached, and this
question paper.
In addition to this paper you should have available the 2002 edition of the Formulae
and Tables and your own electronic calculator from the approved list.

ST5 A2008

Faculty of Actuaries
Institute of Actuaries

(i)

List the types of activity central banks engage in.

[3]

(ii)

Define liquidity risk.

[2]

(iii)

Explain why it might be desirable for a central bank to act as a lender of last
resort to private sector banks, commenting on the nature of banking assets and
liabilities.
[4]

(iv)

Outline the disadvantages of there being a lender of last resort system in place.
[3]

(v)

Describe ways in which these disadvantages could be mitigated.

[2]
[Total 14]

The trustees of a UK pension fund with 800 million in actively managed assets are
looking to restructure the assets in order to more closely match the liabilities. The
current and target structures of the assets are given in the table below. Assets are
managed by three managers currently. All three managers are to be replaced with two
new managers.
Current
Assets
US equities
UK equities
Emerging market equities
Private equity
UK gilts
Total

Current Value
(m)
400
100
100
150
50
800

Target
Assets

Target
Value (m)

UK equities
UK gilts
Overseas bonds

300
400
100

Total

800

The trustees want to move to the target structure immediately but have not yet chosen
the managers for the target structure.
(i)

Describe the biggest mismatches between the current and target portfolios. [2]

(ii)

Outline how the trustees can move the assets towards the target structure
before the new managers for the target portfolio have been appointed.

[4]

Outline the limitations and downsides of the strategies proposed.

[4]

(iii)

The trustees finally decide on the target managers and want to go ahead with the
move to the new structure, however market conditions have changed and liquidity has
decreased and volatility has increased.
(iv)

List the various costs that are incurred when transferring assets.

(v)

Describe how the costs identified in (iv) will be affected by the current market
conditions.
[3]
[Total 15]

ST5 A20082

[2]

You are working for a life office in their investment team and have been presented
with the opportunity to buy for 150m a freehold on an office block that is currently
occupied by a bank. Two years ago, the bank had arranged a 32 year lease with the
current freeholder, as follows:
Term of lease

32 years

Annual rent

First 3 years 9m p.a., payable annually in advance


Thereafter

9m p.a. plus 5 years cumulative inflation,


increasing in line with future inflation, payable
annually in advance

Ground rent

100 p.a. payable annually in advance

Inflation lag

3 months

(i)

Write down an equation for the present value of the remaining rental
payments, expressed in terms of zero coupon interest rates (zt) and inflation
rates (zinf,t)
[3]

When the bank hears that the freehold is in the process of being sold, it offers to set
up an inflation swap to exchange the inflation-linked rental payments for fixed
payments. This would be a separate contract to the lease, and would be subject to
daily collateralisation.

(ii)

Describe the cashflows that would be paid and received under the inflation
swap with the bank.
[4]

(iii)

Explain why the life office might feel the inflation swap makes this transaction
more attractive, despite paying a margin to the bank arranging the swap. [2]

(iv)

Describe the various risks that apply to the life office under the freehold, the
lease and the swap, and explain how they might vary over time and according
to economic factors.
[9]
[Total 18]

(i)

Describe the two principal global equity index series.

(ii)

List five ways in which a large institutional investor can achieve the returns
(gross of costs and tax) of a global equity index.
[3]

(iii)

Explain why this type of index would be more useful for performance
measurement for an overseas investor than the most widely quoted local
equity index.

(iv)

[2]

[2]

Explain why this type of index would be less suitable than the most widely
quoted local equity index as a base for exchange-traded derivative contracts.
[5]
[Total 12]

ST5 A20083

PLEASE TURN OVER

(i)

List the principal aims of financial regulation.

[2]

(ii)

(a)

Define a self-regulatory system.

(b)

Discuss the advantages and disadvantages of self regulation in the


investment management and the securities industries.
[6]
[Total 8]

A risk averse individual coming up to retirement age has around 1% of their


retirement funds invested in the shares of a highly respected bank. The bank has
historically been involved in low risk activities, producing steady returns. Since new
management was put in place 3 years ago the bank has been involved in a number of
high profile risky investments which have gone wrong. As a result there has been a
sharp decline in the share price.
(i)

Outline how the change in management has affected the risk and return profile
of the individuals investment portfolio.
[2]

The individual had the opportunity to sell the stock 6 months ago, but decided to hold
onto the stock. Since then, the share price has fallen further.
(ii)

Outline the various reasons why the stock might not have been sold.

[5]

(iii)

Discuss whether the investment is suitable for the individuals circumstances.


[2]

The investor believes the share price has reached its lowest point and expects it to rise
in the near future. The investor wants to try to make back some of their losses.
(iv)

Describe a technique, using the current share price, that the investor could use
to make a profit on their holding without selling any shares.
[2]

The investor decides to use the technique in (iv).


(v)

Describe the effect on the investors exposure to the bank if the banks share
price rose by 30%.
[2]

(vi)

Describe the effect on the investors exposure to the bank if the banks share
price fell by 30%.
[2]
[Total 15]

ST5 A20084

(i)

(ii)

(iii)

List six factors that are important to take into account when valuing a
company.

[3]

List six sources of information that an analyst may use when valuing a
company.

[3]

Describe how the P/E ratios of the following types of company may change
through an economic cycle:
(a)
(b)
(c)

a house builder
a tobacco company
a retail bank
[9]

Rather than look at P/E ratios an analyst has decided to value the companies within
his sector on a discounted cash flow basis.
(iv)

Discuss the advantages and disadvantages of adopting this methodology. [3]


[Total 18]

END OF PAPER

ST5 A20085

Faculty of Actuaries

Institute of Actuaries

Subject ST5 Finance and Investment


Specialist Technical A
EXAMINERS REPORT
April 2008

Introduction
The attached subject report has been written by the Principal Examiner with the aim of
helping candidates. The questions and comments are based around Core Reading as the
interpretation of the syllabus to which the examiners are working. They have however given
credit for any alternative approach or interpretation which they consider to be reasonable.

M A Stocker
Chairman of the Board of Examiners
June 2008

Faculty of Actuaries
Institute of Actuaries

Subject ST5 (Finance & Investment Specialist Technical A) April 2008 Examiners Report

Comments
Most candidates scored well on questions 5, 7 and to a lesser extent 2, with many achieving
close to full marks. Questions 4, 6 and 3 were the worst answered (3 in particular, with the
average candidate achieving less than a third of the marks available).
In every diet there will be candidates who are very close to the pass mark and yet receive an
FA indeed candidates would be very surprised to see just how tightly distributed the marks
are; deciding where the pass mark falls will have a material impact on the numbers of
candidates who are successful and the examiners take great care to ensure a consistency of
standard across candidates, subjects and diets. That said, it was fairly clear where the
hurdle should have been set with a clear distinction between candidates graded as a Pass
and not. The examiners were disappointed to see that the pass rate for this diet was slightly
lower than last time given the pass mark was lower too. Where candidates scored lower it
was typically because although they were able to reproduce the required bookwork for one or
other question, they were unable to apply the bookwork knowledge appropriately. Few
candidates provided satisfactory answers to calculation questions.
Given the intent of the profession to push out in to wider fields involving the practical
application of actuarial skills in financial risk management and the increasing numbers of
candidates sitting this exam, it continues to be a disappointment that many candidates
achieve such low scores. Indeed, it is most astonishing the numbers who achieve grades of
FC and FD since this would imply very little knowledge and understanding even after a
course of study.
Candidates should note the bias in the paper towards recognising higher level skills and
practical application this is intentional and will continue. Likewise the examination system
does properly allow for prior subject knowledge to be assumed. Investment is a necessarily
practical subject and at this level, the examiners expect candidates to demonstrate a breadth
and depth of competency as would be expected from a practising actuary or senior student in
a frequently evolving discipline, particularly for those looking to progress to SA6. Hence
simple regurgitation of bookwork will never be sufficient to ensure a Pass grade.
As noted before, in order to succeed, candidates must ensure they familiarise themselves with
the prevailing investment issues and the general market background facing institutional
investors in the 18 months preceding a diet, more so the solutions (and sources of) being
debated by the various stakeholders. A recurring theme in recent years has been a move
towards capital market rather than purely insurance and asset management solutions hence
questions regarding corporate finance, banking and derivative approaches to asset and
liability risk management or modern financial theory and commercial applications should be
considered likely scope for examination. Likewise regulation and globalisation are common
issues in many areas.
All extenuating and mitigating circumstances were considered in awarding grades.

Page 2

Subject ST5 (Finance & Investment Specialist Technical A) April 2008 Examiners Report

(i)

determining monetary, interest rate and inflation policy


regulation of banks
implementation of government borrowing
ensuring the performance and integrity of financial markets
intervention in currency markets
printing and minting of notes and coins
taxation
lender of last resort

(ii)

Liquidity risk is the risk that asset owner is unable to recover full value of
asset when sale is desired (or for a borrower, the risk of credit being
unavailable when an maturing loan needs to be refinanced/rolled over)

(iii)

Banks typically hold significant amounts of medium to long dated loan assets
on their balance sheets, which are highly illiquid. In contrast, their liabilities
will typically be shorter-term in nature, including deposits and shorter-term
inter-bank loans.
Without a lender of last resort (LOLR), a bank is exposed to the risk that it
will not be able to maintain payments to its creditors if sufficiently many
deposits are withdrawn or if it is unable to refinance maturing loan payments.
The perception that a bank is nearing such a position can lead to a run on the
bank as deposits are easily withdrawn which can have wider social and
economic impacts.
With a LOLR, a bank is much less likely to end up in such a position. Hence
most developed economies have a LOLR system in place, explicitly or
implicitly.

(iv)

The key disadvantage is moral hazard. The management of banks would have
a weaker incentive to manage liquidity (by term of cashflows under assets and
liabilities) as carefully as if a LOLR was not present. This reflects that in the
latter scenario a bank would become insolvent and either require bailing out
by an acquirer, or creditors to the bank would incur losses (and shareholders
would almost certainly see their capital extinguished, and management would
lose their jobs).
Moral hazard can also be argued to extend to depositors: with a LOLR system
a depositor would not need to assess the credit standing of the bank accepting
the deposit.
Most countries have taken the view that this latter aspect of moral hazard is
acceptable, whereas the former is less acceptable (except where necessary to
prevent contagion to other financial institutions).

Page 3

Subject ST5 (Finance & Investment Specialist Technical A) April 2008 Examiners Report

The other key disadvantage is whether the losses that might accrue to the
central bank under the LOLR system would ultimately be borne by taxpayers.
However in the long run it is not good for the economy for an inefficient
business to receive such support.
(v)

Moral hazard and central bank losses can be reduced by ensuring that banks
borrowing from the LOLR pay a penal rate of interest on loans. In reality this
is unlikely to compensate for the credit risks associated with this type of
lending.
Other measures may include requiring additional collateral for LOLR loans,
nationalisation of a failing bank and ensuring that all other sources of
financing have been explored including acquisitions or other marriage
broking.
Regulation of liquidity management, asset quality, approved persons for
management may also mitigate the disadvantages.

(i)

Asset class mismatch between equities and bonds


Country mismatch between overseas assets and UK assets
No private equity holding going forward

Use of futures/derivatives/swaps to make current assets look like the target


assets
Detailed example e.g.: Short US equity futures by 400m, emerging
market 100m, private equity 150m
Long UK equity 200m, UK gilts 350m, overseas bonds 100m
Ask current managers to restructure their current holdings to replicate asset
classes using physical holdings

(ii)

(iii)

Page 4

Futures might not have suitable futures, especially private equity,


basis risk between active manager and future benchmark,
costs of putting on derivative position,
extra management required to manage future contracts
Physical sell investment costs (commissions, taxes) of moving to interim
strategy and then extra costs of moving to final strategy once selected.
Existing managers might not have capability of managing in other asset
classes.

Subject ST5 (Finance & Investment Specialist Technical A) April 2008 Examiners Report

(iv)

Taxes, commissions, spread costs, market impact costs, FX costs,


administration and custodian costs

Taxes, commissions likely to be unchanged


Spreads widen
Market impact costs increased
Custodian costs likely to increase as larger number of split trades due to
poor liquidity
Probably moving to more liquid/tradable currencies within overseas bond
exposures but will need to sell out of emerging positions

(v)

(i)

Reasonable formulae, with consistent assumptions, such as:

z inf 0.25 29
PV = 9m 1 + 1
1 + ztinf 0.25

Inf
1 0.25 5 t =1

) (1 + z )
t

29
t
+

100

(1 + zt )
t =0

where Inft is the realised inflation index at time t, zt is the zero coupon bond
rate at time t and ztinf 0.25 is the inflation rate at time t (with 3 month lag)
credit also available for assuming that the inflation rate is a series of forwards, in
which case the formula would be as follows:

z inf 0.25 29 t
PV = 9m 1 + 1
1 + ztinf 0.25

Inf
1 0.25 5 t =1 i =1

(ii)

)(1 + z )

29
t
+

100

(1 + zt )
t =0

The first 1 payment (at time t=0) would be excluded from the inflation swap
as there is no inflation linkage. The remaining 29 payments are inflation
linked.
Under these latter payments, the life office would pay out 9m pa plus 5
years known historic inflation (from 5y 3m ago to 3m ago, assuming a 3
month lag in obtaining inflation data) plus actual future inflation (from 3m ago
to the payment date less 3 months) at the time of each annual payment. In
return the life office would receive 9m pa plus 5 years known historic
inflation (from 5y 3m ago to 3m ago, assuming a 3 month lag in obtaining
inflation data) plus expected future inflation under the inflation swap curve
(from 3m ago to the payment date less 3 months).

(iii)

From a regulatory capital perspective the life office may find a fixed nominal
payment more attractive than an inflation-linked payment if its liabilities are
fixed in nature.
From a valuation perspective, the life office may feel that inflation is an asset
that is worth selling if it expects inflation in the future to be lower than the
current breakeven rate of inflation in the swap markets.

Page 5

Subject ST5 (Finance & Investment Specialist Technical A) April 2008 Examiners Report

(iv)

The key sources of risk are:


Freehold

Downturn in the economy can lead to a decline in value as occupancy


levels fall. The long lease provides a degree of protection provided the
bank continues to occupy the building and maintains rental payments.

Oversupply of new office space leads to a decline in value as occupancy


levels fall. The long lease provides a degree of protection.

Location falls out of favour. Reversion value will therefore fall (relative to
similar properties in other locations), and lower rent likely to be realised
on a fresh lease. Long lease provides some protection.

Building design/specification falls out of favour. Reversion value will


therefore fall (relative to other properties in same locations), and lower
rent likely to be realised on a fresh lease. Long lease provides some
protection.

Lease

Tenant cancels lease. This becomes increasingly likely towards the end of
the lease as the tenant will be looking at its needs in the future, and there
may be fewer penalties for cancelling late in the term (assuming a market
rent is being charged, otherwise there would be an incentive to stay or sublet the building).
Earlier in the lease this is a possibility due to restructuring or M&A
activity (or possibly the default of the bank) a new tenant would need to
be found and potentially the inflation swap may no longer match the new
lease.

Tenant renegotiates lease. This could happen at any time during the lease,
and becomes more likely if economic factors are such that rental yields are
falling generally (depending on the terms of the lease). Mismatching issue
if the renegotiated lease breaks the direct inflation link assumed under the
inflation swap (eg move to fixed % increases each year, or rent review
based on rents on comparable properties).

Cost of fulfilling covenants or pursuing leaseholder to fulfil

Void risks. Under either scenario above, there is a risk of void


(incomplete/nil occupancy) which would lead to a loss of rental income.
Void risk increases with time and a downturn in the economy

Inflation swap

Page 6

Changed/broken/new lease. The inflation swap would need to be


cancelled/netted out or run off as the hedge against the inflation linkage in

Subject ST5 (Finance & Investment Specialist Technical A) April 2008 Examiners Report

the rental payments would no longer be valid. This would create further
costs and possible liquidity issues.

(i)

Default risk (if explained well)

FTSE World Indices


The FTSE World Indices cover over 80% of the worlds equity markets in
terms of market capitalisation and are broad market indices that aim to cover
the vast majority of the free float stocks that are available to overseas
investors. Index values are shown for each country, in 5 currencies: Sterling,
US Dollar, Japanese Yen, Euro and the local currency. There are also regional
groupings of countries (weighted by market cap) and an All-World Index.
Morgan Stanley Capital International Indices
This is a series of international equity indices covering both developed and
emerging markets. They are calculated on a market capitalisation weighted
arithmetic basis and total returns are published both gross and net of
withholding tax, and in US Dollar, Euro and local currencies.

(ii)

Replication in tracker portfolio


Sampling
ETF
Buy futures
Buy assets (e.g. cash or stocks) and engage in an OTC swap to pay the return
on the assets and receive equity market return

(iii)

Stocks not available to foreign investors are not included in these indices. This
is not the case for most local indices, so these are often more suitable for
performance measurement purposes than local indices.
Some local indices are weighted averages or total return based
They have a consistent methodology between countries.
They are easier to obtain than some local indices (single data source).
Also, some local indices do not restrict constituent weightings to the free float,
which means they are unsuitable for performance measurement purposes.
Finally, there are index values shown net of withholding taxes and in various
currencies which may be helpful for an overseas investor.

(iv)

The key feature that is needed for an exchange-traded derivative contract is


liquidity. This means that the contract should be based on the most popular
local index to maximise demand from users of the derivative.
This is less of an issue for OTC markets, where client preferences will be
more important.
Liquidity of the underlying stocks is also important to ensure that the
derivative price closely tracks the underlying stocks (as pricing variations can
be arbitraged away rapidly).
This leads to a preference for large-cap stocks over small-cap stocks within the
underlying index.

Page 7

Subject ST5 (Finance & Investment Specialist Technical A) April 2008 Examiners Report

Using a large-cap index as opposed to a broad-market index also means that


the price of the derivative and index value are both available continuously
throughout the day, improving liquidity further.
Where the underlying index is not continuously quoted or the index
constituents are illiquid, it is likely that pricing of the derivatives will include
wider spreads to reflect this illiquidity and uncertainty.

(i)

The principal aims of regulation are:


to correct market inefficiencies and to promote efficient and orderly
markets
to protect consumers of financial products
to maintain confidence in the financial system
to help reduce financial crime

(ii)

A self-regulatory system is organised and operated by the participants in a


particular market without direct government intervention.
Advantages:
The system is implemented by the people with greatest market knowledge.
System is implemented by people who are incentivised to maximise cost
benefit ratio of regulation and ensure system is non-bureaucratic.
Should be more flexible than the alternatives and be able to respond rapidly to
changes in market needs.
Cooperation with a self-regulatory organisation may be more forthcoming
than with a government agency (but not necessarily).
Disadvantages:
Regulator will be perceived to be closer to industry than customers.
This can lead to a lack of confidence from consumers and purchasers
particularly when criticism of industry is high in the wider economy
Self-regulatory organisation likely to have fewer powers to fine and punish
industry members than a government agency established under statute.
Barriers to entry

Page 8

(i)

The bank has become a riskier investment there may be an additional sector
risk if this banks performance impacts or infers a wider contagion.
As the bank increases its risk the expected return investors seek from the
investment should also increase
However, the investment only makes 1% of portfolio so although bank has
increased in risk the impact at the total portfolio level should be minimal.

(ii)

Expect the share price to increase


Regret aversion by maintaining existing arrangements people minimise the
pain associated with feeling of loss
Overconfidence in their ability
Status Quo bias like to keep things the same
Good diversifier in overall portfolio

Subject ST5 (Finance & Investment Specialist Technical A) April 2008 Examiners Report

Could be part of an index tracking portfolio


Tax considerations
Income may be appreciated

(iii)

Individual is close to retirement hence would expect to be in less risky


investments where capital more guaranteed.
However, depends exactly when investor expects to retire.
Small part of overall portfolio so might be suitable if most other holdings are
relatively safe.
If majority of investments are equity then probably not suitable.

(iv)

Write options eg put options for a lower price than the current share price for
which the investor will receive a premium.
The closer to the current price the higher the premium they will receive.

(v)

Using example in (iv) to answer (v) and (vi):


Will keep premium
Current holdings will increase in value by 30%

(vi)

Will make loss on put options and


Would have to buy additional shares at agreed price
Making higher loss and increasing exposure to bank.
Existing holdings decrease in value

(i)

Factors that are important in valuing a company:


Management ability
Quality of products
Prospects for market growth
Competition
Input costs
Retained profits
History

(ii)

Sources of information:
The financial press and other commercial information providers
The trade press
Published accounts
Public statements by the company
The exchange where the securities are listed
Government sources of statutory information that a company has to provide
Visits to the company
Discussions with the companys management
Discussions with competitors
Stockbrokers publications.

Page 9

Subject ST5 (Finance & Investment Specialist Technical A) April 2008 Examiners Report

(iii)

(a)

A house builder is economically sensitive.


The earnings will rise and fall with the economic cycle, they are
volatile.
Investors try to anticipate the future earnings profile.
When the economy is buoyant earnings will be high however investors
will be discounting an economic slowdown and therefore the p/e will
be lower than the average p/e for the company through the cycle.
Similarly when earnings are low the p/e is likely to be high as investors
look forward to an upswing in the economy.

(iv)

(b)

Tobacco companies are economically insensitive.


Therefore the p/e of the tobacco company is relatively insensitive to
the economic cycle.
Investors have no need to anticipate the future earnings profile.

(c)

A retail bank is economically sensitive


The earnings will rise and fall with the economic cycle, they are
volatile.
Investors try to anticipate the future earnings profile.

Discounted cash flow valuations can be very useful in valuing a company.


The result of a discounted cash flow number is an absolute number that can be
compared to other DCF valuations.
DCF valuations can be used where a company generates no profit and pays no
dividend for instance a start up.
It would depend on the sector being analysed, some sectors are more suited to
DCF valuations than others.
DCF valuations are more time consuming.
DCF valuations can be very sensitive to the assumptions made e.g. the
discount rate and the growth rate.
DCF can be difficult to explain to others.

END OF EXAMINERS REPORT

Page 10

Faculty of Actuaries

Institute of Actuaries

EXAMINATION
24 September 2008 (pm)

Subject ST5 Finance and Investment


Specialist Technical A
Time allowed: Three hours
INSTRUCTIONS TO THE CANDIDATE
1.

Enter all the candidate and examination details as requested on the front of your answer
booklet.

2.

You have 15 minutes at the start of the examination in which to read the questions.
You are strongly encouraged to use this time for reading only, but notes may be made.
You then have three hours to complete the paper.

3.

You must not start writing your answers in the booklet until instructed to do so by the
supervisor.

4.

Mark allocations are shown in brackets.

5.

Attempt all 7 questions, beginning your answer to each question on a separate sheet.

6.

Candidates should show calculations where this is appropriate.

AT THE END OF THE EXAMINATION


Hand in BOTH your answer booklet, with any additional sheets firmly attached, and this
question paper.

In addition to this paper you should have available the 2002 edition of the Formulae
and Tables and your own electronic calculator from the approved list.

ST5 S2008

Faculty of Actuaries
Institute of Actuaries

(i)

Set out a formula for calculating the information ratio for a portfolio that
contains two asset classes, defining all terms used.
Asset
Class

1
2
3
Risk Free
(ii)

Tracking
Error (%)

10.0
7.0
5.0
4.0

10.0
5.0
5.0

Asset Class
Correlations
1
2
3
1.00
-0.25
0.25

1.00
0.50

1.00

Using the above data, for each pair of asset classes calculate:
(a)
(b)

Annual
Return (%)

the proportions of each class that satisfy a minimum variance portfolio


the information ratio for each portfolio
[8]

(iii)

Comment on the results in (ii) (b).

(iv)

Explain the effects that gearing and being able to sell short might have on
portfolio returns and the information ratio.
[3]
[Total 16]

[2]

You are the property fund manager at a large life insurance company and have been
approached by a small retail chain that wishes to sell and leaseback the six stores in its
chain.
Discuss the factors that you would consider in determining the sale price.

[3]

[10]

A passive investment manager is planning to launch a high alpha fund with the aim of
outperforming a global index by 3% p.a. The investment manager believes the
outperformance can be generated from three sources:

gaining superior information


processing the information better
eliminating behavioural bias

(i)

(a)

Explain how outperformance might be generated from each of the


three sources.

(b)

Comment on whether the target return can be generated by use of these


sources.

(c)

Comment on whether the 3% p.a. target is reasonable.


[11]

(ii)

ST5 S20082

Discuss the implications the fund launch might have on the structure and
management of the investment management department.
[7]
[Total 18]

A developing country has recently established a stock exchange. There are currently
7 stocks listed, but only one pays a dividend. Dividends are currently paid to
investors free of tax. The company that operates the exchange wishes to create an
index to measure the performance of the listed companies.
(i)

Give the formula that could be used to calculate the index value.

[2]

It has been decided that the initial value of the index will be 10,000 and details of the
7 companies are:
Initial Market
Cap
Initial Price
Company
(EDUm)
(C)
A
B
C
D
E
F
G

500
300
200
700
800
900
1,000

200
150
50
350
400
100
500

Price at end
of first day
(C)

Price at end
of second day
(C)

205
151
50
345
402
102
505

205
155
52
348
380
103
503

(Initial Market Capitalisation is measured in millions of Equivalent Dollar Units


EDU and Prices are in Cents where 100 Cents = 1 EDU)
Company E had declared a dividend of 25 and the stock went ex-dividend at the
beginning of the second day.

(ii)

Calculate the value of the index at the end of day 1 and day 2.

[4]

(iii)

Calculate the total return produced by the index over the two days.

[2]

(iv)

Outline the practical problems that would be encountered by an overseas


pension fund investment manager in using this index as a basis for indextracking management.
[9]
[Total 17]

You are an investment manager working for the investment arm of a small life
insurance company that has used exchange-traded options as part of its equity
portfolio management.
(i)

List the uses of equity market indices.

(ii)

List the main features and characteristics of the main equity indices of UK,
USA, Japan, Germany and France.
[5]

(iii)

State the main uses of exchange-traded options.

(iv)

Outline the appropriateness of using exchange-traded options for your


company.
[4]
[Total 15]

ST5 S20083

[4]

[2]

PLEASE TURN OVER

A wealthy UK investor is considering buying shares in a luxury car producer based in


an emerging country. The unlisted car producer has been running for less than 12
months, but has decided to float its equity on the local stock exchange in 6 weeks
time. The investor has been offered shares at a price which is at a 10% discount to the
expected flotation price.
(i)

Outline the investigations that the investor should carry out prior to deciding
whether to invest in the new company.
[5]

(ii)

Outline the difficulties the investor might encounter when trying to research,
analyse and place a valuation on the company.
[2]

The investor expects the global economy to slow in the next 12 months and enter into
a recession.
(iii)

Discuss how the company and the quoted shares might be affected if the
investors predictions are correct.

[3]

The investor decides not to invest in the stock directly, but takes out a 3 month call
option on the stock at a price which is 5% below the expected flotation price of 50p.
The company floats later in the month. On the first day the stock increases by 3%
over the actual flotation price of 45p.

(iv)

Define the term out of the money for both a put option and a call option
giving a brief example of each.
[2]

(v)

Outline how the price of the call option is likely to have changed in the six
weeks since purchase.
[3]
[Total 15]

The government of an emerging country is encouraging local companies to set up


pension funds for their employees. Historically the government has provided
retirement provision and, therefore, there has been a lack of regulation of institutional
investments. You are a financial advisor to the government and have been asked to
develop a regulatory framework for the new pension funds.
(i)

State why it is important for there to be regulation of institutional investments.


[3]

(ii)

Outline the principles you would recommend adopting under legislation for
institutional investment.
[6]
[Total 9]

END OF PAPER

ST5 S20084

Faculty of Actuaries

Institute of Actuaries

Subject ST5 Finance and Investment


Specialist Technical A
EXAMINERS REPORT
September 2008

Introduction
The attached subject report has been written by the Principal Examiner with the aim of
helping candidates. The questions and comments are based around Core Reading as the
interpretation of the syllabus to which the examiners are working. They have however given
credit for any alternative approach or interpretation which they consider to be reasonable.
R D Muckart
Chairman of the Board of Examiners
December 2008

Faculty of Actuaries
Institute of Actuaries
30/12/2008

Subject ST5 (Finance and Investment Specialist Technical A) September 2008 Examiners Report

General comments
Generally a poorly answered paper than previous diets; although the pass rate was
consistent with recent diets, this equated to a disappointingly lower pass mark. Candidates
typically answered Questions 5 and 6 better than the others, with Questions 1 and 3
attracting the worst responses.
Many candidates seemed to understand the key issues being examined and so appreciated the
general content of solutions that the examiners were looking for however those that were
unsuccessful will find their solutions lacked sufficient (and often the most basic) detail and
scored lower accordingly. Worse, some candidates deviated from the topic and included
irrelevant material although candidates will not be explicitly penalised for this, it gives an
impression of a lack of understanding and, more importantly, wastes valuable time. Where
candidates made relevant points in other parts of their solutions, the examiners have used
their discretion as to whether to recognise these answers or not.
Again there were many candidates close to the pass mark whom were awarded an FA most
candidates would be very surprised to see just how tightly distributed the marks are; deciding
where the pass mark falls will have a material impact on the numbers of candidates who are
successful and the examiners take great care to ensure a consistency of standard across
candidates, subjects and diets. It was fairly clear where the hurdle should have been set; as
a result, the pass rate for this diet was similar to last time. However the pass mark remains
much lower than the examiners feel ought to achievable by candidates, many of whom are
likely to be working as advisers or asset managers in this most practical of fields. Several
candidates were awarded an FD in this diet and the examiners remain concerned by the
numbers of candidates still achieving only an FC grade, since this too would imply little
preparation or, worse, knowledge and understanding.
Candidates are reminded of a bias in the paper towards recognising higher level skills and
practical application this is intentional and will continue. Likewise the examination system
does properly allow for prior subject knowledge to be assumed. Investment is a necessarily
practical subject and, at this level, the examiners expect candidates to demonstrate a breadth
and depth of competency as would be expected from a senior student in a frequently evolving
discipline. Hence simple regurgitation of bookwork will never be sufficient to ensure a Pass
grade and this was evident from the dispersion of candidates responses in the more
differentiating questions.
As noted before, in order to succeed, candidates must ensure they familiarise themselves with
the prevailing investment issues and the general market background facing institutional
investors in the 18 months preceding a diet, more so the solutions (and sources of) being
debated by the various stakeholders. A recurring theme in recent years has been a move
towards capital market rather than purely insurance and asset management solutions hence
questions regarding banking and derivative approaches to asset and liability risk
management or modern financial theory and commercial applications should be considered
likely scope for examination. New asset classes and ways of investment will themselves
generate new types of risk and so the need for new regulation and ways of monitoring and
management.
All extenuating and mitigating circumstances were considered in awarding grades.

Page 2
30/12/2008

Subject ST5 (Finance and Investment Specialist Technical A) September 2008 Examiners Report

(i)

IR = [ri + (1 - )rj rf]/[2i2 + (1 - )2j2 +2(1 - )ijij]


where = proportion in asset class i
ri, rj = annual return for asset class i, j i = 1, 2 i < j 3
rf = risk free return
i, j = tracking error asset class i, j
ij = correlation between asset classes i and j.

(ii)

To satisfy minimum variance differentiate denominator of IR with respect to


and set to zero. This gives a solution of
= [j2 ijij] / [i2 + j2 - 2ijij]
Asset Pair

r (%)

(%)

IR

1,2
1,3
2,3

0.25
0.125
0.5

3.75
1.625
2.00

3.95
4.84
4.33

0.95
0.34
0.46

(iii)

Only the combination of assets 1 and 2 gives a better IR than asset1 or asset 2
on their own. The minimum variance portfolio does not give the best IR for
each combination. This outlines the problem of using minimum variance
portfolios in building a portfolio.

(iv)

Gearing will give additional returns provided the cost of gearing is less than
the assets return. The debt is usually at a fixed rate of interest and so has no
variance. Thus although higher returns are achieved they will be at the cost of
higher risk (tracking error) unless the cost of debt is less than the risk free rate
(unlikely). Thus the information ratio will fall.
Selling short an asset that has a lower expected return and re-investing in a
higher returning asset will increase the return but will significantly increase
the risk. However if the assets are moderately to highly correlated and the
asset being bought has a sufficiently high risk adjusted return over the asset
being sold it will be possible to have an improved IR. However such
situations tend to be arbitraged away very quickly.

The most important consideration is the retailers quality and financial strength as the
rental income from all the properties in the portfolio are dependent on these factors.
It is therefore important to assess the financial position of the company both pre and
post the sale and leaseback.
Why is the company looking to sell the properties?
What will it be doing with the money it raises?
How long has the company been trading?

Page 3
30/12/2008

Subject ST5 (Finance and Investment Specialist Technical A) September 2008 Examiners Report

How long has the current management team been running the chain, do they have any
plans for retirement/succession?
What future plans do the management have for the chain?
Has it been through more than one economic cycle? If so how did it perform during a
recession? If not does it sell necessities or luxury items?
The properties themselves are also important.
Their location both in terms of which towns and the location within the town.
Age and the state of repair.
Could other retailers easily move into the space?
The lease details would need to be determined.
How long are the leases?
Are there any break clauses?
Are the leases on upward only rent reviews?
How often are rents reviewed?
What rent will be paid initially?
Finally details of the yields on similar properties would need to be ascertained.

(i)

(a)+(b) Alpha is the difference between a funds expected returns based on its
beta and its actual returns. Alpha is sometimes called the value that a
portfolio manager adds to the performance. If a fund returns more than
what you'd expect given its beta, it has a positive alpha. If a fund
returns less than its beta predicts, it has a negative alpha.
Looking at the three strands separately:

Gaining superior information

This should not be confused with inside information.


In practice this is probably the most difficult of the three areas to gain a
competitive edge.
Information can be gained from a number of sources the company
itself, their competitors, their customers, their suppliers, the press etc.

Page 4
30/12/2008

Subject ST5 (Finance and Investment Specialist Technical A) September 2008 Examiners Report

In order to gain superior information an analyst will probably need to


spend more time researching the company and its industry.
This has implications for the number of stocks an analyst can cover
and therefore for the total number of analysts required.

Processing the information better

This is an area that has received a lot of attention in the recent past
with the advent of quantitative models.
These models may allow investors to better identity anomalies and
thereby make better decisions.
Given the large amount of data available a system that processes this
information better may well lead to improved or more rapid decision
making.
Given the wide variety of information available any processing system
will have to be very flexible.
Better models of companies and sectors can also be developed to better
predict the future profitability and cash flows of a company or sector.
Again given the diversity of companies and sectors it is difficult to
devise a financial model that will be applicable to all companies.

Eliminating behavioural bias

This is easier said than done.


There are a number of behavioural biases.
Namely: Anchoring, loss aversion, framing, over confidence and
mental accounting.
Elimination of all these sources of bias would take considerable
expertise in the field of behavioural finance.
In order to effectively eliminate these biases some form of mechanistic
investment process may be required.
If it were possible to eliminate behavioural bias this would not
necessarily lead to the generation of alpha as if other investors were
still influenced by behavioural bias these biases could influence asset
prices.
Therefore the elimination of behavioural bias may lead to better long
term performance but it may have a detrimental impact on short term
performance.
Page 5
30/12/2008

Subject ST5 (Finance and Investment Specialist Technical A) September 2008 Examiners Report

(c)

Given/depending on the arguments advanced in (b) it would seem that


the 3% target is challenging, especially allowing for fees of
management. It is possible that such a performance is achievable in
the short term (and this would be borne out by historic manager return
statistics), but may be much harder to sustain a long term competitive
advantage, especially as other managers may come to adopt similar
styles and so arbitrage out the scope for added value.
Credit will be given for other reasoned arguments.

(ii)

In order to implement such a strategy the department would need:

A large number of analysts unless the aim was only to cover a limited part
of the market.

A quantitative team.

Experts in behavioural finance.

A data entry function to input the large amounts of data.

All these functions would need co-ordination and there would need a person or
people to reconcile the different recommendations and actually construct a
portfolio.
It is possible that an investment process with all these inputs may become
unwieldly.
There is also the possibility that the various input streams may produce
conflicting recommendations.
There is a danger that a portfolio constructed using these ideas could exhibit
abnormally high or low tracking errors. This would need careful explanation
to potential clients.
The attribution and explanation of performance would also be very complex.
The cost of such an operation may mean that this approach is only open to
fund managers with significant funds under management.

(i)

The formula is:


Index level at time t = K

Where

i W ( Pit / Pi 0 )
i Wi

Pit is the price of stock i at time t.


Pi0 is the price of stock i at the base date.
Wi is the weight applied to stock i.

Page 6
30/12/2008

Subject ST5 (Finance and Investment Specialist Technical A) September 2008 Examiners Report

K is a constant related to the starting value of the index at the base


date.
The weights used are usually the market capitalisations at the base
date.
The initial value if the index can be set at any number other than zero,
however it is usually a round number e.g. 1,000.
(ii)

From the equation above the value of K in this case = 10,000.


The index value at end of day 1 =
10,000*(512.5+302+200+690+804+918+1010)/4,400 = 10,082.95
The index value at the end of day 2 =
10,000*(512.5+310+208+696+760+927+1,006)/4,400
= 10,044.32

(iii)

In calculating a total return it is usually assumed that dividends are reinvested


back into the index at the ex-dividend date. In this case there was one dividend
of 25 and stock E went ex-dividend at the start of day 2.
Total return = ((10082.95+50/0.44)*10044.32/10082.95)/10,000
= 1.01575
i.e. 1.58%

(iv)

The investors in the fund must pay investment management fees, custody fees,
audit fees, governance fees and administration fees whereas such fees are not
taken into account in the calculation of returns on the Index.
The Index includes the reinvestment of gross dividends paid by its constituent
companies whereas the investment manager will only receive such dividends
net of withholding tax.
The Index does not take into account the costs of rebalancing the index for
such activities as new entrants, exits, mergers and takeovers and changes in
the market capitalisation of constituents.
Such costs include stockbrokers commissions, stamp duty and other levies.
When the fund manager receives small amounts of dividend income, it may
not be cost effective for her to invest such small amounts across the
constituents in the correct proportions.
The manager will therefore have part of the portfolio invested in the
constituents of the index and part invested in cash.
The cash holding will cause the manager to under perform the index in a rising
market and out perform the index in a falling market.

Page 7
30/12/2008

Subject ST5 (Finance and Investment Specialist Technical A) September 2008 Examiners Report

Needs to reflect situation of non-domestic investor and the ability to replicate


or otherwise track individual markets.
There may be limited derivatives available to develop synthetic approaches.

Problem with definition of emerging market. This will vary between investors
and index providers.
Lack of homogeneity means alternatives for stock/sector exposures may not be
closely correlated.
At individual market level and relevant weights, there may be foreign
ownership restrictions, different share classes and different definitions of
capitalisation according to free float.
Some markets may be very concentrated with associated liquidity issues. This
could have implications for investors with caps on exposures to particular
companies.
Marketability and availability of stock will vary and political instability can
cause capital control issues and so grounds for inclusion/exclusion within
index with limited notice of change.
For total return, income adjustment should reflect investor circumstances in
terms of reinvestment (actual receipt may be long after dividend declaration)
and taxation e.g. unrecoverable taxes.
Pricing and valuation information may be poor and untimely which will affect
dealing and monitoring of tracking.
Costs of dealing may be higher and may need to be reflected in judging
tracking success.
Restrictions on investment in certain countries imposed either by trustees or
regulation may render index less appropriate.
May have undue sector or stock biases versus total portfolio.
Research, administration, custody and dealing costs may be disproportionate
or difficult to facilitate.
Taxation will be a particular problem especially capital gains tax.
If making direct investment, unlikely to have portfolio similar to index.
Other practical management and monitoring issues.

Page 8
30/12/2008

Subject ST5 (Finance and Investment Specialist Technical A) September 2008 Examiners Report

(i)

Measure of short term market movements


Providing a history of market movements
Tool for estimating future movements in market given past trends
Benchmark for assessing portfolio performance
Valuing notional portfolio
Analysing sub-sectors of the market
As a basis for index funds to track a particular market
To provide basis for the creation of derivative instruments

UK -FTSE 100- largest 100 companies by market cap. Account for 80% of
total market. Weighted arithmetic average basis. Free float.
USA Dow, 30 shares. Unweighted arithmetic average.
S&P 500, weighted arithmetic index
Japan Nikkei 225 companies, unweighted arithmetic average
Topix 1100 shares, market cap weighted arithmetic
Germany DAX 30 shares, total return index
France CAC, 250 shares free float, market cap

(ii)

(iii)

Efficient portfolio or transition management i.e. Asset allocation,


equitising cash holdings
Long term and/or short term risk reduction i.e. Hedging strategic
exposures
Creating structured products with bespoke payoff profiles
Speculation

(iv)

Options are financial instruments that convey the right, but not the
obligation, to engage in a future transaction on some underlying security,
or in a futures contract.
Exchange traded options have standardized contracts, and are settled
through a clearing house with fulfillment guaranteed by the credit of the
exchange. Since the contracts are standardized, accurate pricing models
are often available.
Trading options entails the risk of the option's value changing over time.
However, unlike traditional securities, the return from holding an option
varies non-linearly with the value of the underlier and other factors.
A further, often ignored, risk in derivatives such as options is counterparty
risk. In an option contract this risk is that the seller won't sell or buy the
underlying asset as agreed. However exchange trading enables
independent parties to engage in price discovery and execute transactions.
As an intermediary to both sides of the transaction, the exchange provides:
- fulfillment of the contract is backed by the credit of the exchange,
which typically has the highest rating (AAA),
- counterparties remain anonymous,

Page 9
30/12/2008

Subject ST5 (Finance and Investment Specialist Technical A) September 2008 Examiners Report

enforcement of market regulation to ensure fairness and transparency,


and
- maintenance of orderly markets, especially during fast trading
conditions.
Basic options are to buy/sell puts/calls (and combinations thereof)
depending on investors view of the markets
As a small life office, ETOs offer low administration/efficient portfolio
management and the ability to hedge risks to comply with regulatory or
statutory requirements, particularly in volatile markets when fund
cashflows are uncertain. Similarly they can be combined with other
investments to create more attractive pay-off profiles with minimal
counter-party risk.
However like all standardised contracts, there will be inherent basis risk
between the option and the offices underlying holdings and there may be
cashflow or other trading risks if options are exercised.
Buying options involves paying a premium which may be subsequently
proven to have been wasted and so impact returns (and potentially
competitive positioning).
-

(i)

What investigations
Management ability
Quality of the cars/products
Prospects for market growth, market research and outlook for future
economy
Competition, who else makes the same type of cars. What is their business
model like
Input costs
R&D costs
Likely Profit
Marketing and sales strategy
The accounting ratios
Predicted level of borrowing

(ii)

Describe the difficulties.


Lack of publicly available information to analyse the company.
Lack of company history for profit and cost analysis.
Upfront costs of company setting up be higher than on-going.
Difficult to predict demand for new brand.
Lack of luxury car companies in same period of development for which to
benchmark.

(iii)

Describe how the company and the quoted shares might be affected.
Car company would be defined as consumer good, durable, cyclical.
As enter into recession PER will fall, share price likely to be depressed
Sales of new cars likely to fall to less demand.
Profits likely to decrease due to reduced demand and potential reduction in
price. Input costs likely to remain unchanged.

Page 10
30/12/2008

Subject ST5 (Finance and Investment Specialist Technical A) September 2008 Examiners Report

(iv)

Define the term out of the money for both a call option and a put option
giving a brief example for each option.
Out of money for a call option means the current share price is less than
the strike price attached to the option. Example if strike price is 250 and
current price is 200, out of the money.
Out of money on put means that the current price of share is higher than
the strike price attached to the option. Example current share price is 300
and the strike price attached to put option is 250.

(v)

Marks were given for reasoned arguments reflecting points such as:
There is a price to pay for the option which once added to current price
means out of the money initially.
The actual floatation price was lower than expected.
Price reflects volatility and time values.

(i)

Describe why it is important.


To protect the ultimate beneficiaries from gross incompetence or
mismanagement by fund managers.
To encourage confidence in investment schemes and the benefits they
secure.
To promote the accumulation of investible funds.

(ii)

The principles you would recommend.


Effective decision making decisions only taken by people with skill,
information etc.
Clear objectives Setting investment objectives which represent best
judgement on funds liabilities.
Take account of attitude to risk
Focus on asset allocation should set strategic asset allocation to be in line
with required risk return characteristics of individual fund circumstances
Expert advice need for expert advice for actuarial and investment advice
Explicit mandates Agree fund manager objectives, benchmark, risk
parameters. Understand the managers approach in attempting to achieve
objective.
Clear time scales of measurement and evaluation.
Activism Managers should have explicit strategy on activism
Appropriate Benchmarks explicit and benchmarks appropriate to assets
being invested in.
Default fund options.
Performance Measurement Have formal performance measurement
agreements including the period which reviewed.
Transparency Statement of investment principles on who is taking
decisions, the funds investment objective, planned asset allocation,
mandates given to managers, fee structures.
Regular Reporting members should be sent annual results of monitoring
of fund managers.

END OF EXAMINERS REPORT


Page 11
30/12/2008

Faculty of Actuaries

Institute of Actuaries

EXAMINATION
22 April 2009 (pm)

Subject ST5 Finance and Investment


Specialist Technical A
Time allowed: Three hours
INSTRUCTIONS TO THE CANDIDATE
1.

Enter all the candidate and examination details as requested on the front of your answer
booklet.

2.

You have 15 minutes before the start of the examination in which to read the
questions. You are strongly encouraged to use this time for reading only, but notes
may be made. You then have three hours to complete the paper.

3.

You must not start writing your answers in the booklet until instructed to do so by the
supervisor.

4.

Mark allocations are shown in brackets.

5.

Attempt all six questions, beginning your answer to each question on a separate sheet.

6.

Candidates should show calculations where this is appropriate.

Graph paper is required for this paper.

AT THE END OF THE EXAMINATION


Hand in BOTH your answer booklet, with any additional sheets firmly attached, and this
question paper.

In addition to this paper you should have available the 2002 edition of the Formulae
and Tables and your own electronic calculator from the approved list.

ST5 A2009

Faculty of Actuaries
Institute of Actuaries

(i)

Describe the uses of performance measurement for an investment portfolio.


[6]

(ii)

[8]

(iii)

Describe the key reasons why hedge fund index returns are likely to overstate
actual returns and understate volatility for a typical hedge fund investor. [4]

(iv)

State the formula for the Sharpe ratio, defining any terms you use.

[2]

(v)

Explain why hedge funds highlight the Sharpe ratio in their promotional
material, rather than the Treynor or Jensen ratios.

[2]

(vi)

Discuss the limitations and disadvantages associated with portfolio


performance measurement.

Describe the key limitations of the Sharpe ratio as a measure of the skill of a
hedge funds managers.
[5]
[Total 27]

You are the portfolio manager for a global equity pooled fund and have received a
quarterly analysis of companies in the European telecoms sector.
(i)

(ii)

(a)

Describe what you would expect to see in a high quality piece of


fundamental research of this nature.

(b)

List the factors that you might expect to see included in the numerical
analysis.
[6]

Outline the additional commentary that you would expect to see for a
company in the sector that is more highly leveraged than average, with a
significant amount of debt due to be repaid in the next two years.

[3]

Your company is considering launching two new global equity pooled funds, the
Global Equity (Higher Leverage) Fund and the Global Equity (Lower Leverage)
Fund. The intention is for a combined investment in the two new funds to broadly
correspond to an investment in the existing fund. The two funds will invest in the
same universe of underlying companies and can make the same buy/sell decisions as
the existing global equity fund. However, companies that are highly geared can only
be invested in by the Higher Leverage Fund.
(iii)

Discuss why a potential investor might find the choice of two new funds more
attractive than the existing global equity fund.
[7]

(iv)

Explain why an investor should not expect investments of $1m in each of the
two new funds to perform precisely in line with a single $2m investment in the
existing global equity fund.
[4]
[Total 20]

ST5 A20092

The trustees of a pension fund decide to purchase a three year swap contract under
which the pension fund will receive a fixed rate payment stream. The pension fund is
required to pay a floating rate payment stream in return. The pension fund receives
the following information about the swap and the likely payments:

Term 3 years
Notional value of swap 50m
Payments are made in arrears semi-annually
The swap year calculations assume there are 360 days in a year
Period

Number of days in period

Annual Forward Interest


Rate

1
2
3
4
5
6

183
181
182
182
181
183

4.00%
4.25%
4.5%
4.75%
5.0%
5.25%

(i)

Define the term puttable swap.

[1]

(ii)

Explain why a pension fund may wish to purchase a puttable swap.

[2]

(iii)

State the type of swap that the trustees have purchased.

[1]

(iv)

Using the information above calculate:


(a)
(b)

Present value of the floating rate payments.


The fixed rate of the swap.

[4]

The pension fund trustees proceed with the proposed contract for the payments
described above and the fixed rate of the swap is set at 4.75% pa.
(v)

Calculate the profit or loss to the pension fund at the end of the swap contract.
[4]

(vi)

Explain what difference there would have been to the profit/loss on the swap if
interest rates had risen during the duration of the swap contract.
[2]
[Total 14]

ST5 A20093

PLEASE TURN OVER

(i)

(a)

Define the term Warrant.

(b)

State two differences between a Warrant and an Option.


[2]

(ii)

(a)

Define the terms American call and European call.

(b)

Explain which one is likely to attract a higher premium.


[2]

(iii)

Draw a diagram for each of the following strategies and explain why an
investor may wish to undertake such strategies.
(a)

Long one call at a strike price of (X a)


Short two calls at a strike price of X
Long one call at a strike price of (X + a)
All three have same expiry date

(b)

Buying one call and one put with the same expiry and strike price

(c)

Buying call options of a certain strike price and selling the same
number of call options at a lower strike price (in the money) with the
same expiry date.
[6]

As part of an investors portfolio there are 100 call options that have been written
with an exercise price of 1.50 and an expiry date of November. The option premium
received was 0.50 per option.
(iv)

State the payoff for the investor.

[1]

(v)

Draw the payoff chart for the entire holding.

[2]

(vi)

Calculate the profit or loss to the investor if the price of the share at expiry is:
(a)
(b)
(c)

0.75
1.50
2.15

State any assumptions made.

[2]

The derivatives exchange where the call options are traded requires an initial margin
of 20% of the premium received. In addition a variation margin has to be paid equal to
100% of the option price movement. The value of the premium at the end of
September was 0.55.
(vii)

Calculate the total margin the investor has had to post to the exchange at the
end of September.
[2]
[Total 17]

ST5 A20094

(i)

Define the par yield of a bond.

[1]

(ii)

Write down the equation of value that needs to be satisfied by the par yield,
C2, of a two year bond (interest paid annually in arrears), in terms of the zero
coupon yield, ZCt, at time t.
[2]

(iii)

Calculate the zero coupon yields at times 1, 2 and 3 from the following par
yield curve, assuming coupons are paid annually in arrears:
Term

Par Yield

1
2
3
4

5.50%
5.40%
5.35%
5.30%
[6]

(iv)

Describe three techniques that can be used to identify bond anomaly switching
opportunities.
[6]
[Total 15]

(i)

(a)

Describe the primary purpose of an investment manager agreement.

(b)

List typical restrictions that might be included within an investment


manager agreement for a global bonds portfolio for a charity fund.
[3]

(ii)

Explain why agreements for active managers generally adopt a relatively


prescriptive approach, rather than giving the manager complete discretion in
how they achieve the target return.
[4]
[Total 7]

END OF PAPER

ST5 A20095

Faculty of Actuaries

Institute of Actuaries

Subject ST5 Finance and Investment


Specialist Technical A
EXAMINERS REPORT
April 2009

Introduction
The attached subject report has been written by the Principal Examiner with the aim of
helping candidates. The questions and comments are based around Core Reading as the
interpretation of the syllabus to which the examiners are working. They have however given
credit for any alternative approach or interpretation which they consider to be reasonable.
R D Muckart
Chairman of the Board of Examiners
July 2009

Faculty of Actuaries
Institute of Actuaries

Subject ST5 (Finance and Investment Specialist Technical A) April 2009 Examiners Report

Comments
Pleasingly, a better answered paper than previous diets leading to a higher pass rate even with a
higher pass mark. Candidates typically answered Questions 1, 4 and 5 much better than the others,
with Questions 2 and 3 attracting the worst responses, considerably so. This is not surprising given
that Questions 2 and 3 represented the opportunity to demonstrate higher level skills in terms of nonstandard/practical application of theory to current issues in investment hence candidates who wish
to progress to SA6 will need to improve their understanding of and approach to this type of question.
That said, most candidates seemed to identify and understand the key issues being examined and so
appreciated the general content of solutions that the examiners were looking for however those that
were unsuccessful will find their solutions lacked sufficient (and often the most basic) detail and
scored lower accordingly (this was most evident in Question 6). Many candidates still deviate from
the topic and include irrelevant material or over emphasise minor points although candidates will
not be explicitly penalised for this, it gives an impression of a lack of understanding and, more
importantly, wastes limited time. Time and priority management are key skills actuaries need to have.
Where candidates made relevant points in other parts of their solutions, the examiners have used their
discretion as to whether to recognise these answers or not. Likewise the examiners share and agree
alternative possible solutions to questions during the marking process.
Again there were many candidates close to the pass mark whom were awarded an FA most
candidates would be very surprised to see just how tightly distributed the marks are; deciding where
the pass mark falls will have a material impact on the numbers of candidates who are successful and
the examiners take great care to ensure a consistency of standard across candidates, subjects and
diets. Several candidates were awarded an FD in this diet and the examiners remain concerned by
the numbers of candidates still achieving only an FC grade, since this too would imply little
preparation or, worse, knowledge and understanding.
Candidates are reminded of a bias in the paper towards recognising higher level skills and practical
application this is intentional and will continue. Likewise the examination system does properly
allow for prior subject knowledge to be assumed. Investment is a necessarily practical subject and, at
this level, the examiners expect candidates to demonstrate a breadth and depth of competency as
would be expected from a senior student in a frequently evolving discipline. Hence simple
regurgitation of bookwork will never be sufficient to ensure a Pass grade and this was evident from
the dispersion of candidates responses in the more differentiating questions.
As noted before, in order to succeed, candidates must ensure they familiarise themselves with the
prevailing investment issues and the general market background facing institutional investors in the
18 months preceding a diet, more so the solutions (and sources of) being debated by the various
stakeholders. A recurring theme in recent years has been a move towards capital market rather than
purely insurance and asset management solutions hence questions regarding banking and
derivative approaches to asset and liability risk management or modern financial theory and
commercial applications should be considered likely scope for examination. New asset classes and
ways of structuring investment will themselves generate new types of risk (such as operations,
liquidity, credit and counterparty), so the need for new ways of monitoring and management.
All extenuating and mitigating circumstances were considered in awarding grades and, where there
was a genuine cause, credit given.

Page 2

Subject ST5 (Finance and Investment Specialist Technical A) April 2009 Examiners Report

(i)

There are various reasons why the performance of an investment portfolio will
be measured:
1. To improve future performance. First, data collected during performance
monitoring can form the inputs for planning future strategy. Secondly, if
fund managers know that their performance is being measured, it might
give them an extra incentive to maximise the returns of the funds they
manage.
2. Comparison of the rate achieved against a target rate. Many funds will
have one or more target rates of return. For example, the trustees of a
pension fund will want to know the rate of return achieved on the
investments compared with the rate of return assumed in the actuarial
valuation.
3. Comparison against the performance of other portfolios, an index and/or a
benchmark portfolio. Those responsible for the funds will want to know
how the performance of the portfolio compares with other portfolios. On
the basis of this information, they are able to make decisions regarding the
future investment of the assets, e.g. should a new fund manager be hired?
Also, by analysing the performance against a notional portfolio, it may be
possible to identify some relative strengths and/or weaknesses of
individual fund managers (e.g. in sector or stock selection).
Other reasons could include the assessment of performance related fees or
more generic assessments of success/failure of the portfolio.

(ii)

There are several limitations and disadvantages of portfolio performance


measurement.
Projection of past results: the fact that a particular result was attained in the
past does not mean that it will occur in the future. There is a random element
in investment returns and it may be difficult to determine how much a fund
managers results are due to method and how much to luck. Furthermore a
technique that proved successful in a particular set of circumstances may not
work so well in changed circumstances in the future.
Risk: in the long term we would expect a riskier strategy to produce higher
average returns. The measurement of relative performance should therefore
take account of the degree of risk taken on by a fund manager.
Timescale: determining the frequency of performance measurement
calculations requires a delicate balance between assessing performance
frequently enough so that problems can be spotted and corrected and avoiding
spurious conclusions based on too short a measurement period.
Differing fund objectives: different funds may have different objectives and
constraints. Comparisons between such funds may not be valid.

Page 3

Subject ST5 (Finance and Investment Specialist Technical A) April 2009 Examiners Report

Impact on fund manager behaviour: knowledge of how, and how often he will
be assessed is likely to influence the investment strategy of a manager. This
may not be in the funds best interests. For example, frequent monitoring can
encourage a short term approach to investment.
Cost: users of performance measurement services must balance the value of
the service against the cost. Also, for a number of assets (e.g. property),
valuation is difficult, time-consuming and very subjective. Detailed, frequent
calculations based on subjective valuations are inappropriate.
(iii)

(iv)

Index statistics will be influenced by:

Survivorship bias unlike in other asset classes, it is difficult to obtain


data on hedge fund failures when backfilling a history at the time an
index is launched by an index provider. This will create an upward bias.

Selection bias funds with a good history are more likely to apply for
inclusion at the time of reviewing index constituents. Similarly, it is not
always possible to obtain accurate performance information from a failing
hedge fund so the provider may only be able to exclude the fund rather
than report its full losses. Both of these factors will create an upward bias.

Marking to market bias where underlying securities are illiquid, funds


may use stale prices or mark asset values to a valuation model. Whilst
this does not necessarily result in a bias to the return data, it is likely to
result in lower volatility figures than would be the case otherwise.

S=

Rp r
p

where Rp is the return on the portfolio


r is the risk-free rate of return (usually taken to be 3 month LIBOR)
p is the standard deviation of portfolio returns
(v)

Page 4

The Sharpe ratio is widely understood by investors.


The Sharpe ratio can be calculated without reference to the beta of a
portfolio...
...making it more practical to compare strategies across different asset
classes...
...including asset classes where there may not be an obvious market portfolio
that can be used as a reference point.
Both the Jensen and the Treynor ratios rely on beta being a proxy for the level
of active risk taken by the asset manager.
[Credit was awarded for any other reasonable points made]

Subject ST5 (Finance and Investment Specialist Technical A) April 2009 Examiners Report

(vi)

The key limitations of the Sharpe ratio with regard to hedge fund returns are as
follows:

Mismatch with utility function the Sharpe ratio falls as downside or


upside volatility increase. In practice, investors will specifically be
concerned to avoid downside risks within their utility function.

Sampling issues historic calculations will be limited by the length of


available history. With hedge fund strategies, it is possible to employ a
strategy that appears low risk for an extended period but has a large tail
risk (e.g. selling volatility by writing out-of-the-money put options), and
the Sharpe ratio will not capture information about low frequency high
severity events.

Non-normal return distributions the Sharpe ratio will give a consistent


measure of excess return to risk for normal distributions, however standard
deviation is not a useful proxy for risk for all return distributions. This is
particularly the case for many hedge fund strategies.

All of these sources of distortion can result in sub-optimal investment


decisions.

(i)

(a)

The analysis should identify and analyse the key factors affecting the
future profitability of companies within the sector...
...and offer an outlook for the sector as a whole.
The analysis should enable the portfolio manager to form a view on the
attractiveness of the sector relative to other sectors...
...and also form a view on the relative attractiveness of individual
companies within the sector.
The analysis should also comment on the timescale over which
differences between perceived value and market prices might converge
(or if not, why they might persist)...
...and the recommendations should be justified by a combination of
numerical analysis and qualitative research...

(b)

The analysis should include historical statistics and forward-looking


estimates for several of the following factors to enable a picture of the
financial position of the companies to be built up:

Revenues
Operating profit
Pre-tax profit
Earnings per share
Price/earnings ratio
Price/book value
Dividend yield
Outstanding debt

Page 5

Subject ST5 (Finance and Investment Specialist Technical A) April 2009 Examiners Report

Where appropriate, the numerical analysis may need to be


supplemented by qualitative commentary to justify the
recommendations for each company.
(ii)

There should be additional commentary/analysis on the following:

(iii)

details of maturities of the companys existing bonds and loans


trends in the companys balance sheet over the next 5 years
trends in revenues and operating expenses [P&L alternatively] over the
next 5 years
the likelihood of the existing borrowings being refinanced in the current
climate without other compensating corporate actions
potential corporate actions that may need to be carried out to facilitate a
refinancing (e.g. disposals or rights issues to raise cash)
the likely change in financing costs as a result of the refinancing, based on
analysis of recent loan spreads for the sector and the economy
impact on credit rating
investor appetite for bonds/syndicated loans from this issuer

A new investor will not necessarily gravitate to the established global equity
fund, depending on his/her requirements.
Particular reasons why the new funds might offer a better fit for the investors
requirements could include one or more of the following factors:
Alpha based view
The investor believes that the fund manager has greater potential to deliver
alpha in the Lower Leverage or Higher Leverage niches than in the main fund
that invests in both categories of company.
This may reflect that the investor believes that a global approach to equity
investment is better suited to, for example, the Lower Leverage category,
where there are fewer country-specific factors that need to be accounted for in
selecting stocks.
Beta-based views

Page 6

Long term: the investor wishes to have a long-term bias towards Lower
Leverage or Higher Leverage companies.
Short-term: the investor believes that at the current point in the economic
cycle, Lower Leverage or Higher Leverage companies have better
prospects.
The above may reflect a risk-based view (e.g. more highly leveraged
companies have higher volatility) or a return-based view (e.g. more highly
leveraged companies will underperform at times of high interest rates /
high credit spreads).

Subject ST5 (Finance and Investment Specialist Technical A) April 2009 Examiners Report

Alternatively the above may reflect a preference to overweight particular


sectors (e.g. mining companies and oil companies tend to employ less
leverage than say financial companies) within the portfolio.
Where one of the two niche funds is being chosen based on beta-based
view (whether long-term or short-term), the particular product would only
be appropriate if the investor believes that the manager is capable of
delivering alpha in the chosen category (otherwise a passive strategy
would better suit the investor).

Portfolio construction/style diversification


The investor may be working around other equity styles within the existing
portfolio, and one of the two niche funds may be more appropriate (e.g. the
investor wishes to invest in a style neutral manner but feels that the Lower
Leverage fund will better complement an investment in a Global Equity
Growth fund offered by another manager).
The investor may also have adopted a risk-budgeting approach and one of the
two niche funds may be a more suitable addition to the portfolio from the
earmarked funds for investment. This would assume that the investor does not
have strong alpha and beta views, that would naturally lead to additional
adjustments elsewhere in the investors portfolio.

(iv)

At the launch date of the two niche funds it would appear that equal
investments in the two funds would in aggregate equal the existing fund.
However, the two approaches would begin to diverge almost immediately,
although not greatly as they are based on broadly the same stock/sector
selection decisions.
Differences would arise due to inflows and outflows from investors into the
different pooled funds, resulting in varying cash weightings and transaction
costs which would impact on the relevant fund.
Further differences will arise if a stock was reclassified as moving from the
Lower Leverage category to the Higher Leverage category (or vice versa).
This reflects that for the existing fund this would not lead to a buy/sell
decision (in the absence of other factors), whereas for the two niche funds one
would need to sell the stock and one would need to buy the stock (in the
absence of other factors). Crossing trades will mitigate against market impact
and transaction costs to the extent that the Lower Leverage and Higher
Leverage funds are making equivalent but opposite changes in a particular
stock.

(i)

A swap agreement in which the fixed rate receiver has the right to terminate
the swap on one or more dates prior to its scheduled maturity. This early
termination provision is designed to protect a party from adverse effects of
large changes in fixed rates.

(ii)

The pension fund wants to enter into swaps to reduce risk but the actual
liabilities are subject to refinement which might mean swaps adjustment.

Page 7

Subject ST5 (Finance and Investment Specialist Technical A) April 2009 Examiners Report

There is a yield pick up on the swap and therefore, is being held for tactical
reasons and not as a long term investment.
(iii)

Interest rate swap.

(iv)
Period

Number of days
in period

1
2
3
4
5
6

183
181
182
182
181
183

Annual
Forward
Interest
4.00%
4.25%
4.50%
4.75%
5.00%
5.25%

1/2 year
interest rate
2.03%
2.14%
2.28%
2.40%
2.51%
2.67%

value
term
payment
days

50000000
3
semi-annual arrears
360

(a)
PV
1
1/[1+(days/360*Interest)]
2 1/[1+(days/360*Interest) 2 periods]
3
etc.
4
etc.
5
etc.
6
etc.
Total

Discount
0.9801
0.9596
0.9382
0.9162
0.8938
0.8705
5.5584

PV of payments
996406
1025205
1067229
1100102
1123398
1161602
6473942

Notional
24910160
24122468
23716197
23160035
22467962
22125746
140502569

Candidates were given credit for rounded solutions rather than the
level of detail shown. Full marks were not available if candidates
assumed a half year rather than a specific day count.
(b)
PV of notional
PV of floating rate
Theoretical swap rate
(v)

140502569
6473942
4.61%

PV of fixed rate = 4.75 / 4.61 6473942 = 6670520


Profit = 6670520 6473942 = 196,578 (or the rounded equivalent)
The fixed rate is higher than the theoretical swap rate so assuming the
payments reflect the assumed then the pension fund will be in the money, i.e.
calculation should show a profit, as they have been paid a higher amount from
the bank (via fixed) than paid out.

Page 8

Subject ST5 (Finance and Investment Specialist Technical A) April 2009 Examiners Report

(vi)

The higher interest rates would mean the pension fund would be paying out
more than assumed and therefore, the profit assumed would be reduced or
turned into a loss (out of the money)

(i)

(a)

An option issued by a company. The holder has the right to purchase


shares at a specified price at specified times in the future.

(b)

Options can have more flexible exercise dates than warrants.


Warrants are new shares issued and therefore dilute share capital,
options are right to buy existing share capital.
Warrants are OTC where as options tend to be exchange traded.
Warrants have longer expiry than options.

(ii)

(a)

European right to purchase at set price at set date in future.


American right to purchase at set price at any point before expiry
date.

(b)
(iii)

American as it has the added flexibility.

The charts illustrate the basic shape of the payoff and credit was given for
similar, suitably annotated graphs
(a)

Butterfly spread

Investor does not believe a stock will rise or fall much before expiry
thinks volatility will be low. Wants limited risk strategy but also
limits profit.

Page 9

Subject ST5 (Finance and Investment Specialist Technical A) April 2009 Examiners Report

(b)

Straddle
Investor believes the underlying price will change significantly but
does not know which way it will go. Profit if volatility is high.

(c)

Bear spreads
A bear call spread is a limited profit, limited risk options trading
strategy that can be used when the options trader is moderately bearish
on the underlying security. Thinks the share price will fall.

(iv)

O St +K where St is greater than K, otherwise O. St = price of stock,


K = exercise price, O is price of option.

(v)

Chart would show 50 profit when share price starts at 0 until exercise price
1.50. The investor would then start to decrease the profit. At 2.00 exercise
the investor profit would be 0. At 2.50 the loss would be 50

Page 10

Subject ST5 (Finance and Investment Specialist Technical A) April 2009 Examiners Report

(vi)

0.75 = 50 profit,
1.50 = 50 profit assuming can buy stocks in market at zero cost
2.15 = 15 loss assuming can buy stocks at zero cost. Loss on purchase of
shares is 65 and profit from premium 50

(vii)

Initial margin is 0.2 50p 100 shares = 10. Then have to post 100% of the
movement which is 5p. The additional margin is then 5 for the 100 shares so
the total margin = 15.

(i)

The par yield is the coupon rate that would be required for a coupon-paying
bond to be valued at par under the current interest rate curve.

(ii)

1 = C2 (1 + zc1 )

+ (1 + C2 )(1 + zc2 )

(iii)
Term
1
2
3
4
(iv)

Par Yield
5.50%
5.40%
5.35%
5.30%

V(Bond)
100.000%
100.000%
100.000%
100.000%

V(Bond exc last)


94.882%
90.114%
85.679%

ZC Yield
5.500%
5.397%
5.346%
5.290%

Any three from:


Yield differences: in considering possible anomaly switches, yield differences
are widely used to identify individual bonds which seem cheap or dear, in
relation to other bonds. However, because of the fact that high coupon bonds
are likely to have higher gross yields than low coupons, a high gross yield
does not in itself indicate that a bond is cheap. The investor must examine
whether the yield difference is greater or less than it has been in the past.
A problem with the evaluation of individual bonds in relation to a fitted yield
curve has been the stability of the method used to fit the curve. It is now more
usual to review a computer generated history of yield spreads between pairs of
actual bonds.
Price ratios: these can be monitored as well as yield differences. Ideally, a
switch under consideration will look attractive, in relation to both yield and
price histories. A practical problem in using price ratios is that they do not
allow for the fact that the two bonds may have different coupons; they will
have different prices but will both be redeemed at 100. So the ratio of the two
prices will display a trend. This history of price ratios may be adjusted by this
trend to produce what are often known as stabilised price ratios.
Price models: some bond analysts have devised price models which try to
assess the correct price for a stock, given the key variables. A stocks price
is considered anomalous if the actual price differs from the price derived from
the model.
Page 11

Subject ST5 (Finance and Investment Specialist Technical A) April 2009 Examiners Report

Yield models: rather than compare a bonds yield with a redemption yield
curve it can be compared with one of the alternatives such as a yield surface or
par yield curve.

(i)

(a)

The primary purpose of the agreement is to act as a business contract


between the investment manager and the investor/client. As a
minimum it would set out the services to be carried out by the manager
for the investor and the agreed fees.

(b)

Typical restrictions would include:

(ii)

limitations on permitted asset classes for manager to invest in


limitations on leverage (explicit and implicit, through derivative
contracts)
maximum and minimum ranges for holdings in particular asset
classes
maximum ranges for holdings in a single company or single
industry sector
prohibitions on particular stocks for ethical/SRI reasons (e.g.
cluster bomb manufacturers)
prohibitions on self-investment in the investors own securities

A fund will often have a number of different managers and mandates


managing its assets. A prescriptive approach within manager agreements
allows the asset allocation to be controlled and managed at a global level,
whereas if the asset managers had complete discretion in security selection it
is unlikely that the actual assets would closely resemble the target asset
allocation.
Additionally, many funds will wish to place restrictions on permitted
investments and leverage as part of their overall risk controls and wider social
responsibilities.
Finally, a relatively prescribed agreement enables the investor to challenge the
manager more easily in the event of the assets not being invested in line with
the investors wishes, or in the event of mismanagement taking place.

END OF EXAMINERS REPORT

Page 12

Faculty of Actuaries

Institute of Actuaries

EXAMINATION
1 October 2009 (pm)

Subject ST5 Finance and Investment


Specialist Technical A
Time allowed: Three hours
INSTRUCTIONS TO THE CANDIDATE
1.

Enter all the candidate and examination details as requested on the front of your answer
booklet.

2.

You have 15 minutes before the start of the examination in which to read the
questions. You are strongly encouraged to use this time for reading only, but notes
may be made. You then have three hours to complete the paper.

3.

You must not start writing your answers in the booklet until instructed to do so by the
supervisor.

4.

Mark allocations are shown in brackets.

5.

Attempt all seven questions, beginning your answer to each question on a separate
sheet.

6.

Candidates should show calculations where this is appropriate.

AT THE END OF THE EXAMINATION


Hand in BOTH your answer booklet, with any additional sheets firmly attached, and this
question paper.

In addition to this paper you should have available the 2002 edition of the Formulae
and Tables and your own electronic calculator from the approved list.

ST5 S2009

Faculty of Actuaries
Institute of Actuaries

Describe the following terms:


(a)
(b)
(c)
(d)

split capital investment trust.


rights issue.
scrip issue.
share split.
[6]

(i)

Explain the relationship between forward and futures prices.

[5]

(ii)

(a)

Define the term basis risk.

(b)

Explain the reasons why basis risk may arise when a futures contract is
used to hedge a position in the cash market.
[3]

(iii)

State the formula for the optimal hedge ratio, defining the terms used.

(iv)

Outline why fixed income derivatives are more difficult to value than equity
derivatives.
[4]

(v)

Determine the price of a 10-month European call option on a 9.75 year bond
with a face value of 1,000. Assume that:

[2]

the current cash bond price is 1269


the strike price is 1300
the 10-month risk free interest rate is 2.6% p.a., and
the volatility of the forward bond price in 10 months is 9% p.a.

The bond pays a semi-annual coupon of 6% and coupon payments of 30 are


expected in 3 months and 9 months. Assume that the 3-month and 9-month
risk-free interest rates are 2% and 2.5% p.a. respectively.
[8]
[Total 22]

ST5 S20092

An investment consultant advises two pension funds that are both long-term investors
in separate global equity portfolios managed by Makeoff Global Investment
Company. Over the 12 months to 31 December 2008 the returns for the two clients
have been materially different. On further investigation, the investment consultant
obtains the following information:

Beta of portfolio
Holding in Banks
Investment Style
(i)

(ii)

Pension Fund A

Pension Fund B

0.8
4% underweight to benchmark
Value

1.2
10% overweight to benchmark
Growth

(a)

Define the term Beta.

(b)

Describe how the Betas quoted above will have impacted performance
over the period under review.
[3]
Explain what is meant by the terms Value and Growth.

(a)
(b)

Give an example of the type of shares that Value and Growth style
investors would invest in.
[4]

(iii)

Explain, using the information in the table above, which pension fund would
have been expected to have performed better during the period under review.
[3]

Another long-term investor follows the same investment strategy as Pension Fund B.
However, during the 12 months to 31 December 2008 they have experienced different
performance to Pension Fund B.

(iv)

State two reasons why the performances might be different.

(i)

Discuss the key factors to be considered in monitoring and controlling credit


risk.
[4]

(ii)

List the principal questions that a credit rating agency will ask in assessing and
ascribing an issuer rating for a company that issues debt.
[4]

(iii)

Explain why a bond issued by a company might have a higher or lower credit
rating than the company itself.
[2]
[Total 10]

ST5 S20093

[2]
[Total 12]

PLEASE TURN OVER

(i)

Describe the problems with, and the possible solutions to, the investment
technique known as liability hedging.

[8]

The trustees of a pension scheme with two sections (Section A and Section B) wish to
reduce the impact of interest rate changes on the amount of the difference between the
present value of the assets and the present value of the liabilities.
The table below shows the payments that are due to be paid out from each scheme
section and also those from a bond the trustees are thinking of purchasing to achieve
their investment objective.
Bond
Year (t) Cashflows
1
2
3
4
5

10
10
10
10
100

Section A
Liabilities
11
0
5
32
93

Section B
Liabilities
5
10
13
27
85

Assumptions

All payments are made annually in arrears.


Interest rate is 4.75% per annum.
All final calculations are rounded to nearest whole number.

(ii)

ST5 S20094

Assuming no other investments, state with reasons for which Section the bond
is better suited to achieve the trustees objectives. Show all your calculations.
[5]
[Total 13]

(i)

Describe the key features of a Real Estate Investment Trust (REIT).

[3]

REITs are relatively high-yield investments and a REIT must pay out at least 90% of
its taxable profit as a dividend to shareholders.
(ii)

Explain how you would expect the share price of a REIT to change with a rise
in interest rates.
[3]

You have been asked to assess the value of a possible REIT investment, Equity in
Property, which has a current market capitalisation of $8bn. You have been given the
following accounting information:

Rental income
Fee and asset management
Total Revenues

2008
2007
1,808,925 1,799,581
14,373
9,582
1,823,298 1,809,163

Property maintenance
498,608
464,981
Taxes and insurance
196,987
181,890
Property management
68,058
72,416
Fee and asset management
7,819
7,885
Depreciation
444,339
419,039
General and administration
38,810
46,492
Other costs
1,162
18,284
Total Expenses
1,255,783 1,210,987

(iii)

Operating Income
Net earnings

567,515
543,847

598,176
421,313

Capital Expenditures

181,948

156,776

Explain why traditional equity valuation metrics like the earnings-per-share


(EPS) ratio, earnings growth, and the price-to-earnings (P/E) multiple do not
apply.
[2]

You have proposed basing your valuation on a measure of Funds from Operations
(FFO), which excludes depreciation and the gains on sales of depreciable property.
(iv)

Calculate and reconcile FFO for each of the two years with net earnings.

[2]

Shareholders real estate holdings must be maintained (apartments must be regularly


redecorated, for example), so FFO is not quite the true residual cash flow remaining
after all expenses and expenditures.
(v)

Calculate an Adjusted FFO (AFFO) for each year as a better measure of


distributable income.
[1]

(vi)

Explain how you would use FFO and AFFO to value the proposed investment
in Equity in Property in order to recommend a purchase or not.
[6]
[Total 17]

ST5 S20095

PLEASE TURN OVER

Two investors have the same time horizon to complete the following trades.

Investor A trading 100m of equities.


Investor B trading 1bn of equities.

(i)

(a)

List four types of transaction costs.

(b)

Explain how these will differ between the two investors.


[4]

Another investor with 500m invested in equities believes equity markets will fall by
35% over the next 12 months. The general market consensus is markets will rise by
5% over the next 12 months.
(ii)

Set out three strategies that the investor could adopt to protect themselves
from a fall in equity markets.
[6]

(iii)

Explain the residual risks that remain with each strategy.

(iv)

Describe the effect adopting each strategy would have on the investors
investment performance if the equity markets increased by 5% over the next
12 months.
[6]
[Total 20]

END OF PAPER

ST5 S20096

[4]

Faculty of Actuaries

Institute of Actuaries

Subject ST5 Finance and Investment


Specialist Technical A
September 2009 examinations

EXAMINERS REPORT

Introduction
The attached subject report has been written by the Principal Examiner with the aim of
helping candidates. The questions and comments are based around Core Reading as the
interpretation of the syllabus to which the examiners are working. They have however given
credit for any alternative approach or interpretation which they consider to be reasonable.
R D Muckart
Chairman of the Board of Examiners
December 2009
Comments for individual questions are given with the solutions that follow.

Faculty of Actuaries
Institute of Actuaries

Subject ST5 (Finance and Investment Specialist Technical A) September 2009 Examiners
Report

1
a. An investment trust where the ordinary share capital consists of income
and capital shares. Holders of income get distributed income, holders of
capital little or no income but get residual value of assets after income
shares have been redeemed at fixed value.
b. Issue of further shares at a given price to existing shareholders in
proportion to their existing shareholdings. The purpose is for the issuing
company to raise more money.
c. Sometime called capitalisation or bonus issue is a further issue of new
shares (with the original nominal value) to existing shareholders in
proportion to their holdings. Reserves are capitalised to provide the
additional shareholders' equity.
d. Existing shares are split into two shares of half the original nominal
value. No new capital is raised and no reserves are capitalised.
[6]

2
(i) The main difference between (OTC) forwards and (exchange-traded) futures is
that, for a forward, there is no cash flow until the maturity. For a future, there
are daily marking-to-market and settlement of margin requirements.
If interest rates are constant then the values of the cash flows are equal and,
hence, the prices must also be equal. When interest rates vary unpredictably,
forward and futures prices are no longer the same because of the daily cash
flows from settlement and the interest earned on cash received (or paid on
borrowing). When the price of the underlying asset is strongly positively
correlated with interest rates, a long futures contract will be more attractive
than a similar long forward contract and futures prices will tend to be higher
than forward prices. The reverse holds true when the asset price is strongly
negatively correlated with interest rates.
The theoretical differences between forward and futures prices for contracts
that last only a few months are, in most circumstances, sufficiently small to be
ignored. However, for long-term futures contracts, the differences between
forward and futures rates are likely to become significant. To convert futures
to forward interest rates, a convexity adjustment is applied:
Forward rate = Futures rate

2t1t2

where t1 is the time to maturity of the futures contract, t2 is the time to


maturity of the rate underlying the futures contract and is the standard
deviation of the change in the short-term interest rate in one year. (A typical
value for is 1.2%). [Note that the forward and futures rates in this
expression are expressed in continuously compounded form .]
(ii)
a. Basis risk can be defined as the residual risk that results when the two
sides of a hedge do not move exactly together.

Page 2

Subject ST5 (Finance and Investment Specialist Technical A) September 2009 Examiners
Report

b. It may arise if:

The asset whose price is to be hedged is not exactly the same as the
asset underlying the futures contract

The hedger is uncertain as to the exact date when the asset will be
bought or sold.

The hedge requires the futures contract to be closed out well before
its expiration date.

(iii) The optimal hedge ratio, h, (ratio of the size of the position taken in futures
contracts to the size of the exposure) is given by:
h=

where
F

is the standard deviation of S, the change in spot prices

is the standard deviation in F, the change in futures price

and

is the correlation coefficient between S and F.

(iv) Fixed income derivative payoffs will be dependent in some way on the level of
interest rates. They are therefore more difficult to value than equity
derivatives, since:

The behaviour of an individual interest rate is more complicated than


that of a stock price.

For the valuation of many products, it is necessary to develop a model


describing the behaviour of the entire yield curve.

The volatilities of different points on the yield curve are different.

Interest rates are used for discounting as well as for determining


payoffs from the derivative.

(v) Assuming that the bond prices at the maturity of the option are log-normally
distributed, the value of the call option c is given by
c = P(0,t) [F0
where

(d1) X (d2)]

(x) is the standard cumulative Normal distribution function,

d1 = (ln (F0 / X) + ( 2T / 2) /

and

d2 = (ln (F0 / X) ( 2T / 2) /
F0 (the forward bond price) = (B0 I) / P(0,T)
where B0 is the bond price at time zero and
I is the present value of the coupons that would be paid during the life of the
option.
In this case, I = 30 e

0.25

0.02

+ 30e

Thus F0 = (1269 59.293) e0.8333

Page 3

0.75

0.026

0.025

= 59.293

= 1236.203

Subject ST5 (Finance and Investment Specialist Technical A) September 2009 Examiners
Report

Then d1

= (ln (1236.203 / 1300) + 0.092

10/24) / (0.09 (10/12) )

= ( 0.0503197 + 0.003375) / 0.0821583


= 0.57139
and d2

= ( 0. 0503197 0.003375) / 0.0821583


= 0.65355
0.8333

Hence, c = e

0.026

1236.203

= 0.97857 [(1236.203

0.57139) 1300

0.2839) (1300

0.65355)]

0.2567)]

= 16.83

3
(i)
a. Beta is a measure of a stock's volatility relative to movements in the
whole of the market and is thus a measure of systematic risk. It is usually
defined as the covariance of the return on the stock with the return on the
market, divided by the variance of the market return.
b. Pension Fund A would have been less volatile than the market, Pension
Fund B would have shown more volatility.
(ii)
a. Value investing is a style of investing based on picking shares that have
low valuations relative to their current profits, cash flows and dividend
yield. Value factors commonly analysed include:
Low

Book to Price

Earnings
Sales

Yield

to Price

Growth shares are shares with high price to book values. The expectation
is that earnings and profits will grow above average. Other factors
analysed include:
Sales

Growth

Return

on Equity

Earnings

Revisions

b. Growth internet/tech, clean tech


Value utilities, consumer staples
(iii) Performance over 12 months has been negative

Page 4

Subject ST5 (Finance and Investment Specialist Technical A) September 2009 Examiners
Report

Low beta expected to perform better as less volatile than high beta (everything
being equal)
Financials underperformed market in general so being underweight would be
better
Growth stocks tend to underperform value when markets are falling
Overall we would expect Pension A to perform better
(iv) Cashflows
Tax differences
Management Fee structure
Performance calculation in different base currencies
Credit was given for other sensible reasons

4
(i) The key factors in managing credit risk are:
the

creditworthiness of the counterparties with which an institution deals

the

total exposure to each counterparty

Creditworthiness of counterparties can be controlled by placing limits on the


credit ratings (as published by the major rating agencies) with which an
institution may deal. It can be also controlled in derivatives transactions by
dealing on a recognised exchange with a central clearing house which stands
as counterparty to all deals, rather than over-the-counter. The clearing house
will seek to protect itself by requiring the counterparties to deposit margin
with it. These margin payments are a particular example of the use of
collateral provided by a counterparty as a tool against credit risk.
It is important to monitor and place limits on the credit exposure to any single
counterparty. This will involve aggregating exposures in different areas. For
example a pension fund may hold both equity and debt issued by a bank as
well as having cash on deposit with the same bank and having them as a
counterparty to a derivatives deal. It will also be necessary to be aware of the
particular relationships between different companies within the same group.
Credit risk can be controlled by the use of Credit Default Swaps and other
credit derivatives.
(ii) Ratings agencies will seek to understand the following issues:
fundamental
competitive

Page 5

risks of the companys industry

position (relative to peers)

Subject ST5 (Finance and Investment Specialist Technical A) September 2009 Examiners
Report
downside risk
quality of
cash

vs. upside potential

profitability vs. EPS growth

flow generation vs. book profitability

forward

looking analysis

strategy,

management track record and risk appetite

capital

structure and financial flexibility

Specifically:
Purpose
What does the company do and why do they need to borrow? Possible reasons
for seeking finance include:
organic

growth

acquisition
investment
capital

in an associated company

expenditure

dividend

/ share buy-back

Payback
What is the expected source of repayment? Is there a secondary source? Issues
to consider include:
cash

flow / profit profile (over time)

possible

sale of assets and / or businesses

refinancing

Risks
What risks (quantitative and qualitative) could jeopardise debt servicing in
future? Factors to consider include:
macro

considerations (industry analysis and competitive trends,


regulatory environment, sovereign macro-economic analysis)

company specific issues

market position)
Structure

Page 6

(qualitative analysis, financial performance,

Subject ST5 (Finance and Investment Specialist Technical A) September 2009 Examiners
Report

Does the bond structure reflect the risks and protect investors interests?
(Structure, Status, Safeguards, Pricing)
(iii) A higher rating would apply where the bond has additional security relative to
an unsecured creditor of the issuer (e.g. a fixed or floating charge, or seniority
due to some other factor). [1; 1/2 if no example]
A lower rating would apply where the bond has weaker security relative to an
unsecured creditor of the issuer (e.g. the bond is subordinate to unsecured
creditors). [1; 1/2 if no example]

5
(i) Liability hedging is where the assets are chosen in such a way as to perform in
the same way as the liabilities. A specific example of this is the familiar
concept of immunisation, where assets are matched to liabilities by term in
order to hedge interest rate risk (to some degree). Other familiar forms of
hedging would include matching by currency and the consideration of the real
or nominal nature of liabilities when determining the choice of assets.
However, these examples relate only to specific characteristics of the
liabilities, whereas liability hedging aims to select assets which perform
exactly like the liabilities in all states.
The most familiar example would therefore be the choice of assets to hold in
order to hedge unit-linked liabilities.
In most cases the problem is solved by establishing a portfolio of assets,
determining a unit price by reference to the value of the asset portfolio, and
then using this price to value units held, allocated or realised.
However, even this simple approach can generate many practical problems
use of historic prices for transactions, moving between bid and offer pricing
bases, delays in notification of new money / withdrawals / units allocated or
realised.
A particular problem may arise when intermediaries are given delegated
authority to switch clients holdings between funds, which may result in
extreme volatility of movements for myriad small holdings.
A potentially greater problem arises when the assets held are not the same as
those underlying the value of the liabilities.
Thus, if units are allocated and realised by reference to some external fund,
then it is likely that the internal investment manager will not know what assets
are held by the external manager at any given point in time.
Alternatively, the requisite information may only be available after some
delay, by which time the assets actually held by the external manager are
likely to have changed.
An extreme example of this problem is where the value of liabilities is linked
to some external index (for example, guaranteed contracts where the

Page 7

Subject ST5 (Finance and Investment Specialist Technical A) September 2009 Examiners
Report

movement of market indices determines the value of the contract in some


way). In order to hedge such liabilities, use is often made of over-the-counter
derivatives purchased from an investment bank, thereby avoiding the
uncertainty (and expense) of rolling-over short term exchange traded
derivatives over the lifetime of the underlying contract.
Credit was given for other sensible issues discussed
(ii)

Year (t) Interest rate


1
1.0475
2
3
4
5

Year (t)
1
2
3
4
5

Year (t)
1
2
3
4
5

Bond
10
10
10
10
100

1st
condition
10
9
9
8
79
115

2nd
condition 3rd condition
10
10
18
36
26
78
33
133
396
1982
484
2240

Interest
rate
1.0475

1st
Liability A condition
11
11
0
0
5
4
32
27
93
74
115

2nd
condition
11
0
13
106
369
499

Interest
rate
1.0475

1st
Liability B condition
5
5
10
9
13
11
27
22
85
67
115

2nd
condition 3rd condition
5
5
18
36
34
102
90
359
337
1685
484
2187

Liability A fails at the second test of immunisation, Liability B matches all


three conditions.

6
(i) REITs work much like closed-end pooled funds, but instead of owning a
portfolio of securities, the REIT owns a portfolio of real estate properties
and/or mortgages.

Page 8

Subject ST5 (Finance and Investment Specialist Technical A) September 2009 Examiners
Report

REITs are registered securities and trade in the secondary market, like stocks.
As a result, investors get the benefit of diversification (since most REITs own
a large number of properties) and liquidity.
Unlike other pooled funds, REITs are permitted to use leverage the income
from the properties within the REIT is then used to pay the costs of any loans
involved.
There are two main types of REITs:
Equity REITs these invest mainly in actual real estate properties, such as
office buildings, residential property eg apartments, warehouses and shopping
centres. Equity REITs are usually not highly leveraged.
Mortgage REITs these invest mainly in mortgages and construction loans
for commercial properties and tend to use leverage to a greater degree than
equity REITs.
(ii) Total return from REIT is dividends plus price appreciation. Unlike other
quoted equities, most of the expected return of a REIT comes not from price
appreciation but from dividends.
On average, about two thirds of a REIT's return comes from dividends.
As a high-yield investment, a REIT can be expected to exhibit sensitivity to
interest rate changes.
Typically there is a strong inverse relationship between REIT prices and
interest rates.
On average, it would be safe to assume that interest rate increases are likely to
be met by REIT price declines although the actual change will vary by sector.
For example, some argue that in the case of residential and office REITs rising
interest rates would drive up REIT prices because increasing rates correspond
to economic growth and more demand.
However individual REITS may perform differently depending on their
underlying property exposures and degree of leverage.
(iii) From 2007 to 2008, Equity in Property's net income, or earnings grew by
almost 30% (+$122,500 to $543,847).
These net income numbers, however, include depreciation expenses, which are
significant line items.
For most businesses, depreciation is an acceptable non-cash charge that
allocates the cost of an investment made in a prior period.
But real estate is different than most fixed-plant or equipment investments in
that property rarely 'depreciates' in value (in the short term) as the result of
physical wear.
Net income, a measure reduced by depreciation, is therefore an inferior gauge
of performance and so valuation measures based on earnings are equally
flawed.

Page 9

Subject ST5 (Finance and Investment Specialist Technical A) September 2009 Examiners
Report

(iv) The general calculation involves adding depreciation back to net earnings
(since depreciation is not a real use of cash) and subtracting the gains on the
sales of depreciable property.
These gains are subtracted because we assume that they are not recurring and
therefore do not contribute to the sustainable dividend-paying capacity of the
REIT.
Hence the calculation and reconciliation of net income to FFO for EiP is:

Net earnings
Plus Depreciation
Gain on Depreciable Property
Sales
Other miscellaneous Depreciation
items and gains
FFO

2008
543847
444339
(300426)

2007
421313
419039
(102614)

69838

100651

757598

838389

Credit was given for appropriate description of the calculation, since the requisite
data was not provided in the question.
(v) FFO does not deduct for capital expenditures required to maintain the existing
portfolio of properties, hence the most important adjustment made to calculate
AFFO is the subtraction of capital expenditures.
FFO
Minus Capital Expenditures
AFFO

757598
(181948)
575650

838389
(156776)
681613

This number can be taken directly from the accounts as an estimate of the cash
required to maintain existing properties, although you could make a better
estimate by looking at the specific properties in the REIT.
(vi) Once we have the FFO and the AFFO, we can try to estimate the value of the
REIT.
The key assumption here is the expected growth in FFO or AFFO.
This involves analysing the underlying prospects of the REIT and its sector
exposure, considering:
Prospects

for rent increases

Prospects

to improve/maintain occupancy rates

specific plan to upgrade/upscale properties A popular and successful


tactic is to acquire low-end properties and upgrade them to attract a
higher quality tenant. Often a virtuous cycle ensues. Better tenants lead
to higher occupancy rates (fewer evictions) and higher rents.
growth prospects Many REITs favour fostering FFO growth
through acquisition, but it's easier said than done. An REIT must

External

Page 10

Subject ST5 (Finance and Investment Specialist Technical A) September 2009 Examiners
Report

distribute most of its profits and therefore does not have a lot of excess
capital to deploy. Many REITs, however, successfully prune their
portfolios: they sell underperforming properties to finance the
acquisition of undervalued properties.
The total return on a REIT investment comes from two sources: (1) dividends
paid and (2) price appreciation.
Expected price appreciation comprises two components:
1. Growth in FFO/AFFO
2. Expansion in the price-to-FFO or price-to-AFFO multiple
Given a market capitalisation of $8 billion, then:
Price/FFO = 8000/758 = 10.55x
Price/AFFO = 8000/575.7 = 13.9x
Interpreting price-to-FFO or price-to-AFFO multiples is not an exact science,
and the multiples will vary with market conditions and specific REIT subsectors (for example, apartments, offices, industrial).
Want to avoid buying into a multiple that is too high.
If you are looking at a REIT with favourable FFO/AFFO growth prospects,
then consider both sources together.
If FFO grows at 10%, for example, and the multiple of 10.55x is maintained,
then the price will grow 10%. But if the multiple expands about 5% to 11x,
then price appreciation will be approximately 15% (10% FFO growth + 5%
multiple expansion) making the current market valuation more attractive.
Debt is ignored by assuming that Equity in Property's debt burden is modest
and in line with the industry peers.
If EiPs leverage (debt-to-equity or debt-to-total capital) were above average,
we would need to consider the extra risk implied by the additional debt and
adjust the valuation accordingly.

7
(i)
a. Bid/offer spreads
Taxes
Market impact costs
Commission costs
Opportunity costs
There may be rebates payable if a Multilateral Trading Facility (MTF) is
used.
b. Trades are relatively small compared to market and you would have to
establish the names they are trading in, as larger trade might be highly
liquid where as small trade might be small cap

Page 11

Subject ST5 (Finance and Investment Specialist Technical A) September 2009 Examiners
Report

Everything being equal (timing and stocks traded), bid/offer slightly


higher on 1bn trade
Taxes the same (proportionally)
Market impact higher on 1bn trade
Commission depends but might be lower on larger trade
Opportunity costs will depend on trading time etc. Would be
proportionally equal if traded together
(ii) Sell the equities
Short equity futures
Buy puts set at a level investor is willing to see market value decrease
Use Total Return swaps.
(iii) Sell equities market risk, if equities rise then miss out on the upside, risks
sell at the wrong time
Short equity futures investment performance risk, basis risk
Buy puts investment performance risk, might not be able to buy puts for all
shares in portfolio.
Swaps investment performance risk.
For all derivative-based strategies, counterparty / default risk is a further issue
when over the counter approaches are used.
(iv) Sell equities Would be in cash. Mismatch performance between cash and the
equity markets. Lose out on dividends.
Short equity futures would lose out on market performance would still pick
up alpha from mismatch of futures and underlying portfolio. Could under or
out perform depending on alpha.
Buy puts depends the levels that they are set at. There would be the negative
drag on paying for puts. However, for some shares in the portfolio that fall in
value then could positively impact overall performance as cap losses on those
securities.
Swaps Mismatch performance between the equity market and the other side
of the swap.

END OF EXAMINERS REPORT

Page 12

Potrebbero piacerti anche